<Назад
Помощник Главного обвинителя от США У. Уолш представляет сборник документов Т: «Преследование евреев».Продолжение от 13 декабря 1945 г.
Это полный текст многодневного выступления (13.12.1945—14.12.1945).
13
Помощник Главного обвинителя от США У. Уолш представляет сборник документов Т: «Преследование евреев». (13.12.1945)
2857
Майор Уильям Уолш (помощник обвинителя от Соединённых Штатов): От имени обвинения Соединенных Штатов Америки я представляю Трибуналу документальные доказательства, подтверждающие положения обвинительного заключения, изложенные в разделе первом — военные преступления и преступления против человечности, и, в соответствии с соглашением, достигнутым между обвинителями, сформулированные в пункте четвертом, параграфе Х (В), преступления против человечности. Представляемые мною документы относятся к обвинению в преследовании евреев.
MAJOR WILLIAM F. WALSH(Assistant Trial Counsel for the United States): If the Tribunal please, on behalf of the United States Counsel, I now present to this august Tribunal the evidence to establish certain phases of the Indictment alleged in Count One under War Crimes and Crimes against Humanity, and by agreement between the prosecutors the allegations in Count Four, Paragraph X(B), Crimes against Humanity. The topical tine of this presentation is «The Persecution of the Jews.»
COMMANDANT WALSH(substitut du Procureur Général américain): Plaise au Tribunal. Je vais maintenant exposer les preuves fondant la partie de l’Acte d’accusation relative aux crimes de guerre et aux crimes contre l’Humanité, sous le chef d’accusation nº 1, et, en accord avec le Ministère Public sous le chef d’accusation nº 4, paragraphe X (B), crimes contre l’Humanité. Le titre de cet exposé est: «La persécution des Juifs. «
MAJOR WILLIAM B. WALSH(HILFSANKLÄGER FÜR DIE VEREINIGTEN STAATEN): Hoher Gerichtshof! Im Namen des amerikanischen Anklage vertreters lege ich jetzt diesem erhabenen Gerichtshof das Beweismaterial für bestimmte Taten vor, die in der Anklageschrift im Anklagepunkt 1 unter: Kriegsverbrechen und Verbrechen gegen die Menschlichkeit enthalten sind. Ferner auf Grund eines Übereinkommens der Anklagevertreter die Anschuldigung gemäß Punkt 4, Absatz X (B), Verbrechen gegen die Menschlichkeit. Der Titel dieser Darlegung lautet: «Die Verfolgung der Juden».
58
Я приобщаю в качестве доказательства документальную книгу переводов помеченную буквой «Т». Эти документы находятся в книге согласно Д, Л, ПС и Р; и согласно сериям переводов они приводятся в номерном порядке. Заголовок «преследование евреев» явно неуместен в свете доказательств, которые будут представлены. В теории слово «преследовать» означает причинять страдания, беспокоить, досаждать. Однако этот термин не выражает действительного существа преступлений нацистов, так как конечной целью они провозгласили уничтожение еврейской расы.
At this time I offer in evidence a Document Book of translations, lettered «T.» These documents contained in the books are arranged according to the D-, L-, PS-, and R-series; and under the series the translations are listed numerically. This title, «The Persecution of the Jews,» is singularly inappropriate when weighed in the light of the evidence to follow. Academically, I am told, to persecute is to afflict, harass, and annoy. The term used does not convey, and indeed I cannot conjure a term that does convey the ultimate aim, the avowed purpose to obliterate the Jewish race.
Je dépose comme preuve le livre de documents traduits «T «. Les documents sont rangés en séries: D, L, PS et R; dans chaque série, les documents sont rangés dans l’ordre numérique. Ce titre: Persécution des Juifs, est singulièrement impropre si on le considère à la lumière des preuves qui vont suivre. Étymologiquement, persécuter signifie: affliger, tourmenter, harceler. Le terme ne convient pas et je ne puis en trouver un qui stigmatise l’intention fondamentale, le but avoué de rayer du monde la race juive.
Ich lege jetzt als Beweismaterial ein Dokumentenbuch von Übersetzungen vor, das mit «T» bezeichnet ist. Die in diesem Buch enthaltenen Urkunden sind gemäß den D-, L-, PS — und R-Serien geordnet, und innerhalb dieser Serien sind die Übersetzungen den Nummern nach gelegt. Der Titel «Die Verfolgung der Juden» ist völlig unzureichend, wenn man ihn im Lichte des folgenden Beweismaterials betrachtet. Akademisch bedeutet das Wort «verfolgen», soviel ich weiß, quälen, belästigen und beunruhigen. Der hier gebrauchte Ausdruck beschreibt in keiner Weise auch nur annähernd das – und ich weiß keinen Ausdruck, der dieses letzte Ziel kennzeichnen würde – den eingestandenen Zweck, die jüdische Rasse auszulöschen.
59
Я не собираюсь полностью перечислять здесь все преступления, совершенные ими в отношении евреев. Масштабы совершенных преступлений были столь колоссальны, что охватили всю германскую нацию, ее народ, ее организации.
This presentation is not intended to be a complete recital of all the crimes committed against the Jews. The extent and the scope of the crimes was so great that it permeated the entire German nation its people and its organizations.
Cet exposé n’a pas la prétention d’être un récit complet de tous les crimes commis contre les Juifs; ils sont si nombreux, si divers, que la nation allemande tout entière en est marquée dans son peuple et dans ses organisations.
Diese Darstellung ist nicht als eine vollkommene Aufzählung aller Verbrechen gegen die Juden gedacht. Ausmaß und Reichweite dieser Verbrechen waren so groß, daß sie die ganze deutsche Nation, ihre Bevölkerung und Organisationen durchdrangen.
60
Меня будут дополнять те из моих коллег, которые выступят вслед за мною по другим разделам обвинения. Доказательства, относящиеся к партийным и государственным организациям, преступность которых обвинение будет стремиться установить, покажут их роль и участие в подготовке и планировании уничтожения евреев.
I am informed that others to follow me will offer additional evidence under other phases of the Prosecution's case. Evidence relating to the Party organizations and state organizations, whose criminality the Prosecution will seek to establish, will disclose and emphasize the part that these organizations played in the pattern and plan for annihilation.
Les orateurs qui me succéderont compléteront l’exposé des preuves en traitant d’autres parties de l’accusation; les documents relatifs aux organisations du Parti et à celles de l’État, dont le Ministère Public tentera d’établir la culpabilité, révéleront quel rôle ces organisations ont joué dans ces plans d’extermination.
Ich bin davon unterrichtet, daß andere, die nach mir sprechen werden, zusätzliches Beweismaterial bei anderen Teilen des Anklagevorbringens vorlegen werden. Das Beweismaterial, das sich auf die Partei — und Staatsorganisationen bezieht, deren verbrecherischer Charakter die Anklagebehörde festzustellen bemüht ist, wird die von den Organisationen in der Planung der Vernichtung gespielte Rolle enthüllen und hervorheben.
61
Французский и советский обвинители также располагают документальными доказательствами по этому вопросу, они в свое время представят их в ходе процесса.
The French and the Soviet Prosecutors, too, have a volume of evidence all related to this subject, which will be submitted in the course of the Trial.
Le Ministère Public français et le Ministère Public soviétique ont eux aussi un volume de preuves se rapportant toutes à cette question et ils le soumettront en temps voulu.
Die französischen und russischen Anklagevertreter haben ebenfalls je einen Band Beweismaterial, das sich auf dieses Thema bezieht und im Verlaufe des Prozesses zur Vorlage kommen wird.
62
Прежде чем я начну перечислять действия, ведущие к уничтожению евреев, я хотел бы показать, что эти действия, а также политика, которая проводилась внутри Германии, начиная с 1933 года до конца войны, относятся к планированию, подготовке, развязыванию и ведению агрессивных войн и, таким образом, подпадают под определение преступлений против человечности, как это указано в п. «с» статьи 6 Устава.
Before I begin a recital of the overt acts leading to the elimination of the Jews, I am prepared to show that these acts and policies within Germany from the year 1933 to the end of the war related to the planning, preparation, initiation, and waging of aggressive wars, thus falling within the definition of Crimes against Humanity as defined in Article 6(c) of the Charter.
Avant de commencer le récit des actes manifestement accomplis dans le but de supprimer les Juifs, je montrerai que la politique poursuivie à l’intérieur de l’Allemagne depuis 1933 jusqu’à la fin de la guerre était liée aux plans de préparation et d’exécution de guerres d’agression, ce qui concorde avec la définition des crimes contre l’Humanité donnée à l’article 6,c du Statut.
Bevor ich jedoch mit der Verlesung der offenkundigen Handlungen, die zur Vernichtung der Juden führten, beginne, möchte ich zeigen, daß diese Handlungen und diese Politik innerhalb Deutschlands von 1933 bis zum Kriegsende mit der Planung, Vorbereitung, dem Beginn und der Durchführung von Angriffskriegen zusammenhingen und damit unter den Anklagepunkt: Verbrechen gegen die Menschlichkeit fallen, wie sie im Artikel 6 (c) des Statuts definiert sind.
63
С давних пор в Германии существовало убеждение, что первая мировая война окончилась для нее поражением потому, что Германия потерпела крах на внутреннем фронте. Вследствие этого при планировании новых войн было решено, что необходимо предотвратить возможность повторения на внутреннем фронте катастрофы 1918 года. Для успешного планирования и ведения войны надо было добиться консолидации германского народа по принципу нацистов: «Одна раса, одно государство, один фюрер».
It had long been a German theory that the first World War ended in Germany's defeat because of a collapse in the zone of the interior. In planning for future wars it was determined that the home front must be secure to prevent a repetition of this 1918 debacle. Unification of the German people was essential to successful planning and waging of war, and the Nazi political premise must be established-»One race, one state, one Fuehrer.»
On a longtemps soutenu en Allemagne la thèse que la perte de la première guerre mondiale avait été due à un écroulement de l’arrière. Les plans pour la guerre future garantissaient la sécurité du front intérieur dans le but de prévenir une répétition de la débâcle de 1918. L’unification du peuple allemand était une condition essentielle de la réussite des plans et des opérations de guerre, et la base de la politique nazie était: «Un peuple, un Reich, un Führer».
Seit Jahren war es eine deutsche Theorie, daß der erste Weltkrieg mit der deutschen Niederlage endete, weil es innerhalb des Landes zum Zusammenbruch gekommen sei. Bei der Planung zukünftiger Kriege beschloß man daher, daß man die Heimatfront sicher in der Hand haben müßte, um einer Wiederholung des Debakels von 1918 vorzubeugen. Die Einigung des deutschen Volkes war für die erfolgreiche Planung und Durchführung des Krieges wesentlich, und der politische Grundsatz der Nazis: «Ein Volk, ein Reich, ein Führer» mußte festgelegt werden.
64
Поэтому было решено ликвидировать свободные профсоюзы, запретить политические партии (кроме национал-социалистской), отменить гражданские свободы и уничтожить всякую оппозицию. Было объявлено, что приверженность богу, церкви и научной истине несовместима с нацистским режимом. Расовая антиеврейская политика была частью этого плана консолидации, так как нацисты были уверены, что евреи не только не будут способствовать военной программе Германии, а, напротив, будут ей мешать. Поэтому евреи должны быть уничтожены.
Free trade unions must be abolished, political parties Other than the National Socialist Party) must be outlawed, civil liberties must be suspended, and opposition of every kind must be swept away Loyalty to God, church, and scientific truth was declared to be incompatible with the Nazi regime. The anti-Jewish policy was part of this plan for unification because it was the conviction of the Nazis that the Jews would not contribute to Germany's military program, but on the contrary would hamper it. The Jew must therefore be eliminated.
Il fallait supprimer les syndicats libres; les partis politiques (autres que le parti nazi) devaient être mis hors la loi, les libertés civiques devaient être supprimées et toute opposition devait être brisée. La loyauté envers Dieu et l’Église, la vérité scientifique, furent déclarées incompatibles avec le régime nazi. La politique antijuive faisait partie de ce plan d’unification, car les nazis étaient convaincus que les Juifs ne contribueraient pas au programme militaire de l’Allemagne, et même qu’ils le contrecarreraient. Il fallait donc supprimer les Juifs.
Die freien Gewerkschaften mußten abgeschafft, politische Parteien außer der Nationalsozialistischen Partei verboten, die bürgerlichen Freiheiten aufgehoben und jede Art von Opposition weggefegt werden. Gottesfurcht, der Glaube an die Kirche und Achtung vor der wissenschaftlichen Wahrheit wurden als nicht vereinbar mit dem Nazi-Regime erklärt. Die antijüdische Politik war ein Teil dieses Vereinheitlichungsplanes, weil die Nazis davon überzeugt waren, daß die Juden das deutsche Militärprogramm nicht unterstützen, sondern im Gegenteil es hindern würden. Darum mußte der Jude ausgerottet werden.
65
Эта позиция находит ясное подтверждение в документе ПС-1919, США-170. Это стенограмма речи Гиммлера на совещании генералитета СС 4 октября 1943 года. Зачитаю краткий отрывок:
This view is clearly borne out try a statement contained in Document 1919-PS, Exhibit USA-170. This document is a transcript of a Himmler speech at a meeting of the SS major generals on 4 October 1943, and from Page 4, Paragraph 3, of the translation before the Court, I read a very short passage:
Ceci ressort clairement d’une déclaration contenue dans le document PS-1919 (USA-170) qui est la sténographie d’un discours fait par Himmler à une réunion de généraux SS tenue le 4 octobre 1943. Je lis un bref passage du paragraphe 3 de la page 4 de la traduction:
Dieser Standpunkt geht deutlich aus einer Erklärung hervor, die in 1919-PS, US-170, enthalten ist. Dieses Dokument ist die Niederschrift einer Himmler-Rede bei einer Besprechung der SS-Gruppenführer am 4. Oktober 1943. Von Seite 4, Absatz 3, der dem Gerichtshof vorliegenden Übersetzung lese ich einen sehr kurzen Absatz:
66
Нам известно, насколько усложнили бы мы для себя ситуацию, если бы в условиях бомбардировок, тягот и лишений войны у нас бы сегодня в каждом городе имелись евреи. Они выступали бы в качестве тайных саботажников, агитаторов, возмутителей спокойствия. Мы бы теперь, вероятно, дошли бы до уровня 1916—1917 годов, когда евреи все еще присутствовали в германском национальном организме».
We know how difficult we should have made it for ourselves if with the bombing raids, the burdens and deprivations of war, we still had Jews today in every town as secret saboteurs, agitators, and trouble mongers; we would now probably have reached the 1916–17 stage when the Jews were still in the German national body.
Nous savons combien notre position serait difficile si, en plus des raids aériens, des fardeaux et des privations de la guerre, nous avions encore des Juifs aujourd’hui dans toutes les villes pour jouer le rôle de saboteurs, d’agitateurs et de fauteurs de trouble; nous aurions probablement atteint le niveau de 1916–1917, alors que les Juifs faisaient encore partie de la nation allemande.
... wir wissen, wie schwer wir uns täten, wenn wir heute noch in jeder Stadt, – bei den Bombenangriffen, bei den Lasten und bei den Entbehrungen des Krieges, – noch die Juden als Geheimsaboteure, Agitatoren und Hetzer hätten. Wir würden wahrscheinlich jetzt in das Stadium des Jahres 1916/17 gekommen sein, wenn die Juden noch im deutschen Volkskörper säßen.
67
В связи с этим обращение с евреями в Германии рассматривалось как составная часть планирования и подготовки агрессивной войны, так же как производство вооружения и воинская повинность. На этом основании обвинение в преследовании евреев подлежит рассмотрению Трибуналом как составная часть планирования и подготовки агрессивной войны.
The treatment of the Jews within Germany was therefore as much of a plan for aggressive war as was the building of armaments and the conscription of manpower. It falls within the jurisdiction of this Tribunal as an integral part of the planning and preparation to wage a war of aggression.
Le traitement des Juifs à l’intérieur de l’Allemagne faisait donc partie du plan de la guerre d’agression au même titre que la fabrication d’armes et la mobilisation de la main-d’œuvre. Cette question, en tant que partie intégrante des plans et préparatifs du déclenchement d’une guerre d’agression, est du ressort du Tribunal.
Die Behandlung der Juden in Deutschland war deshalb ebenso wie die Herstellung von Waffen und die Wehrpflicht ein Teil des Planes für den Angriffskrieg. Sie fällt in die Gerichtsbarkeit dieses Gerichtshofs als integrierender Bestandteil des Planes und der Vorbereitung zur Führung eines Angriffskrieges.
68
Очевидно, что преследования и убийства евреев начиная с 1939 года на захваченных территориях Европы являются и военными преступлениями, как это определено статьей 6 Устава. Эти преступления являются нарушением статьи 46 Гаагской Конвенции 1907 года, которая также была подписана Германией. Я сейчас зачитаю статью 46 и прошу Трибунал принять ее без доказательства:
It is obvious that the persecution and murder of Jews throughout the conquered territories of Europe following 1939 are War Crimes as defined by Article 6(b)of the Charter. It further violates Article 46 of the Regulations of the Hague Convention of 1907, to which Germany was a signatory. I quote Article 46 and ask the Court to take judicial notice thereof:
Il est clair que les persécutions et les meurtres dont furent victimes les Juifs de tous les pays d’Europe après 1939 sont des crimes de guerre tels que les définit l’article 6,b du Statut. C’est de plus une violation de l’article 46 de la Convention de La Haye de 1907, à laquelle l’Allemagne avait apposé sa signature. Je cite cet article 46 et je demande au Tribunal de lui accorder une valeur probatoire:
Es liegt auf der Hand, daß die Verfolgung und Ermordung von Juden in sämtlichen besetzten Gebieten Europas nach 1939 Kriegsverbrechen darstellen, wie sie im Artikel 6 (b) des Statuts definiert sind. Außerdem verstößt sie gegen Artikel 46 des Haager Abkommens von 1907, das auch von Deutschland unterschrieben worden war. Ich zitiere Artikel 46 und ersuche den Gerichtshof, davon amtlich Kenntnis zu nehmen.
69
Должны уважаться права и честь семьи, человеческая жизнь и частная собственность, так же как религиозные убеждения и исполнение религиозных обрядов».
Family honor and rights, the lives of persons, and private property, as well as religious convictions and practices, must be respected.
Les droits et l’honneur des familles, la vie des personnes et la propriété privée, ainsi que les convictions et les habitudes religieuses, doivent être respectés.
Die Ehre und die Rechte der Familie, das Leben der Bürger und das Privateigentum, sowie die religiösen Überzeugungen und gottesdienstlichen Handlungen sollen geachtet werden.
70
Я не знаю других преступлений в истории человечества, которые были бы более ужасными, чем обращение с евреями. Я намереваюсь доказать, что нацистская партия поставила перед собой цель, позднее нашедшую отражение в политике германского государства, о которой часто открыто заявляли подсудимые, — уничтожить еврейский народ. Я постараюсь избежать соблазна, как бы велик он ни был, истолковывать документы или делать какие-либо выводы. Я предпочитаю, чтобы документы говорили сами за себя в своем неприкрашенном реализме. Возможно, кровожадность могла играть некоторую роль во всех этих жесточайших преступлениях, но главной целью уничтожения еврейской расы было осуществление одного из основных пунктов нацистского плана — подготовки и ведения агрессивной войны. С этого момента я ограничу представление доказательств лишь теми, которые относятся к действиям, доказательства по которым еще не представлены, но я прошу Трибунал быть снисходительным, если окажется необходимым для полноты картины ссылаться на уже ранее представленные документы и доказательства.
I know of no crime in the history of mankind more horrible in its details than the treatment of the Jews. It is intended to establish that the Nazi Party precepts, later incorporated within the policies of the German State, often expressed by the defendants at bar, were to annihilate the Jewish people. I shall seek to avoid the temptation to editorialize or to draw inferences from the documents, however great the provocation; rather I shall let the documentary evidence speak for itself — its stark realism will be unvarnished. Blood lust may have played some part in these savage crimes, but the underlying purpose and objective to fixate the Jewish race was one of the fundamental principles of the Nazi plan to prepare for and to wage aggressive war. I shall from this point limit my proof to the overt acts committed; but I dare to request the Court's indulgence, if it is necessary in weaving the pattern of evidence, to make reference to certain documents and evidence previously submitted.
Je ne connais pas, dans l’Histoire humaine, de crimes plus horribles dans leurs détails que la façon dont les Juifs furent traités. Je peux prouver que les préceptes du parti nazi, qui prirent corps ensuite dans la politique de l’État allemand et furent fréquemment exprimés à la barre par les accusés, visaient à la destruction du peuple Juif. Je m’efforcerai de ne pas céder à la tentation de citer trop de documents, bien que cette tentation soit forte; je laisserai plutôt les preuves parler elles-mêmes, je n’adoucirai pas la violence de leur réalisme. Le plaisir de tuer peut avoir joué un rôle dans ces crimes sauvages, mais leur vrai but, leur raison d’être, était la destruction de la race juive, l’un des principes fondamentaux du plan nazi pour la préparation et le déclenchement d’une guerre d’agression. Je me bornerai à la mention des actes manifestes qui furent commis, mais je demande l’indulgence du Tribunal au cas où il me semblerait nécessaire pour l’établissement des preuves de me référer à certains documents déjà déposés.
Ich kenne kein Verbrechen in der Geschichte der Menschheit, das in seinen Einzelheiten fürchterlicher ist, als die Behandlung der Juden. Wir beabsichtigen zu beweisen, daß die Gebote der Nazi-Partei, die später in die Politik des deutschen Staates übernommen und von den Angeklagten oft ausdrücklich bestätigt wurden, zum Ziele hatten, das jüdische Volk zu vernichten. Ich werde mich bemühen, der Versuchung zu widerstehen, Kommentare zu geben oder Schlußfolgerungen aus den Dokumenten zu ziehen, wie groß der Anreiz dazu auch sein mag. Ich ziehe vor, die urkundlichen Beweisstücke in ihrer reinen Wirklichkeit für sich selbst sprechen zu lassen. Blutdurst mag bei diesen wüsten Verbrechen eine gewisse Rolle gespielt haben; aber der Zweck und das Ziel, die zugrunde lagen, nämlich die jüdische Rasse zu vernichten, war eines der grundlegenden Prinzipien des Nazi-Planes für die Vorbereitung und Führung des Angriffskrieges. Ich werde von jetzt an meine Beweisführung auf die begangenen offenkundigen Handlungen beschränken, aber ich bitte den Gerichtshof um Nachsicht, falls es bei der Zusammenstellung des Beweismaterials notwendig sein sollte, auf bestimmte früher vorgelegte Dokumente Bezug zu nehmen.
71
Окончательной целью было уничтожить евреев. Однако этого нельзя было выполнить без предварительных мероприятий. Сначала нацистской партии надо было захватить власть в германском государстве, показать свою силу перед лицом мирового общественного мнения и привить приведенному в повиновение германскому народу чувство ненависти к евреям.
Now this ultimate objective, that is, the elimination and extermination of the Jews, could not be accomplished without preliminary steps and measures. The German State must first be seized by the Nazi Party, the force of world opinion must be faced, and even the regimented German people must be indoctrinated with hatred against the Jews.
Pour atteindre cet objectif, il fallait des mesures préliminaires. Le parti nazi devait tout d’abord s’emparer du pouvoir en Allemagne, il fallait faire face à l’opinion mondiale, et il fallait endoctriner le peuple allemand et lui inculquer la haine du Juif.
Dieses letzte Ziel, nämlich die Beseitigung und Ausrottung der Juden, konnte nicht ohne vorbereitende Schritte und Maßnahmen ausgeführt werden. Zu erst mußte der deutsche Staat von der Nazi-Partei übernommen werden, der Macht der Weltmeinung mußte ins Auge gesehen werden, und selbst dem gedrillten deutschen Volk mußte der Haß gegen die Juden als Doktrin eingeimpft werden.
72
Первое доказательство политики фашистской партии в отношении евреев содержится в программе партии, принятой в феврале 1920 года. Я приобщаю в качестве доказательства документ 1708-ПС «Программа национал-социалистической партии», экземпляр США-255, и я хочу привести имеющую к делу часть этой программы, параграф (4):
The first clear-cut evidence of the Party policies concerning the Jews was expressed in the Party program in February 1920. I offer in evidence Document 1708-PS, «Program of the National Socialist Party,» Exhibit USA-255. With the Court's permission, I would like to quote the relevant part of that program, Paragraph (4):
La politique du Parti à l’égard des Juifs se révéla clairement pour la première fois dans le programme du Parti de février 1920. Je dépose ce document PS-1708 (USA-255): programme du parti national-socialiste. Que le Tribunal me permette de citer les passages de ce programme qui nous intéressentParagraphe 4.
Der erste klare Beweis für die Nazi-Politik in Bezug auf die Juden ist im Parteiprogramm vom Februar 1920 zu finden. Ich lege als Beweismittel Dokument 1708-PS, US-255, das Programm der NSDAP vor. Mit Erlaubnis des Gerichtshofs möchte ich den bezüglichen Teil dieses Programms zitieren: Punkt vier:
73
Только тот, кто принадлежит к германской расе, может быть гражданином. Только тот принадлежит к германской расе, в чьих жилах течет германская кровь, независимо от вероисповедания...
Only a member of the race can be a citizen. A. member of the race can only be one who is of German blood without consideration of confession....
Seul un membre de la race peut être citoyen. N’est membre de la race que le sujet de sang allemand, quelle que soit sa confession.
Staatsbürger kann nur sein, wer Volksgenosse ist, Volksgenosse kann nur sein, wer deutschen Blutes ist, ohne Rücksicht auf die Konfession....»
2974
БиддлМогу я вас прервать на секунду. Довольно трудно понять где эти экземпляры и из какого тома вы цитируете.
THE TRIBUNAL(Mr. Biddle): May I interrupt a minute. It is a little hard to know where these exhibits are or what volume you are now quoting from.
MR. BIDDLE Darf ich Sie einen Augenblick unterbrechen. Es ist etwas schwer zu finden, wo sich diese Beweisstücke befinden und aus welchem Bande Sie jetzt vorlesen.
3075
УолшСэр, это 1708-ПС.
MAJOR WALSH This, Sir, is 1708-PS.
MAJOR WALSH Aus Dokument 1708-PS.
3176
Биддл Второй том?
THE TRIBUNAL(Mr. Biddle): Volume 2?
MR. BIDDLE Band zwei?
3277
Уолш Второй том.
MAJOR WALSH Volume 2.
MAJOR WALSH Band zwei.
MAJOR WALSH Punkt 4 und 6 auf der ersten Seite. Punkt vier:
3378
Биддл А какая страница экземпляра?
THE TRIBUNAL(Mr. Biddle): And what page of that exhibit?
MR. BIDDLE Und von welcher Seite dieses Beweisstücks?
3479
Уолш Это параграф (4) и параграф (6) сэр, на первой странице.
MAJOR WALSH That is Paragraph (4) and Paragraph (6), Sir, on the first page.
COMMANDANT WALSH «En conséquence, aucun Juif ne peut être membre de la race.»
80
Параграф (4):
Paragraph (4):
81
Только тот, кто принадлежит к германской расе, может быть гражданином. Только тот принадлежит к германской расе, в чьих жилах течет германская кровь, независимо от вероисповедания... Следовательно, ни один еврей не может принадлежать к германской расе...
Only a member of the race can be a citizen. A member of the race can only be one who is d German blood, without consideration of confession. Consequently, no Jew can be a member of the race.
Staatsbürger kann nur sein, wer Volksgenosse ist. Volksgenosse kann nur sein, wer deutschen Blutes ist, ohne Rücksicht auf Konfession. Kein Jude kann daher Volksgenosse sein.
82
И параграф (6):
And again, in Paragraph (6):
Puis au paragraphe 6:
Und dann Punkt 6:
83
Право решать вопросы, касающиеся администрации и закона, принадлежит только гражданину. Поэтому мы требуем, чтобы на все должности во всех государственных учреждениях любого рода, будь то в империи, земле или муниципалитете, назначались только граждане.
The right to determine matters concerning administration and law belongs only to the citizen; therefore, we demand that every public office of any sort whatsoever, whether in the Reich, the county, or municipality, be filled only by citizens.
Le droit de s’occuper d’affaires administratives et juridiques n’appartient qu’au citoyen; en conséquence, nous demandons que tout service public du Reich, d’une province ou d’une ville, n’emploie que des citoyens.
Das Recht, über Führung und Gesetze des Staates zu bestimmen, darf nur dem Staatsbürger zustehen. Daher fordern wir, daß jedes öffentliche Amt, gleichgültig welcher Art, gleich ob im Reich, Land oder Gemeinde, nur durch Staatsbürger bekleidet werden darf.
84
Представляю документ ПС-2662 под номером США — 256 — книга «Майн кампф». На страницах 724—725 этой книги Гитлер говоря о евреях, говорил что для выполнения своей задачи национал-социалистское движение, я цитирую:
I now offer Document 2662-PS, Mein Kampf, Exhibit Number USA-256. On Pages 724–725, Hitler, in this book, speaking of the Jew, said that if the National Socialist movement was to fulfill its task-and I quote:
Je dépose maintenant le document PS-2662,Mein Kampf, (USA-256). Aux pages 724 et 725 de ce livre, Hitler parle des Juifs et dit que, pour atteindre son but, le mouvement national-socialiste...
Und nun lege ich Dokument 2662-PS, US-256, «Mein Kampf», vor. Auf den Seiten 724 und 725 schrieb Hitler, von den Juden sprechend, wenn die nationalsozialistische Bewegung ihre Aufgabe erfüllen sollte, und ich zitiere:
85
Должно открывать глаза народам в отношении зарубежных наций и снова и снова напоминать им об истинном враге нынешнего мира. Вместо ненависти к арийцам — с которыми нас может разделять почти всё, однако с которыми мы связаны одинаковой кровью или тугим узлом общей культуры, должно направлять гнев на зловещего врага человечества как истинную причину всех страданий.
It must open the eyes of the people with regard to foreign nations and must remind them again and again of the true enemy of our present-day world. In the place of hate against Aryans-from whom we may be separated by almost everything but to whom, however, we are tied by common blood or the great tie of a common culture-it must dedicate to the general anger the evil enemy of mankind as the true cause of all suffering.
… doit ouvrir les yeux du peuple sur les nations étrangères et lui rappeler sans cesse quel est l’ennemi réel de notre monde contemporain. Au lieu de haïr les Aryens — dont bien des choses peuvent nous séparer, mais auxquels nous sommes liés toutefois par la communauté de sang ou de culture — il faut réserver sa colère à l’adversaire malfaisant de l’Humanité, cause profonde de toute souffrance.
Sie muß dem Volke die Augen öffnen über die fremden Nationen und muß den wahren Feind unserer heutigen Welt immer und immer wieder in Erinnerung bringen. An Stelle des Hasses gegen Arier, von denen uns fast alles trennen kann, mit denen uns jedoch gemeinsames Blut oder die große Linie einer zusammengehörigen Kultur verbindet, muß sie den bösen Feind der Menschheit, als den wirklichen Urheber allen Leides, dem allgemeinen Zorne weihen.
86
Однако, следует понимать, что по крайней мере в нашей стране его признают как смертельного врага и что борьба против него, может показать, путеводный луч в лучшую эпоху, также и для других наций, дорогу к спасению в борьбе за арийское человечество».
It must see to it, however, that at least in our country he be recognized as the most mortal enemy and that the struggle against him may show, like a flaming beacon of a better era, to other nations, too, the road to salvation for a struggling Aryan mankind.
Mais il doit faire en sorte que, tout au moins dans notre pays, l’ennemi mortel soit démasqué et que la lutte engagée contre lui soit le signe d’une ère plus lumineuse et plus sereine, et éclaire pour les autres peuples aussi, dans une humanité aryenne combattante, la route du Salut.
Sorgen aber muß sie dafür, daß wenigstens in unserem Lande der tödlichste Gegner erkannt und der Kampf gegen ihn als leuchtendes Zeichen einer lichteren Zeit auch den anderen Völkern den Weg weisen möge zum Heil einer ringenden arischen Menschheit.
87
Целый поток гнусной литературы всех видов и для людей всех возрастов издавался и распространялся по всей Германии.Показательным в этом плане является книга под названием «Der Giftpilz». Я приобщаю в качестве доказательства документ 1778-ПС, экземпляр номер США-257. Эта книга навешивает на еврея ярлык гонителя рабочего класса, осквернителя расы, дьявола в человеческом обличье, ядовитого гриба и убийцы. Эта книга инструктировала школьников распознавать еврея по карикатурным физическим чертам, показанным на страницах 6 и 7; учила их тому, что евреи домогаются мальчиков и девочек, на странице 30; и что еврейская библия допускает любые преступления, страницы 13—17. Газета подсудимого Штрайхера «Der Sturmer» номер 14, от апреля 1937, в частности, доходила до таких крайностей как заявления о том евреи на пасху убивали христиан.
A flood of abusive literature of all types and for all age groups was published and circulated throughout Germany. Illustrative of this type of publication is the book entitled Der Giftpilz. I offer in evidence Document 1778-PS, Exhibit Number USA-257. This book brands the Jew as a persecutor of the labor class, as a race defiler, devil in human form, a poisonous mushroom, and a murderer. This particular book instructed school children to recognize the Jew by caricature of his physical features, shown on Pages 6 and 7; taught them that the Jew abuses little boys and girls, on Page 30; and that the Jewish Bible permits all crimes, Pages 13–17. The Defendant Streicher's periodical Der Sturmer, Number 14, April 1937, in particular, went to such extremes as to publish the statement that Jews at the ritual celebration of their Passover slaughtered Christians.
Un flot de littérature injurieuse de tout genre, destinée aux gens de tous âges, fut publié et mis en circulation en Allemagne; un exemple typique en est le livre intituléDer Giftpilz (Le champignon vénéneux). Je présente comme preuve ce document PS-1778 (USA-257). Ce livre stigmatise le Juif comme persécuteur des classes laborieuses, démon sous forme humaine, champignon vénéneux et assassin. Ce livre était destiné aux enfants des écoles: il leur apprenait à reconnaître le Juif grâce à des caricatures de ses caractéristiques physiques (figurant aux pages 6 et 7), leur enseignait que le Juif abuse des petits garçons et des petites filles (à la page 30), et que la Bible juive autorise tous les crimes (pages 13 à 17). Le périodique de l’accusé Streicher,Der Stürmer, numéro du 14 avril 1937, en particulier, alla même jusqu’à publier la déclaration selon laquelle les Juifs, à la célébration rituelle de l’Exode, tuaient les Chrétiens.
Ein Strom von Schmutzliteratur aller Art und für alle Altersgruppen wurde in Deutschland herausgegeben und in Umlauf gesetzt. Bezeichnend für diese Art von Veröffentlichungen ist ein Buch mit dem Titel «Der Giftpilz».Ich bringe es als Beweismaterial, Dokument 1778-PS, US-257, zur Vorlage. Dieses Buch bezeichnet den Juden als einen Verfolger der Arbeiterklasse, als Rassenschänder, als einen Teufel in Menschengestalt, als Giftpilz und Mörder. In diesem Buch wurde den Schulkindern durch Karikaturen, die auf Seite 6 und 7 erscheinen, gelehrt, wie sie den Juden an seiner Körperbeschaffenheit erkennen könnten. Auf Seite 30 wird gezeigt, daß der Jude kleine Jungen und Mädchen mißbraucht, und auf Seite 13 bis 17, daß die jüdische Bibel alle Verbrechen erlaubt. Nummer 14, vom April 1937, Seite 6 der Wochenschrift «Der Stürmer», herausgegeben von dem Angeklagten Streicher, ging in ihren Übertreibungen so weit, daß sie behauptete, die Juden würden bei der Ritualfeier des Passahfestes Christen abschlachten.
88
Я приобщаю документ 2699-ПС, экземпляр номер США-258. На странице 2, столбец 1, абзацы с 6 по 9, я цитирую:
I offer Document 2699-PS, Exhibit Number USA-258. On Page 2, Column 1, Paragraphs 6 to 9, I quote:
Je présente le document PS-2699 (USA-258), page 2, première colonne, paragraphes 6 à 9. Je cite:
Ich lege Dokument 2699-PS, US-258, vor. Von Seite 2, Spalte 1, zitiere ich die Absätze 6 bis 9:
89
Также многие верования евреев показывают, что совершение ритуальных убийств является законом талмудического еврея. Бывший главный раввин (и позже монах) Теофити заявил о том, что ритуальные убийства проводятся на еврейский пурим (в память об убийстве персов) и пасху (в память убийства Христа). Правила следующие:
Also the numerous confessions made by the Jews show that the execution of ritual murders is a law of the Talmud Jew. The former chief Rabbi (and later monk) Teofiti declares that the ritual murders take place especially on the Jewish Purim (in memory of the Persian murders) and Passover (in memory of the murder of Christ). The rules are as follows:
Bien des Juifs ont confessé que la loi du Talmud exige des meurtres rituels. Le Grand Rabbin Jeofiti, qui devint moine plus tard, déclare que les meurtres rituels ont lieu particulièrement au «Purim Juif» (en mémoire des meurtres de Perse), et à «l’Exode» (en mémoire de la mort du Christ). Les instructions sont les suivantes:
Auch aus vielen Geständnissen der Juden geht hervor, daß die Ausübung von Ritualmorden dem Talmudjuden Gesetz ist. Der ehemalige Oberrabbiner und spätere Mönch Teofiti erklärt zum Beispiel, daß die Ritualmorde vornehmlich anläßlich des jüdischen Purimfestes (Erinnerung an den Persermord) und des Passahfestes (Erinnerung an den Christusmord) ausgeführt werden.
90
Кровь жертв следует добывать силой. На пасху её добавляют в вино и мацу. Так, небольшим количеством крови окропляют закваску для мацы и вино. Смешивание проводит глава еврейской семьи.
The blood of the victims is to be tapped by force. On Passover it is to be used in wine and matzos. Thus a small part of the blood is to be poured into the dough of the matzos and into the wine. The mixing is done by the head of the Jewish family.
La victime doit être saignée. À l’Exode, on se sert du sang pour le vin et le pain azyme: un peu de ce sang doit être versé dans la pâte du pain azyme et dans le vin. C’est le père de famille juif qui fait ce mélange. Il opère de la façon suivante:
Die Vorschriften sind folgende: Den Opfern ist das Blut mit Gewalt abzuzapfen. Es soll beim Passahfest im Wein und in den Mazzen Verwendung finden. Das heißt: ein kleiner Teil des Blutes ist in den Mazzenteig und in den Wein zu schütten. Die Beimischung geschieht durch den jüdischen Familienvater. Der Vorgang ist folgendermaßen:
91
Процедура следующая: глава семьи наливает несколько капель свежей и засоленной крови в стакан, погружает в неё пальцы левой руки и окропляет (благословляет) всех сидящих за столом. Глава семьи произносит: «Сим просим бога наслать 10 проклятий на всех врагов еврейской веры». Потом они едят и в конце глава семьи восклицает: «Пусть сгинут неверные, как и дитя, чья кровь находится в хлебах и вине».
The procedure is as follows: The family head empties a few drops of the fresh and powdered blood into a glass, wets the fulgers of the left hand with it and sprays (blesses) with it everything on the table. The head of the family then says, 'Thus we ask God to send the 10 plagues to all enemies of the Jewish faith.' Then they eat, and at the end the head of the family exclaims, 'May all Gentiles perish, as the child whose blood is contained in the bread and wine.'
Le chef de famille vide quelques gouttes de sang frais, ou du sang séché, dans le verre, en mouille les doigts de sa main gauche et le répand en bénédiction sur tout ce qui se trouve sur la table. Puis il dit: «Ainsi demandons-nous à Dieu d’envoyer les Dix Plaies à tous les ennemis de la loi juive». Ensuite, ils mangent et, à la fin du repas, le chef de famille s’exclame: «Puissent tous les Gentils périr comme a péri l’enfant dont le sang est contenu dans ce pain et ce vin».
Der Familienvater schüttet einige Tropfen des frischen und gepulverten Blutes in das Glas, tunkt den Finger der linken Hand hinein und besprengt (segnet) damit alles, was auf dem Tisch steht, worauf der Familienvater spricht: ›Also bitten wir Gott, daß er die zehn Plagen senden möge allen Feinden des jüdischen Glaubens.‹ Hierauf speisen sie und der Familienvater ruft zum Schluß: ›Also (wie das Kind, dessen Blut in Brot und Wein enthalten) mögen alle Gojim untergehen!‹
92
Свежая (или высушенная и засоленная кровь) убитого в дальнейшем используется еврейскими супругами, беременными еврейками, при обрезании и тому подобном. Ритуальное убийство признаётся всеми талмудическими евреями. Еврей верит, что этим он снимает с себя грехи».
The fresh (or dried and powdered) blood of the slaughtered is further used by young married Jewish couples, by pregnant Jewesses, for circumcision and so on. Ritual murder is recognized by all Talmud Jews. The Jew believes he absolves himself thus of his sins.
Le sang frais (ou séché, ou en poudre) de ceux qui ont été immolés est utilisé par les couples de jeunes mariés juifs, par les juives enceintes, et pour la circoncision, etc. Tous les Juifs talmudiques reconnaissent le meurtre rituel. Le Juif croit se laver de ses péchés en agissant ainsi.
Weiter findet das frische Blut (oder das getrocknete und zu Pulver verriebene Blut) der Geschächteten Verwendung für junge verheiratete jüdische Ehepaare, für schwangere Jüdinnen, für die Beschneidung usw. Der Ritualmord wird von allen Talmudjuden anerkannt. Der Jude glaubt, sich damit zu entsühnen.
93
Нам трудно понять, как такое вранье смогло пасть на плодородную почву, что образованная нация нация могла читать, воспринимать или верить этим доктринам. Однако, нам следует понимать, что пресса находилась под строгим контролем, что предвосхищало появление такой лживой пропаганды, в которую могли поверить невежественные и ведомые.
It is difficult for our minds to grasp that falsehoods such as these could fall on fertile soil, that a literate nation could read, digest, or believe these doctrines. We must realize, however, that with a rigidly controlled press which precluded an expose of such lying propaganda, some of the ignorant and gullible would be led to believe.
Il nous est difficile de comprendre comment des mensonges tels que celui-ci pouvaient trouver audience et comment une nation cultivée pouvait lire, assimiler ou croire de telles doctrines. Nous devons réaliser néanmoins que, avec une presse strictement contrôlée qui excluait l’exposé de propagande mensongère, les crédules et les ignorants aient été amenés à y ajouter foi.
Es ist uns schwer verständlich, daß derartige falsche Darstellungen auf fruchtbaren Boden fallen konnten, und daß eine gebildete Nation derartige Lehren lesen, annehmen und glauben konnte. Wir müssen aber daran denken, daß die strenge Pressekontrolle eine Bloßlegung dieser Lügenpropaganda ausschloß, so daß einige Unwissende oder Leichtgläubige dazu gebracht werden konnten, dies zu glauben.
94
Документ ПС-2697 — экземпляр «Дер Штюрмер», представленный под номером США-259. Он издавался подсудимым Штрайхером. В этом издании Штрайхер говоря о еврейской вере сказал: «Священное писание это ужасный криминальный роман переполненный убийством, инцестом, обманом и бесчестьем».
I now offer in evidence Document 2697-PS, a copy of Der Sturmer, Exhibit Number USA-259. This publication, Der Sturmer, was published by the Defendant Streicher's publishing firm. In this publication, Streicher, speaking of the Jewish faith, said, «The Holy Scripture is a horrible criminal romance abounding with murder, incest, fraud, and indecency.»
Je dépose maintenant comme preuve le document PS-2597, un numéro deDer Stürmer, nº USA-259. Ce journal,Der Stürmer, était publié par la maison d’édition de l’accusé Streicher. En parlant de la foi juive, Streicher y disait: «La Sainte Écriture est un horrible roman criminel où fourmillent le meurtre, l’inceste, la fraude et l’indécence. «
Ich lege nun Dokument 2779-PS vor, eine Ausgabe des «Stürmers», US-259. Diese Zeitung «Der Stürmer» wurde von der Verlagsfirma des Angeklagten Streicher herausgegeben. In dieser Zeitung sagte Streicher über den jüdischen Glauben: «... mutet das Geschichtsbuch der Juden, das man als Heilige Schrift zu bezeichnen sich angewöhnt hat, wie ein einziger schauerlich-schauriger Kriminalroman an.... Von Mord und Blutschande, Betrug, Diebstahl und Sittlichkeitsverbrechen wimmelt es in diesem heiligen Buch geradezu...»
95
И далее он снова говорил: «Талмуд это величайшая еврейская книга с преступными инструкциями о том как жить еврею каждый день». Это имеется в документе 2698-ПС, «Der Sturmer», который я приобщаю в качестве доказательства как экземпляр номер США-260.
And again he said, «The Talmud is the great Jewish book of criminal instructions that the Jew practices in his daily life.» This is contained in Document 2698-PS, Der Sturmer, which I now offer in evidence, Exhibit Number USA-260.
Il disait encore: «Le Talmud est le grand livre des crimes que le Juif commet dans sa vie quotidienne.» Ce dernier passage est contenu dans le document PS-2598,Der Stürmer, que je dépose maintenant comme preuve sous le nº USA-260.
Weiter sagt er: «Der Talmud ist das große jüdische Verbrecherlehrbuch, das der Jude täglich im praktischen Leben anwendet.» Letzteres steht in dem Dokument 2698-PS, «Der Stürmer», das ich ebenfalls zum Beweis vorlegen möchte. Es ist dies Beweisstück US-260.
96
«Пропагандистская кампания ненависти была слишком масштабной и оскорбительной, чтобы рассматривать её дальше. В документах представленных в качестве доказательств и на других этапах дела будут обнаружены похожие и даже более оскорбительные заявления, многих подсудимых и их сообщников. Когда нацисты захватили в Германии власть, они получили новое и ужасное орудие против евреев, а именно возможность использовать против них государственную власть. Это делалось путем издания декретов.
This propaganda campaign of hate was too widespread and notorious to require further elaboration. Within the documents offered in evidence in this and in other phases of the case will be found similar and even more scurrilous statements, many by the defendants themselves and others by their accomplices. When the Nazi Party gained control of the German State, a new and terrible weapon against the Jews was placed within their grasp, the power to apply the force of the state against them. This was done by the issuance of decrees.
Cette propagande de haine était trop répandue et trop connue pour qu’on l’approfondisse davantage. Tout au cours du procès on trouvera dans les preuves des documents semblables et même plus injurieux, émanant des accusés eux-mêmes ou de leurs complices. Lorsque le parti nazi se fut assuré le contrôle de l’État allemand, une arme nouvelle et terrible contre les Juifs fut placée entre leurs mains: le pouvoir de disposer contre eux de la force de l’État. La voie d’exécution en était la promulgation des décrets.
Dieser Propagandafeldzug des Hasses war zu ausgebreitet und zu allgemein bekannt, als daß noch Beschreibungen nötig wären. Unter den Urkunden, die zu diesem wie auch zu anderen Teilen des Tatbestands als Beweismittel vorgelegt wurden, findet man ähnliche und sogar noch schmutzigere Behauptungen, von denen viele von den Angeklagten selbst, andere von ihren Mitschuldigen stammen. Als die Nazi-Partei die Kontrolle über den Staat erlangte, wurde eine neue und furchtbare Waffe gegen die Juden in ihre Hände gelegt, nämlich die Anwendung der Macht des Staates. Dies geschah durch die Herausgabe von Verordnungen.
97
Так, еврейские иммигранты были лишены прав гражданства («Рейхсгезетцблатт», 1933 год, часть 1, страница 480). Соответствующий декрет был подписан подсудимыми Фриком и Нейратом.
Jewish immigrants were denaturalized: 1933 Reichgesetzblatt, Part I, Page 480, signed by Defendants Frick and Neurath.
Des immigrants juifs furent dépouillés de leur nationalité: 1933,Reichsgesetzblatt, première partie, page 480; décret signé par les accusés Frick et Neurath.
Eingewanderte Juden wurden ausgebürgert: Reichsgesetzblatt 1933, Teil I, Seite 480, unterzeichnet von den Angeklagten Frick und Neurath.
98
Евреям, родившимся в Германии, не разрешалось иметь германское гражданство («Рейхсгезетцблатт», 1935 год, часть 1, страница 1146), подписано подсудимыми Фриком.
Native Jews were precluded from citizenship: 1935 Reichsgesetzblatt, Part I, Page 1146, signed by Defendant Frick.
Les Juifs de naissance perdirent leurs droits de citoyen: 1935,Reichsgesetzblatt, première partie, page 1146; décret signé par l’accusé Frick.
In Deutschland geborenen Juden wurde die Staatsangehörigkeit entzogen: Reichsgesetzblatt 1935, Teil I, Seite 1146, unterzeichnet von dem Angeklagten Frick.
99
Евреям запрещалось состоять в браке или вступать во внебрачные отношения с лицами германской крови. Декрет подписан подсудимым Фриком и Гессом.
Jews were forbidden to live in marriage or to have extramarital relations with persons of German blood: 1935 Reichspesetzblatt, Part I, Page 1146, signed by Frick and Hess.
Il fut interdit aux Juifs de se marier ou d’avoir des relations extra-matrimoniales avec des personnes de sang allemand: 1935,Reichsgesetzblatt, première partie, page 1146; décret signé par Frick et Hess.
Juden durften mit Personen deutschen Blutes weder verheiratet sein noch außereheliche Verbindungen haben: Reichsgesetzblatt 1935, Teil I, Seite 1146, unterzeichnet von den Angeklagten Frick und Heß.
100
Евреи лишались права голоса («Рейхсгезетцблатт», 1936 год, часть 1, страница 133). Декрет подписан подсудимым Фриком.
Jews were denied the right to vote: 1936 Reichsgesetzblatt Part I, Page 133, signed by Defendant Frick.
Les Juifs se virent refuser le droit de vote: 1936,Reichsgesetzblatt, première partie, page 133; décret signé par l’accusé Frick.
Juden wurde das Wahlrecht aberkannt: Reichsgesetzblatt 1936, Teil I, Seite 133, unterzeichnet von dem Angeklagten Frick.
101
Евреям запрещалось занимать официальные должности или состоять на государственной службе («Рейхсгезетцблатт», 1933 год, часть 1, страница 277) подписан подсудимым Фриком.
Jews were denied the right to hold public office or civil service positions: Reichsgesetzblatt 1933, Part I, Page 277, signed by Defendant Frick.
Les Juifs se virent refuser le droit d’occuper des postes officiels ou administratifs: 1933,Reichsgesetzblatt, première partie, page 277; décret signé par l’accusé Frick.
Juden konnten keine öffentlichen Ämter oder Staatsstellen bekleiden: Reichsgesetzblatt 1933, Teil I, Seite 277, unterzeichnet von dem Angeklagten Frick.
102
Было решено низвести евреев на низшую ступень общественной лестницы путем лишения их общих прав и свобод. Так, им был запрещен доступ в определенные районы города, им запрещалось пользоваться тротуарами, транспортом, увеселительными заведениями и ресторанами («Рейхсгезетцблатт», 1938 год, часть 1, страница 1676).
It was determined to relegate the Jews to an inferior status by denying them common privileges and freedoms. Thus, they were denied access to certain city areas, sidewalks, transportation, places of amusement, restaurants: 1938 Reichsgesetzblatt, Part I, Page 1676.
Décision fut prise de donner aux Juifs une situation subalterne en leur refusant les privilèges et les libertés normales. Puis on leur interdit l’accès de certains quartiers de la ville; certains trottoirs, moyens de transport, lieux de distraction et restaurants leur furent interdits: 1938,Reichsgesetzblatt, première partie, page 1676.
Es wurde beschlossen, die Juden auf ein niedrigeres Niveau herabzudrücken, indem man ihnen die allgemeinen Rechte und Freiheiten entzog. Gewisse Stadtteile und Straßen, Verkehrsmittel, Unterhaltungsstätten und Gaststätten waren ihnen verboten: Reichsgesetzblatt 1938, Teil I, Seite 1676.
103
Постепенно применялись все более строгие меры, вплоть до запрета заниматься частной практикой. («Рейхсгезетцблатт», 1939 год, часть 1, страница 47). Декрет подписан подсудимым Гессом.
Progressively more and still more stringent measures were applied, even to the denial of private pursuits. They were excluded from the practice of dentistry: 1939 Reichsgesetzblatt, Part I, Page 47, signed by Defendant Hess.
Petit à petit, des mesures de plus en plus sévères furent prises pour leur interdire même l’accès de carrières indépendantes de l’État. Ils furent exclus de la profession de dentiste: 1939,Reichsgesetzblatt, première partie, page 47; décret signé par l’accusé Hess.
Mit der Zeit wurden immer strengere Maßnahmen angewandt, die soweit gingen, daß man ihnen die Ausübung ihrer privaten Beschäftigung untersagte. Sie durften den zahnärztlichen Beruf nicht mehr ausüben: Reichsgesetzblatt 1939, Teil I, Seite 47, unterzeichnet von dem Angeklagten Heß.
104
Запрещалось заниматься юриспруденцией («Рейхсгезетцблатт», 1938 год, часть 1, страница 1403). Декрет подписан подсудимым Фриком и Гессом.
The practice of law was denied: 1938 Reichspesetzblatt, Part I, Page 1403, signed by Defendants Frick and Hess.
L’exercice des professions juridiques leur fut refusé: 1939,Reichsgesetzblatt, première partie, page 969; décret signé par les accusés Frick et Hess.
Sie wurden vom Rechtsanwaltsberuf ausgeschlossen: Reichsgesetzblatt 1938, Teil I, Seite 1403, unterzeichnet von den Angeklagten Frick und Heß.
105
Запрещалась врачебная практика («Рейхсгезетцблатт», 1938 год, часть 1, страница 969). Декрет подписан подсудимым Фриком и Гессом.
The practice of medicine was denied: 1938 Reichsgesetzblatt, Part I, Page 969, signed by Defendants Frick and Hess.
Sie durften ihrer ärztlichen Praxis nicht mehr nachgehen: Reichsgesetzblatt 1938, Teil I, Seite 969, unterzeichnet von Frick und Heß.
106
Запрещалось заниматься делами печати и радио («Рейхсгезетцблатт», 1933 год, часть 1, страница 661).
They were denied employment by press and radio: 1933 Reichsgesetzblatt, Part I, Page 661.
Ils ne furent plus autorisés à occuper un emploi dans la presse et la radio: 1939,Reichsgesetzblatt, première partie, page 661.
Sie durften weder in der Presse noch am Rundfunk tätig sein: Reichsgesetzblatt 1933, Teil I, Seite 661.
107
Запрещалось участвовать в биржевых операциях и быть биржевыми маклерами («Рейхсгезетцблатт», 1934 год, часть 1, страница 169).
From stock exchanges and stock brokerage: 1934 Reichspesetzblatt, Part I, Page 169.
Ils furent évincés de la Bourse et des affaires de change: 1934,Reichsgesetzblatt, première partie, page 169,
Sie wurden von der Börse und dem Aktienhandel ausgeschlossen: Reichsgesetzblatt 1934, Teil I, Seite 169;
108
Запрещалось заниматься даже сельским хозяйством («Рейхсгезетцблатт», 1933 год, часть 1, страница 685).
And even from farming: 1933 Reichsgesetzblatt, Part I, Page 685.
Et même de l’agriculture: 1933;Reichsgesetzblatt, première partie, page 685.
und sogar von der Landwirtschaft: Reichsgesetzblatt 1933, Teil I, Seite 685.
109
In 1938 they were excluded from business in general and from the economic life of Germany: 1938 Reichsgesetzblatt, Part I, Page 1580, signed by the Defendant Goering.
En 1938, ils furent exclus des affaires en général et de la vie économique de l’Allemagne: 1938, Reichsgesetzblatt, première partie, page 1580; décret signé par l’accusé Göring.
Im Jahre 1938 wurden sie aus dem allgemeinen Geschäfts — und Wirtschaftsleben Deutschlands ausgeschaltet: Reichsgesetzblatt 1938, Teil I, Seite 1580, unterzeichnet von Göring.
110
Евреев заставляли платить большие налоги и огромные штрафы. Их дома, банковские счета, недвижимая собственность и ценности — все было экспроприировано.
The Jews were forced to pay discriminatory taxes and huge atonement fines. Their homes, bank accounts, real estate, and intangibles were expropriated.
Les Juifs étaient obligés de payer des impôts particuliers et des amendes énormes. Leurs maisons, leurs comptes en banque, leurs biens immobiliers et leurs biens insaisissables furent confisqués.
Die Juden wurden gezwungen, nachteilig hohe Steuern und hohe Sühnestrafen zu zahlen; ihre Heimstätten, Bankkonten, Grundstückbesitze usw. wurden ihnen genommen.
111
Документ ПС-1816, США-261, представляет собой стенограмму совещания под председательством подсудимого Геринга, на котором среди прочих присутствовал подсудимый Функ. Совещание состоялось 12 ноября 1938 г. в 11.00 в имперском министерстве авиации. Приводятся слова подсудимого Геринга:
To digress for a moment from a recital of decrees and to refer specifically to the atonement fines, I wish to offer Document 1816-PS, Exhibit Number USA-261. This exhibit is a stenographic report of a conference under the chairmanship of the Defendant Goering, attended by the Defendant Funk among others, held at 11 o'clock on 12 November 1938 at the Reich Ministry for Air. From Pages 8 and 9 of Section 7, I quote the Defendant Goering:
J’arrête cette énumération de décrets pour parler spécialement de ces amendes; je dépose le document PS-1816 (USA-261). C’est un compte rendu sténographié d’une conférence présidée par l’accusé Göring, assisté de l’accusé Funk, entre autres, et tenue le 12 novembre 1938, à 11 heures, au ministère de l’Air du Reich. Je cite les paroles de l’accusé Göring, aux pages 8 et 9, section 7:
Um einen Augenblick von der Aufzählung der Verordnungen abzugehen, und um besonders auf die Sühnestrafen zu sprechen zu kommen, möchte ich Dokument 1816-PS, US-261, unterbreiten. Es handelt sich hier um einen stenographischen Bericht über eine Sitzung unter Leitung des Angeklagten Göring, an der unter anderen der Angeklagte Funk teilgenommen hat, und die am 12. November 1938, um 11.00 Uhr, im Reichsluftfahrtministerium abgehalten worden ist. Von Seite 8 und 9 des siebenten Abschnitts zitiere ich den Angeklagten Göring:
112
...Еще один вопрос, господа. Что бы вы сказали, если бы я сегодня заявил, что еврейство в качестве наказания должно уплатить один миллиард...
One more question, gentlemen, what would you think the situation would be if I announced today that Jewry shall have to contribute this 1,000,000,000 as a punishment.
Encore une question, Messieurs; comment pensez-vous que se présenterait la situation si je vous annonçais aujourd’hui que la juiverie doit payer une amende d’un milliard?
Noch eine Frage meine Herren! Wie beurteilen Sie die Lage, wenn ich heute verkünde, daß dem Judentum als Strafe diese 1 Milliarde als Kontribution auferlegt wird?
113
Цитирую последний абзац, страница 22 перевода:
And then the last paragraph on Page 22 of the translation before the Court-I quote:
Je cite encore la page 22 de la traduction anglaise:
Und dann den letzten Absatz auf Seite 22 der Übersetzung, die dem Gerichtshof vorliegt. Ich zitiere:
114
Я это сформулирую так: германские евреи должны, в наказание за свои ужасные преступления, уплатить один миллиард.Это сработает. Свиньи не пойдут ещё на одно убийство. Хочу сказать, что я не хотел бы быть евреем в Германии».
I shall choose the wording this way-that German Jewry shall, as punishment for their abominable crimes, et cetera,et cetera, have to make a contribution of 1,000,000,000. That will work. The pigs won't commit another murder in a hurry. I should like to say again that I would not like to be a Jew in Germany.
Je terminerai par ces mots: la juiverie allemande, en punition de ses abominables crimes, etc. payera une amende d’un milliard. Cela fera l’affaire; ces cochons ne commettront pas d’autres crimes. J’aime à répéter que je n’aimerais pas être Juif, en Allemagne.
Ich werde den Wortlaut wählen, daß die deutschen Juden in ihrer Gesamtheit als Strafe für ihre ruchlosen Verbrechen usw. eine Kontribution von 1 Milliarde auferlegt bekommen. Das wird hinhauen, die Schweine werden einen zweiten Mord so schnell nicht machen. Im übrigen muß ich noch einmal feststellen: Ich möchte kein Jude in Deutschland sein.
115
Вслед за этим совещанием был издан декрет о том, что на германских евреев был наложен штраф в размере одного миллиарда рейхсмарок («Рейхсгезетцблатт», 1938 год, часть 1, страница 1579). Декрет подписан подсудимым Герингом и датирован 12 ноября 1938 г.
It was whimsical remarks such as these that originated decrees, for following this meeting a decree was issued placing upon the German Jews the burden of 1,000,000,000 Reichsmark fine: 1938 Reichsgesetzblatt, Part I, Page 1579, date 12 November 1938, signed by the Defendant Goering.
C’étaient des boutades de ce genre qui étaient à l’origine des décrets; en effet, à la suite de cette conférence, un décret fut promulgué, infligeant aux Juifs allemands une amende de 1 milliard de Reichsmark: 1938,Reichsgesetzblatt, première partie, page 1579; décret signé le 12 novembre par l’accusé Göring.
Solche absonderliche Bemerkungen waren es, die Verordnungen zur Folge hatten, denn nach dieser Sitzung wurde eine Verordnung erlassen, die den deutschen Juden eine Geldstrafe von 1 Milliarde auferlegte: Reichsgesetzblatt 1938, Teil I, Seite 1579, datiert vom 12. November 1938, unterzeichnet von dem Angeklagten Göring.
116
Аналогичные декреты содержатся в «Рейхсгезетцблатт», 1939 год, часть 1, страница 282, подписанные подсудимым Герингом, и в «Рейхсгезетцблатт», 1941 год, часть 1, страница 722. Декрет подписан подсудимыми Фриком и Борманом.
Similar decrees are contained in 1939 Reichsgesetzblatt, Part I, Page 282, signed by Defendant Goering, and 1941 Reichsgesetzblatt, Part I, Page 722, signed by Defendants Frick and Bormann.
On trouve de semblables décrets dans leReichsgesetzblatt de 1939, première partie, page 282, avec les signatures de l’accusé Göring, et dans celui de 1941, première partie, page 722, avec celles de Frick et Bormann.
Ähnliche Verordnungen finden sich im Reichsgesetzblatt 1939, Teil I, Seite 282, unterzeichnet von dem Angeklagten Göring; und Reichsgesetzblatt 1941, Teil I, Seite 722, unterzeichnet von den Angeklagten Frick und Bormann.
117
И, наконец, в 1943 году евреев на основании декрета, подписанного подсудимыми Борманом, Фриком и другими, лишили права обращаться в суд. При всяком разборе дел, касающихся их, единственным арбитром была полиция, решавшая вопросы о наказании и даже смерти («Рейхсгезетцблатт», 1943 год, часть 1, страница 372). подписанного подсудимыми Борманом, Фриком.
Finally, in the year 1943, the Jews were placed beyond the protection of any judicial process by a decree signed by the Defendants Bormann and Frick and others; and the police became the sole arbiters of punishment and death: 1943 Reichsgesetzblatt, Part I, Page 372, signed by Frick and Bormann.
Finalement, en 1943, les Juifs furent exclus du domaine de la protection légale par un décret que signèrent Frick, Bormann et d’autres, et la police devint le seul arbitre des châtiments et de la mort: 1943,Reichsgesetzblatt, première partie, page 372.
Schließlich wurde im Jahre 1943 den Juden auch der letzte Rechtsschutz durch einen Erlaß entzogen, den die Angeklagten Bormann und Frick und einige andere unterzeichneten, und die Polizei allein bekam die Entscheidungsbefugnis über Strafe und Tod: Reichsgesetzblatt 1943, Teil I, Seite 372, unterzeichnet von Frick und Bormann.
118
Прошу Суд принять декреты без доказательств указанные декреты «Reichsgesetzblatt».
I ask the Court to take judicial notice of the Reichsgesetzblatt decrees cited.
Je prie le Tribunal de bien vouloir accorder valeur probatoire aux décrets que je viens de citer.
Ich ersuche den Gerichtshof, von den aus dem Reichsgesetzblatt zitierten Verordnungen amtlich Kenntnis zu nehmen.
119
Рука об руку с принятием декретов и их исполнением шло ещё одно оружие, поощряемое партией и контролируемым партией государством. Они открыто содействовали официальным антиеврейским бойкотам. Я приобщаю документ 2409-ПС, опубликованный дневник Йозефа Геббельса, экземпляр номер США-262, и я обращаю внимание суда на страницу 290, в котором ниже даты 29 марта 1933 — суд обнаружит цитату вверху страницы 1 перевода 2409-ПС: «Призыв к бойкоту одобрен всем кабинетом». И снова 31 марта 1933, он писал, на странице 1, первое предложение абзаца 2: «Провели дискуссию в самом узком кругу и решили, что бойкот начнётся завтра со всей жестокостью».
Side by side with the passage of these decrees and their execution went still another weapon, wielded by the Party and the Party-controlled state. These were the openly sponsored and official anti-Jewish boycotts against Jews. I now offer Document 2409-PS, the published diary of Joseph Goebbels, Exhibit Number USA-262, and I invite the Court's attention to Page 290 where, under date of 29 March 1933-the Court will find the quotation on the top of Page 1 of the translation of 2409-PS-»The boycott appeal is approved by the entire Cabinet.» And again on the 31st of March 1933 he wrote, on Page 1, first sentence of Paragraph 2, «We are having a last discussion among a very small circle and decide that the boycott is to start tomorrow with all severity.»
En même temps que ces décrets étaient promulgués et exécutés, le Parti et l’État placé sous son contrôle brandissaient encore une autre arme; le boycottage, officiel et ouvertement encouragé, de tous les Juifs. Je dépose maintenant le document PS-2409, journal de Joseph Goebbels (USA-262), et j’attire l’attention du Tribunal sur la page 290 où il a écrit à la date du 29 mars 1933 — le Tribunal trouvera la citation au haut de la page 1 de la traduction du PS-2420: La motion de boycottage est approuvée par le Cabinet tout entier. De nouveau, le 31 mars 1933, il écrivait, page 1, première phrase du paragraphe 2: «Nous avons une dernière discussion en très petit comité et décidons que le boycottage commencera demain d’une façon très sérieuse. «
Gleichzeitig mit der Herausgabe solcher Verordnungen und ihrer Durchführung erhielten die Partei und der von der Partei kontrollierte Staat noch eine weitere Waffe, nämlich den öffentlich unterstützten und offiziellen antijüdischen Boykott. Ich lege Dokument 2409-PS, das veröffentlichte Tagebuch von Joseph Goebbels, Beweisstück US-262, vor und verweise den Gerichtshof auf Seite 290, wo Goebbels unter dem Datum vom 29. März 1933 das Folgende schrieb; der Gerichtshof findet das Zitat am Anfang der ersten Seite der Übersetzung von 2409-PS: «Der Boykottaufruf wird von der ganzen Regierung gebilligt.» Weiter schrieb er am 31. März 1933, erster Satz auf Seite 1, Absatz 2: «Wir halten in kleinem Kreise eine letzte Besprechung ab und beschließen, daß der Boykott morgen in aller Schärfe beginnen soll.»
120
Подсудимый Штрайхер и подсудимый Франк, вместе с Гиммлером, Леем и другими являлись членами центрального комитета, который в 1933 проводил бойкот против евреев. Их имена приводятся в документе 2156-ПС, «Национал-социалистическая пресса», 29 марта 1933, экземпляр номер США-63.
The Defendant Streicher and the Defendant Frank, together with Himmler, Ley, and others, were members of a central committee who conducted the 1933 boycott against the Jews. Their names are listed in Document 2156-PS, National Socialist Party Correspondence, 29 March 1933, Exhibit Number USA-263.
L’accusé Streicher et l’accusé Frank, ainsi que Himmler, Ley et quelques autres, étaient membres d’un comité central qui dirigea le boycottage des Juifs en 1933. Les noms figurent dans le document PS-2156, «Correspondance du parti national-socialiste», 29 mars 1943 (USA-263).
Die Angeklagten Streicher und Frank waren zusammen mit Himmler, Ley und einigen anderen Mitglieder eines Zentralausschusses, der den Boykott gegen die Juden 1933 durchführte. Ihre Namen finden wir in dem Dokument 2156-PS, Nationalsozialistische Partei-Korrespondenz vom 29. März 1933, Beweisstück US-263.
121
Акты насилия против евреев начались еще в 1933 году. Одетые в форму нацисты совершали налеты на синагоги, когда там шла служба, избивали лиц, которые присутствовали в синагоге, и оскверняли религиозные эмблемы и предметы. Отчет о таких событиях содержался в официальном донесении американского генерального консула в Лейпциге от 5 апреля 1933 г.
As early as 1933 violence against the Jews was undertaken. Raids were conducted, by uniformed Nazis, on services within synagogues. Attending members of the synagogues were assaulted and religious insignia and emblems were desecrated. A report of such an occurrence is contained in the official dispatch from the American Consul General in Leipzig, dated 5 April 1933.
Dès 1933, les Juifs furent victimes d’actes de violence. Des nazis en uniforme firent irruption dans les synagogues pendant les offices; les assistants furent attaqués et des insignes et emblèmes religieux furent profanés. La dépêche officielle du Consul général américain à Leipzig en date du 5 avril 1933 contient une relation de cet événement.
Schon 1933 ging man mit Gewalttätigkeiten gegen die Juden vor. Von uniformierten Nazis wurden Razzien auf Synagogen während des Gottesdienstes durchgeführt. Personen, die in den Synagogen am Gottesdienst teilnahmen, wurden angegriffen, und religiöse Wahrzeichen und Sinnbilder wurden entweiht. Ein Bericht über einen solchen Vorgang findet sich in dem amtlichen Bericht des amerikanischen Generalkonsuls in Leipzig vom 5. April 1933.
122
Я приобщаю в качестве доказательства документ ПС-2709…
I offer in evidence Document 2709-PS...
Je dépose comme preuve le document PS-2709.
Ich lege ihn als Dokument 2709-PS vor.
35123
Председатель На что вы ссылаетесь в 2156?
THE PRESIDENT What do you refer to 2156 for?
LE PRÉSIDENT Pourquoi avez-vous cité le 2156?
VORSITZENDER Warum beziehen Sie sich auf 2156?
36124
УолшСэр, только на список фамилий показывающий подсудимых Штрайхера и Франка в качестве членов комитета по бойкоту.
MAJOR WALSH Only, Sir, to show the names of the Defendants Streicher and Frank as members of the boycott committee.
COMMANDANT WALSH Seulement pour montrer que les noms des accusés Streicher et Frank figurent parmi ceux des membres du comité de boycottage.
MAJOR WALSH Nur um zu zeigen, daß Streicher und Frank Mitglieder des Boykott-Ausschusses waren.
37125
Председатель Я понял.
THE PRESIDENT I see.
LE PRÉSIDENT Je comprends.
VORSITZENDER Ich verstehe.
38126
УолшДокументу 2709 присвоен номер экземпляра США-265. Абзац 1 страницы 1, я цитирую:
MAJOR WALSH Document 2709 has been given Exhibit Number USA-265. From Paragraph 1 of Page 1, I quote:
COMMANDANT WALSH Le document PS-2709 porte le nº USA-265. Je cite le paragraphe 1 de la page 1:
MAJOR WALSH Dokument 2709-PS ist Beweisstück US-265. Ich zitiere von Absatz 1 auf Seite 1:
127
Несколько недель назад в Дрездене одетые в форму нацисты произвели налет на синагогу, прервали вечернюю службу, арестовали 25 верующих и сорвали священную эмблему с их головных покрывал, которые надевают во время молитвы».
In Dresden, several weeks ago, uniformed Nazis raided the Jewish prayer house, interrupted the evening religious service, arrested 25 worshippers, and tore the holy insignia or emblems from their head covering worn while praying.
Il y a quelques semaines, à Dresde, des nazis en uniforme ont fait irruption dans un lieu de prières juif, interrompu le service religieux du soir, arrêté 25 fidèles et déchiré les insignes et emblèmes sacrés qu’ils portent pour la prière sur leurs coiffures.
In Dresden haben vor mehreren Wochen uniformierte ›Nazis‹ das jüdische Bethaus überfallen, den Abend-Gottesdienst unterbrochen, 25 Andächtige verhaftet und heilige Abzeichen oder Sinnbilder von ihren Kopfbedeckungen gerissen, die während des Gebets getragen werden.
128
На конференции представителей германской печати в Нюрнберге подсудимый Штрейхер и Либель, мэр города Нюрнберга, заранее сообщили собравшимся представителям печати, что нюрнбергская синагога должна быть разрушена.
At a meeting here in Nuremberg, before the representatives of the German press, the Defendant Streicher and Mayor Liebel of Nuremberg revealed in advance to the gathered members of the press that the Nuremberg synagogue was to be destroyed.
Au cours d’une réunion tenue ici, à Nuremberg, l’accusé Streicher et le bourgmestre de la ville, Liebel, révélèrent aux membres de la presse allemande assemblés que la synagogue de Nuremberg allait être détruite.
Bei einer Zusammenkunft hier in Nürnberg vor den deutschen Pressevertretern eröffnete der Angeklagte Streicher und Oberbürgermeister Liebel von Nürnberg den versammelten Pressevertretern im voraus, daß die Nürnberger Synagoge zerstört werden würde.
129
Протокол этой конференции от 4 августа 1938 г. представляется за номером ПС-1724, США-266. Цитирую со страницы 1, абзац четвертый оригинала:
I offer in evidence Document 1724-PS, Exhibit Number USA-266, which is minutes of this meeting, dated 4 August 1938. From Page 1, Paragraph 4 of the original, I quote the translation before the Court:
Je dépose comme preuve le document PS-1724 (USA-266); c’est un compte rendu de la réunion, daté du 4 août 1938. Je cite la traduction de la page 1, paragraphe 4, de l’original:
Zum Beweis unterbreite ich Dokument 1724-PS, US-266, das ein Protokoll der Sitzung vom 4. August 1938 darstellt. Von Seite 1, Absatz 4, des Originals verlese ich aus der Übersetzung, die dem Gerichtshof vorliegt, das Folgende:
130
Разрушение синагоги (информация пока еще является секретной). 10 августа 1938 года в 10 часов утра начнется разрушение синагоги. Гаулейтер Юлиус Штрейхер лично приведет в действие кран, с помощью которого будут сорваны еврейские символы — звезда Давида и т. д. Это должна быть крупномасштабная операция. Подробности пока неизвестны».
The breaking up of the synagogue (information must still be secret). On August 10, 1938, at 10 o'clock a.m., the breakup of the synagogue will commence. Gauleiter Julius Streicher will personally set the crane into motion with which the Jewish symbols, Star of David, et cetera, will be torn down. This should be arranged in a big way. Closer details are still unknown.
Démolition de la synagogue. (Information à ne pas encore divulguer.) La démolition de la synagogue commencera le 10 août 1938 à 10 heures du matin. Le Gauleiter Julius Streicher mettra lui-même en marche la grue qui servira à renverser les symboles juifs: étoile de David, etc. Il faudrait faire les choses en grand. Nous n’avons pour l’instant aucun autre détail.
Abbruch der Synagoge (Mitteilung muß noch geheim gehalten werden). Am 10. August 1938, vormittags 10.00 Uhr, wird mit dem Abbruch der Synagoge begonnen werden. Gauleiter Julius Streicher wird persönlich den Kran in Bewegung setzen, mit dem die jüdischen Symbole (Davidstern usw.) heruntergeholt werden. Die Sache soll groß aufgezogen werden. Näheres noch unbekannt.
131
Подсудимый Штрайхер лично руководил операцией.
The Defendant Streicher himself supervised the demolition.
L’accusé Streicher surveilla lui-même la démolition.
Der Angeklagte Streicher selbst beaufsichtigte die Zerstörung.
132
Документ ПС-2711, США-267. 11 августа 1938 г. в газетах сообщалось:
In support of this, I offer Document 2711-PS, a newspaper account of 11 August 1938, Exhibit Number USA-267, Paragraph 1 of the translation before the Court:
Pour compléter cette preuve, je dépose le document PS-2711; c’est un compte rendu de journal du 11 août 1938 (USA-267). Voici le paragraphe 1 de la traduction:
Zum Beweis lege ich Dokument 2711-PS vor, eine Zeitungsnachricht vom 11. August 1938, Beweisstück US-267. Absatz 1 der dem Gerichtshof vorliegenden Übersetzung lautet:
133
В Нюрнберге производится разрушение синагоги. Сам -Юлиус Штрейхер лично перед началом работы произнес речь, которая длилась более чем полтора часа. Затем по его приказу в виде как бы прелюдии к разрушению была сорвана огромная звезда Давида с купола синагоги».
In Nuremberg the synagogue is being demolished; Julius Streicher himself inaugurates the work by a speech lasting more than an hour and a half. By his order then-so to speak as a prelude of the demolition-the tremendous Star of David came off the cupola.
On détruit la synagogue de Nuremberg. Julius Streicher inaugure lui-même les travaux par un discours de plus d’une heure et demie. Puis, sur son ordre — prélude, pour ainsi dire de la démolition — l’énorme étoile de David fut arrachée de la coupole.
In Nürnberg wird die Synagoge abgebrochen. Julius Streicher leitete selbst durch eine mehr als eineinhalbstündige Rede den Beginn der Arbeiten ein. Auf seinen Befehl löste sich dann, gewissermaßen als Auftakt des Abbruchs, der riesige Davidstern von der Kuppel.
134
Эти акты насилия не были местными антисемитскими демонстрациями. Ими руководил и приказы об их проведении отдавал центральный штаб в Берлине. Это доказывается серией телеграфных сообщений, посланных берлинским штабом государственной тайной полиции начальникам полиции по всей Германии 10 ноября 1938 г. В этих телеграммах содержались инструкции относительно подготовляющихся демонстраций.
These accounts of violence were not localized anti-Semitic demonstrations but were directed and ordered from a centralized headquarters in Berlin. This is established by a series of teletype messages sent by the Berlin Secret State Police headquarters to chiefs of police throughout Germany on 10 November 1938, which contained instructions pertaining to the pre-arranged demonstration.
Ces violences n’étaient pas des manifestations locales d’antisémitisme; elles étaient dirigées et ordonnées par un état-major centralisé à Berlin, comme en font foi une série de télétypes, messages émanant du Quartier Général de la Police secrète d’État à Berlin et adressés aux chefs de la Police dans toute l’Allemagne, le 10 novembre 1938; ils contenaient des instructions relatives aux manifestations organisées.
Diese Gewaltakte waren nicht örtliche antisemitische Demonstrationen, sondern wurden von einer Zentrale in Berlin geleitet und angeordnet. Dies geht aus einer Reihe von Fernschreiben hervor, die aus der Berliner Gestapo-Zentrale an die Polizeichefs in ganz Deutschland am 10. November 1938 gesandt worden waren. Sie enthielten Vorschriften über die im voraus vorbereiteten Demonstrationen.
135
Документ ПС-3051, США-240, ранее уже представлялся в качестве доказательства. Процитирую относящуюся к делу часть одного из секретных приказов, подписанных Гейдрихом:
I now refer to Document 3051-PS, previously offered in evidence as Exhibit Number USA-240. I shall quote the relevant part of one of these confidential orders signed by Heydrich, the translation before the Court, the last half on Page 2:
Je me réfère maintenant au document PS-3051, déjà présenté comme preuve sous le nº USA-240. Je citerai, dans la traduction, les passages principaux de ces ordres confidentiels signés par Heydrich; c’est à la deuxième moitié de la page 2.
Ich beziehe mich nun auf Dokument 3051-PS, US-240. Ich werde den erheblichen Teil eines dieser von Heydrich unterschriebenen vertraulichen Befehle verlesen. Die Übersetzung liegt dem Gerichtshof vor, zweite Hälfte der zweiten Seite:
136
В связи с покушением на жизнь секретаря миссии фон Рата в Париже в ночь с 9 на 10 ноября 1938 г. по всей империи ожидаются демонстрации против евреев. В отношении этих событий следует руководствоваться нижеследующим:
Because of the attempt on the life of the Secretary of the Legation, Vom Rath, in Paris tonight, 9–10 November 1938, demonstrations against Jews are to be expected throughout the Reich. The following instructions are given on how to treat these events:
En raison de l’attentat dont a été victime à Paris, en cette nuit du 9 au 10 novembre 1938, le secrétaire de légation vom Rath, on s’attend à ce que des manifestations contre les Juifs aient lieu dans tout le Reich. Les instructions suivantes indiquent la conduite à tenir en face de ces événements:
Auf Grund des Attentats gegen den Leg. Sekretär vom Rath in Paris sind im Laufe der heutigen Nacht – 9. auf 10. 11. 1938 – im ganzen Reich Demonstrationen gegen die Juden zu erwarten. Für die Behandlung dieser Vorgänge ergehen die folgenden Anordnungen:
137
1) Руководителям полицейских органов или их заместителям по получении этой телеграммы надлежит немедленно установить телефонную связь с политическими (районными, областными или окружными) руководителями и договориться с ними о проведении совместного совещания с участием инспектора или начальника полиции порядка для обсуждения организации демонстраций. На этих совещаниях политические руководители должны быть информированы о том, что германская полиция получила от рейхсфюрера СС и начальника германской полиции следующие указания, в соответствии с которыми должны организовывать свои мероприятия и политические руководители:
1) The Chiefs of the State Police or their deputies must. get in telephonic contact with the political leaders who have jurisdiction over their Districts and must arrange a joint meeting with the appropriate inspector or commander of the Order Police to discuss the organization of the demonstrations At these discussions the political leaders have to be informed that the German Police has received from the Reichsfuehrer SS and Chief of the German Police the following instructions, in accordance with which the political leaders should adjust their own measures.
1. Les chefs de la Police d’État, ou leurs adjoints, doivent rester en contact par téléphone avec les chefs politiques de leurs districts et organiser une rencontre avec l’inspecteur ou le chef de la Police d’ordre compétent, pour discuter de la façon dont seront réglées ces manifestations. Au cours de ces discussions, les chefs politiques seront informés du fait que la Police allemande a reçu du Reichsführer SS et chef de la Police allemande les instructions suivantes, auxquelles les chefs politiques devront se conformer:
1. Die Leiter der Staatspolizeistellen oder ihre Stellvertreter haben sofort nach Eingang dieses Fernschreibens mit den für ihren Bezirk zuständigen politischen Leitungen – Gauleitung oder Kreisleitung – fernmündlich Verbindung aufzunehmen und eine Besprechung über die Durchführung der Demonstrationen zu vereinbaren, zu der der zuständige Inspekteur oder Kommandeur der Ordnungspolizei zuzuziehen ist. In dieser Besprechung ist der politischen Leitung mitzuteilen, daß die Deutsche Polizei vom Reichsführer SS und Chef der Deutschen Polizei die folgenden Weisungen erhalten hat, denen die Maßnahmen, der politischen Leitungen zweckmäßig anzupassen wären:
138
а) должны проводиться только такие мероприятия, которые не будут угрожать жизни или германской собственности (например, синагоги следует сжигать только в том случае, если нет опасности распространения пожара на прилегающие здания);
a) Only such measures should be taken which do not involve danger to German life or property. (For instance synagogues are to be burned down only when there is no danger of fire to the surroundings.)
a) Seules seront prises les mesures qui ne mettront en danger ni vies ni biens allemands. (Par exemple, le feu ne sera mis aux synagogues que s’il n’y a aucun danger d’incendie pour le voisinage.)
a) Es dürfen nur solche Maßnahmen getroffen werden, die keine Gefährdung deutschen Lebens oder Eigentums mit sich bringen (z. B. Synagogenbrände nur, wenn keine Brandgefahr für die Umgebung vorhanden ist),
139
б) магазины, учреждения и частные квартиры евреев надо разрушать, но не следует их грабить. Полиции поручается наблюдать за выполнением этого указания и задерживать грабителей».
b) Business and private apartments of Jews may be destroyed but not looted. The police is instructed to supervise the execution of this order and to arrest looters.
b) Les entreprises des Juifs, leurs logements, pourront être détruits mais non pillés. Ordre est donné à la Police de veiller à ce que ces dispositions soient observées, et d’arrêter les pillards.
b) Geschäfte und Wohnungen von Juden dürfen nur zerstört, nicht geplündert werden. Die Polizei ist angewiesen, die Durchführung dieser Anordnungen zu überwachen und Plünderer festzunehmen.
140
До этого момента мы сталкивались с постепенно усиливавшейся и расширявшейся кампанией против евреев, что было одним из основных принципов нацистской партии и государства. А с этого времени пламя предрассудков было так раздуто, что оно возбуждало весь германский народ. Германское государство было уже вооружено и готово для захватов, а мировое общественное мнение теперь без опасений можно было игнорировать. К этому времени из Германии уже было изгнано 200 тысяч из общего количества 500 тысяч евреев. Поэтому контролируемое нацистами германское государство осмелело, и Гитлер, в ожидании агрессивной войны, которая давно планировалась, уже решил вопрос о том, кого сделать козлом отпущения за готовящуюся мировую катастрофу. В документе ПС-2663, который я представляю в качестве доказательства под номером США-268, содержится речь Гитлера в рейхстаге 30 января 1939 г.:
To this point we have found a gradual and a mounting emphasis in the campaign against the Jews, one of the basic tenets of the Nazi Party and of the state. The flame of prejudice has now been lighted and fanned. The German people have been to a large degree indoctrinated, and the seeds of hatred have been sown. The German State is now armed and is prepared for conquest and the force of world opinion can now safely be ignored. Already they have forced out of Germany 200,000 of its original 500,000 Jews. The Nazi-controlled German State is therefore emboldened; and Hitler, in anticipation of the aggressive wars already planned, casts about for a «whipping boy» upon whose shoulders can be placed the blame for the world catastrophe yet to come. The speech before the Reichstag on 30 January 193g is set forth in Document Number 2663-PS, which I now offer in evidence as Exhibit Number USA-268. I quote:
Nous avons vu que la campagne contre les Juifs, l’une des doctrines fondamentales du Parti et de l’État nazi fut une progression ascendante et graduelle. La flamme du mal avait été allumée et attisée. Les Allemands ont été endoctrinés sur une vaste échelle et l’on a semé les graines de la haine. L’État allemand est armé maintenant et prêt à la conquête; il peut en toute tranquillité ignorer l’opinion mondiale. Déjà 200.000 Juifs ont été chassés d’Allemagne, sur les 500.000 qui s’y trouvaient naguère. L’État allemand s’est donc enhardi sous le contrôle nazi, et Hitler, anticipant sur les guerres d’agression déjà projetées, songe au «bouc émissaire» auquel on pourrait imputer la faute de la catastrophe à venir. Son discours prononcé devant le Reichstag le 10 janvier 1939, figure dans le document PS-2663 que je dépose maintenant comme preuve sous le nº USA-268. J’en cite un bref extrait:
Bis zu diesem Zeitpunkt haben wir eine immer stärker werdende Hetze gegen die Juden beobachtet, einen der fundamentalen Grundsätze der Nazi-Partei und des Staates. Die Flamme des Vorurteils wurde nun entzündet und angefacht. Dem deutschen Volk ist die Doktrin weitgehend eingeimpft und die Saat des Hasses gesät worden. Der Deutsche Staat ist jetzt bewaffnet und bereit, Eroberungen zu machen, und die Macht der Weltmeinung kann nun ruhig unbeachtet bleiben. Bereits 200000 von den ursprünglich vorhandenen 500000 Juden waren aus Deutschland herausgetrieben. Der nazi-kontrollierte Deutsche Staat wurde daher ermutigt, und Hitler dachte unter Vorwegnahme der bereits geplanten Angriffskriege daran, sie als Sündenböcke zu benützen, auf deren Schultern der Vorwurf für die kommende Weltkatastrophe abgewälzt werden konnte. Die Reichstagsrede vom 30. Januar 1939 ist in der Urkunde 2663-PS enthalten, die ich nunmehr als Beweisstück US-268 vorlege. Ich zitiere:
141
В случае, если международные еврейские финансисты внутри и вне Европы сумеют еще раз ввергнуть нации в мировую войну, то результатом такой войны будет не большевизация мира и победа еврейства, а уничтожение еврейской расы в Европе
If the international Jewish financiers within and without Europe succeed in plunging the nations once more into a world war, the result will not be the Bolshevization of the world and the victory of Jewry, but the obliteration of the Jewish race in Europe.
Si les Juifs de la finance internationale, tant à l’intérieur qu’à l’extérieur de l’Europe, parviennent à plonger une fois de plus les nations dans une guerre mondiale, le résultat ne sera pas la bolchévisation du Monde et la victoire d’Israël, ce sera la suppression de la race juive en Europe.
Wenn es dem internationalen Finanzjudentum in — und außerhalb Europas gelingen sollte, die Völker noch einmal in einen Weltkrieg zu stürzen, dann wird das Ergebnis nicht die Bolschewisierung der Erde und damit der Sieg des Judentums sein, son dern die Vernichtung der jüdischen Rasse in Europa.»
39142
ПредседательМы прервёмся на 10 минут.
THE PRESIDENT We will adjourn for 10 minutes.
LE PRÉSIDENT Nous suspendons l’audience dix minutes.
VORSITZENDER Wir werden uns nun auf 10 Minuten vertagen.
Объявлен перерыв
A recess was taken.
L’audience est suspendue.
Pause von 10 Minuten.
40143
ПредседательМайор Уолш, думаю трибуналу, будет удобнее если вы будете точно называть номер ПС, который есть у нас медленнее и яснее. Поймите, у нас нет номер экземпляра Соединённых Штатов, и я не знаю будет ли лучше сначала называть номер экземпляра Соединённых Штатов, а затем приводить номер ПС; я не уверен, но всё же, если вы будете говорить медленнее и яснее, чтобы мы понимали номер ПС, это будет полезным.
THE PRESIDENT Major Walsh, it would, I think, assist the Tribunal if you were careful to state the PS number which we have rather more clearly and slowly. You see, the United States Exhibit number we do not have and I do not know whether it would be better to state the United States Exhibit number first and then give us the PS number; I am not sure it would. Anyhow, if you would go a little more slowly and make certain we get the PS number, it would be helpful.
LE PRÉSIDENT Major Walsh, pourriez-vous lire plus lentement et plus clairement les numéros PS car nous n’avons pas les numéros USA. Cela serait utile pour le Tribunal.
VORSITZENDER Major Walsh, ich glaube, daß Sie dem Gerichtshof behilflich sein könnten, wenn sie die PS-Nummern der vorliegenden Dokumente etwas langsamer und deutlicher aussprechen würden. Wir kennen die US-Beweisstück-Nummern nicht, und Ich weiß nicht, ob es nicht besser wäre, die US-Beweisstück-Nummer zuerst zu nennen und erst hierauf die PS-Nummer. Ich bin mir darüber nicht ganz klar. Jedenfalls wäre es besser, wenn Sie bei Erwähnung der PS-Nummer etwas langsamer sprechen würden, um sicher zu sein, daß wir sie verstehen.
41144
УолшДа, ваша честь.
MAJOR WALSH Yes, Your Honor.
COMMANDANT WALSH Bien, Votre Honneur.
MAJOR WALSH Gewiß, Herr Präsident.
145
Главный редактор официального органа СС «Schwarze Korps», выражал схожие чувства 8 августа 1940.Я приобщаю в качестве доказательства документ 2668-ПС; это экземпляр номер США-269, страница 2 оригинала и полная выдержка находится в переводе у суда:
The Chief Editor of the official organ of the SS, the Schwarze Corps, expressed similar sentiments on August 8, 1940. I offer in evidence Document 2668-PS; this is Exhibit Number USA-269, Page 2 of the original and the full excerpt before the Court in translation, as follows:
Le rédacteur en chef de l’organe officiel des SSDas schwarze Korps exprimait, le 8 août 1940, des sentiments semblables. Je dépose comme preuve le document PS-2668 (USA-270) et je cite la traduction complète de la page 2 de l’original:
Der Hauptschriftleiter des offiziellen Organs der SS, das «Schwarze Korps», drückte am 8. August 1940 ähnliche Ansichten aus. Ich unterbreite Dokument 2668-PS als Beweismittel, Beweisstück US-269; ich lese dem Gerichtshof Seite 2 des Originals und den ganzen, dem Gerichtshof in der Übersetzung vorliegenden Auszug vor:
146
Также как и еврейский вопрос в Германии будет разрешён только тогда, когда последний еврей будет депортирован, так и остальная Европа также осознаёт, что германский мир, который грядёт должен быть миром без евреев».
Just as the Jewish question will be solved for Germany only when the last Jew has been deported, so the rest of Europe should also realize that the German peace which awaits it must be a peace without Jews.'
De même que la question juive ne sera pas résolue pour l’Allemagne tant que le dernier Juif n’aura pas été déporté, de même le reste de l’Europe réalisera que la paix allemande tant attendue doit être une paix sans Juifs.
So wie für Deutschland selbst die Judenfrage erst gelöst ist, sobald der letzte Jude ausgetrieben wurde, so mag auch das übrige Europa wissen, daß der deutsche Friede, der seiner harrt, ein Friede ohne Juden sein muß.
147
Не только сотрудники партии и государства озвучивали схожие взгляды. Подсудимый Розенберг в документе ПС-2665, США-270, том 1 и 2, апрель и сентябрь 1941, написал для журнала «Мировая борьба», следующее:
These were not the only officials of the Party and of the State to voice the same views. The Defendant Rosenberg wrote for the publication World Struggle. I offer in evidence Document 2665-PS, Exhibit Number USA-270. This publication, Volumes 1 and 2, April and September 1941, Page 71 of the original, reads,
Les personnalités officielles du Parti et de l’État n’étaient pas les seules à émettre de semblables opinions. Je cite un passage deWeltkampf, revue dans laquelle écrivait l’accusé Rosenberg. Je présente comme preuve la page 71 des cahiers 1 et 2 (avril-septembre 1941); c’est le document PS-2665 (USA-270):
Aber dies waren nicht die einzigen Funktionäre der Partei oder des Staates, die diese Ansichten vertraten. Der Angeklagte Rosenberg schrieb für die Zeitschrift «Weltkampf». Ich lege Urkunde 2665-PS, US-270, zum Beweis vor. In dieser Veröffentlichung, Nummer 1 und 2 vom April und September 1941, auf Seite 71 des Originals heißt es:
148
Еврейский вопрос будет разрешен только тогда, когда на европейском континенте не останется ни одного еврея».
' The Jewish question will be solved only when the last Jew has left the European continent.
La question juive ne sera pas résolue tant que le dernier Juif n’aura pas quitté le continent européen.
Für Europa ist die Judenfrage erst dann gelöst, wenn der letzte Jude den europäischen Kontinent verlassen hat.
149
Суд вспомнит ссылку господина судьи Джексона на документ ПС-2233(с), США-271. Ганс Франк в своем дневнике, как бы извиняясь, писал:
The Court will recall Mr. Justice Jackson's reference to the apologetic note contained in the diary of Hans Frank when he wrote, and I quote from Document 2233(c)-PS, Exhibit Number USA-271, bottom of Page 1 of the translation:
Le Tribunal se rappellera que M. Justice Jackson avait mentionné la note d’excuse contenue dans le journal de Hans Frank; je la cite, à la page 1 de la traduction du document PS-2233 (c) (USA-271):
Der Gerichtshof wird sich an die Bezugnahme von Justice Jackson auf die entschuldigende Notiz im Tagebuch des Angeklagten Hans Frank erinnern, wo er schrieb, und ich zitiere von der Urkunde 2233-C-PS, US-271, am Ende der ersten Seite der Übersetzung:
150
Я, конечно, не могу истребить всех вшей и всех евреев в течение одного только года, но с течением времени, а главным образом если вы поможете мне, эта цель будет достигнута».
Of course, I could neither eliminate all lice nor al1 Jews in only 1 year's time. But in the course of time and, above all, if you will help me, this end will be attained.
Il est certain que je n’ai pas pu me débarrasser de toute la vermine et de tous les Juifs en une seule année. Mais avec le temps, et surtout si vous voulez m’aider, j’y arriverai.
Freilich, in einem Jahre konnte ich weder sämtliche Läuse noch sämtliche Juden beseitigen. Aber im Laufe der Zeit, und vor allem dann, wenn Ihr mir helft, wird sich das schon erreichen lassen.»
42151
ПредседательМайор Уолш, забыл сказать, нам также поможет, когда вы не начинаете с начала абзаца, чтобы вы указывали на то где он находится.
THE PRESIDENT I forgot to say, Major Walsh, it would help us too, when you do not begin at the beginning of a paragraph, if you would indicate about where it is.
VORSITZENDER Ich vergaß zu erwähnen, Major Walsh, daß es, wenn Sie nicht mit dem Anfang eines Absatzes beginnen, zweckmäßig wäre, uns zu sagen, wo es ungefähr ist.
43152
УолшДа, сэр; выполню. При том, что у этой презентации нет хронологического изложения событий обращения с еврейским народом, нам кажется что следует сделать паузу, чтобы изучить материалы дела. Мы установили, что в своих высказываниях, писаниях, декретах и официальных актах нацисты совершенно ясно выразили свое намерение истребить евреев.
MAJOR WALSH Yes, Sir; I shall do that. While this presentation is not necessarily intended to be a chronological narrative of events in the treatment of the Jewish people, it would appear at this point that we should pause to examine the record to date. We find that the Nazi Party and the Nazi-dominated State have, by writings and by utterances, by decrees and by official acts, clearly expressed their intent: the Jew must be eliminated.
MAJOR WALSH Puisque cet exposé n’entend pas nécessairement retracer dans l’ordre chronologique l’évolution de la condition du peuple Juif, nous pourrions marquer un temps d’arrêt et examiner la situation à ce moment donné. Nous voyons que le parti nazi et l’État placé sous son contrôle ont exprimé clairement leurs intentions, par écrit et oralement, par décrets et par actes officiels: les Juifs doivent être éliminés.
MAJOR WALSH Herr Präsident, ich werde es tun. Da diese Darlegung nicht unbedingt als eine chronologische Erzählung der Ereignisse in der Behandlung des jüdischen Volkes anzusehen ist, erscheint es hier angebracht, zu unterbrechen, um das bisher Dargelegte zu untersuchen. Wir sehen, daß die Nazi-Partei und der von den Nazis beherrschte Staat durch Wort und Schrift, durch Erlasse und offizielle Handlungen ihre Absicht klar ausdrückten. Der Jude muß ausgestoßen werden. Wie schritten sie nun zur Ausführung dieses Zieles?
153
Как же они проводили в жизнь эту программу? Первым их требованием была поголовная регистрация всех евреев, и, поскольку политика, проводимая в отношении евреев, шла по пятам за германской агрессией, такая регистрация требовалась не только в пределах Германии, но также и на последовательно захватываемых территориях.Например, в Германии регистрацию требовал декрет («Reichsgesetzblatt», часть I, 1938, страница 922, 23 июля, подписан подсудимым Фриком); в Австрии («Reichsgesetzblatt», том 1, 1940, страница 694, 29 апреля); в Польше (Kurjer Krakowski, 5 октября 1939); во Франции (Journal Ofliciel, номер 9, страница 92, 30 сентября 1940); в Голландии (Verordnungsblatt, номер 6, 10 января 1941, подписанный подсудимым Зейсс-Инквартом).
How do they now progress to the accomplishment of this purpose? The first requirement was a complete registration of all Jews; and inasmuch as the policy relating to the Jews followed on the heels of German aggression, such registration was required not only within the Reich but successively within the conquered territories. For example, within Germany registration was required by decree Reichsgesetzblatt, Part I, 1938, Page 922, 23 July, signed by the Defendant Frick; within Austria Reichsgesetzblatt, Volume 1, 1940, Page 694, 29 April; within Poland Courier Krakowski, 5 October 1939); in France (Journal Official Number 9, Page 92, 30 September 1940); in Holland ( Verordnungsblatt, Number 6, 10 January 1941, signed by the Defendant Seyss-Inquart).
Comment procèdent-ils pour atteindre ce but? Il fallait d’abord recenser tous les Juifs et dans la mesure où le système appliqué aux Juifs fut un corollaire de l’agression allemande, le faire non seulement à l’intérieur du Reich mais aussi au fur et à mesure dans les territoires occupés. Par exemple en Allemagne, le décret de recensement fut signé le 23 juillet 1938 par l’accusé Frick (Reichsgesetzblatt 1938, première partie, page 922); en Autriche, le 29 avril 1940 (Reichsgesetzblatt 1940, volume I, page 694); en Pologne, le 5 octobre 1939 (Kurjer Krakowski); en France, le 30 septembre 1940 (Journal Officiel nº 9, page 92); en Hollande, le 10 janvier 1941, par l’accusé Seyss-Inquart (Verordnungsblatt 1941, nº 6).
Die erste Forderung war eine vollständige Registrierung aller Juden. Da die Juden-Politik den deutschen Angriffen auf dem Fuße folgte, wurde eine solche Registrierung nicht allein innerhalb des Reiches, sondern auch in den eroberten Gebieten verlangt. Beispielsweise wurde die Registrierung innerhalb Deutschlands durch Verordnung festgelegt, Reichsgesetzblatt 1938, Seite 922, vom 23. Juli, gezeichnet von dem Angeklagten Frick; in österreich, Reichsgesetzblatt 1940, Teil I, Seite 694, vom 29. April; in Polen, Kurjer Krawkowki vom 5. Oktober 1939; in Frankreich, Journal Officiel Nummer 9 vom 30. September 1940, Seite 92; in Holland, Verordnungsblatt Nummer 6 vom 10. Januar 1941, unterzeichnet vom Angeklagten Seyß-Inquart.
154
Следующий шаг заключался в том, чтобы собрать и сконцентрировать евреев на ограниченной территории, которая называлась гетто. Эта политика тщательно разрабатывалась. Об этом свидетельствуют секретные заметки, взятые из архива подсудимого Розенберга.
The second step was to segregate and concentrate the Jews within restricted areas called ghettos. This policy was carefully worked out, and perhaps the confidential statement taken from the files of the Defendant Rosenberg will best serve as an illustration.
Ensuite, il fallait isoler les Juifs dans des zones réservées, des ghettos et les y concentrer. Cette politique fut établie avec soin; peut-être la déclaration confidentielle trouvée dans les archives de l’accusé Rosenberg fournira-t-elle une illustration précieuse de cette matière.
Der zweite Schritt war, die Juden innerhalb fest begrenzter Gebiete, die Ghettos genannt wurden, abzusondern und zusammenzufassen. Diese Politik wurde sorgfältig ausgearbeitet, und vielleicht wird die folgende vertrauliche Erklärung aus den Archiven des Angeklagten Rosenberg als beste Illustration dienen.
155
Я приобщаю в качестве доказательства копию меморандума подсудимого Розенберга «Руководство еврейским вопросом» это документ ПС-212, США-272.
I offer in evidence a copy of a memorandum from Defendant Rosenberg's file entitled, «Directions for Handling of the Jewish Question,» Document 212-PS, Exhibit Number USA-272. I quote from the top of Page 2 of the translation before the Court:
Je présente comme preuve un exemplaire du mémorandum provenant des archives de l’accusé Rosenberg et intitulé «Directives sur la question juive»; c’est le document PS-212 (USA-272). Je cite un passage du haut de la page 2 de la traduction:
Ich lege zum Beweis die Kopie eines Memorandums aus den Akten des Angeklagten Rosenberg vor, mit der Überschrift: «Richtlinien für die Behandlung der Judenfrage», Dokument 212-PS, US-272. Ich zitiere vom Beginn der zweiten Seite der dem Gerichtshof vorliegenden Übersetzung:
156
Первой основной целью немецких мер, проводимых в этом вопросе, должно быть строжайшее отделение евреев от остального населения… При проведении этой меры прежде всего нужно проводить регистрацию еврейского населения путем введения приказа о принудительной регистрации и аналогичных мер…
The first main goal of the German measures must be strict segregation of Jewry from the rest of the population. The presupposition of this is, first of all, the registration of the Jewish population by the introduction of a compulsory registration order and similar appropriate measures....
Le premier des buts principaux visés par les mesures allemandes est la stricte mise à l’écart des Juifs. Les mesures préliminaires comporteront tout d’abord le recensement de la population juive ou toute autre mesure appropriée.
Ein erstes Hauptziel der deutschen Maßnahmen muß sein, das Judentum streng von der übrigen Bevölkerung abzusondern. Voraussetzung hierfür ist zunächst die restlose Erfassung der jüdischen Bevölkerung durch Einführung der Meldepflicht und sonstige geeignete Maßnahmen....
157
И затем, во втором предложении, второй абзац, на странице 2, я продолжаю:
And then, in the second sentence, in the second paragraph, on Page 2, I continue:
Je continue à la page 2, deuxième phrase du deuxième paragraphe:
Und dann heißt es weiter auf Seite 2, zweiter Absatz, zweiter Satz:
158
…все права на свободу должны быть отняты у евреев, они должны помещаться в гетто и в то же самое время они должны быть разделены согласно полу. Наличие целых еврейских общин и поселений в Белоруссии и на Украине делает это особенно простым. Более того, следует избрать места, где будет удобнее применять еврейскую рабочую силу в ходе осуществления в настоящее время трудовых программ. Эти гетто могут быть созданы под руководством еврейского самоуправления, с еврейскими чиновниками. Однако охрана этих гетто и отделение их от остальной местности должны быть поручены полиции.
...all rights of freedom for Jews are to be withdrawn-. They are to be placed in ghettos and at the same time are to be separated according to sexes. The presence of many more or less closed Jewish settlements in White Ruthenia and in the Ukraine makes this mission easier. More over, places are to be chosen which make possible the fun use of the Jewish manpower as a consequence of present labor programs. These ghettos can be placed under the supervision of a Jewish self government with Jewish officials. The guarding of the boundaries between the ghettos and the outer world is, however, the duty of the police.
Tout droit de liberté doit être retiré aux Juifs. On doit les mettre dans des ghettos et en même temps séparer les hommes des femmes. L’existence, en Ruthénie Blanche et en Ukraine, de colonies juives plus ou moins fermées facilite la tâche. Il faut, de plus, choisir des endroits qui permettent d’utiliser au maximum la main-d’œuvre juive en cas de besoin. Ces ghettos peuvent être administrés par un gouvernement autonome juif et des fonctionnaires juifs. Toutefois, c’est à la Police qu’est dévolue la garde des abords du ghetto.
... die Freizügigkeit für alle Juden aufzuheben. Eine Überführung in Ghettos unter gleichzeitiger Trennung der Geschlechter ist anzustreben. Das Vorhandensein zahlreicher mehr oder weniger geschlossener jüdischer Niederlassungen in Weißruthenien und in der Ukraine erleichtert diese Aufgabe. Im übrigen sind hierfür Orte auszuwählen, die infolge vorliegender Arbeitsvorhaben die völlige Ausnutzung der jüdischen Arbeitskraft ermöglichen. Diesen Ghettos kann unter Aufsicht eine jü dische Selbstverwaltung mit jüdischem Ordnungsdienst gegeben werden. Die Bewachung der Grenzen zwischen den Ghettos und der Außenwelt ist jedoch Sache der Polizei.
159
В тех случаях, где еще нельзя устроить гетто, следует проследить, чтобы были введены суровые меры, которые запрещали бы продолжение смешения крови между евреями и остальным населением».
Also, in the case in which a ghetto could not yet be established, care is to be taken through strict prohibition and similar suitable measures that a further intermingling of blood of the Jews and the rest of the populace does not continue.
De même, dans les cas où il serait impossible d’établir un ghetto, il faut veiller, par de strictes prohibitions et toutes mesures nécessaires, à mettre fin aux mélanges de sang entre Juifs et autres membres de la population.
Auch in den Fällen, in denen ein Ghetto noch nicht errichtet werden konnte, ist durch scharfe Verbote und sonstige geeignet erscheinende Maßnahmen dafür Sorge zu tragen, daß eine weitere blutmäßige Vermischung mit der übrigen Bevölkerung nicht mehr erfolgt.
160
В мае 1941 года Розенберг как имперский министр по делам оккупированных восточных территорий издал директиву об организации гетто на Украине.
In May 1941 Rosenberg, as the Reich Mister for the Eastern regions, issued directions confining the Jews to ghettos in the Ukraine.
En mai 1941, Rosenberg prit, en sa qualité de ministre du Reich pour les Territoires de l’Est, des dispositions tendant à confiner les Juifs d’Ukraine dans des ghettos.
Im Mai 1941 erließ Rosenberg, Reichsminister für die Ostgebiete, Anweisungen, die Juden in der Ukraine in Ghettos zu sperren.
161
Я приобщаю в качестве доказательства документ ПС-1028, США-273 и из первого предложения перевода суда, я читаю:
I offer in evidence Document 1028-PS, Inhibit Number USA-273, and from the first sentence of the translation before the Court, I read:
Je dépose comme preuve le document PS-1028 (USA-273) et je cite la première phrase de la traduction:
Ich überreiche Dokument 1028-PS, US-273, zum Beweis und lese den ersten Satz der dem Gerichtshof vorliegenden Übersetzung vor:
162
После того как евреи будут отстранены от работы во всех гражданских учреждениях, еврейский вопрос будет разрешен путем организации гетто».
After the customary removal of Jews from all public offices, the Jewish question Will have to be solved conclusively through the institution of ghettos.
Après le retrait des Juifs de toutes les fonctions publiques, c’est l’institution de ghettos qui résoudra définitivement le problème juif.
Die Judenfrage wird nach der selbstverständlichen Ausscheidung der Juden aus allen öffentlichen Stellen eine entscheidende Lösung erfahren durch Einrichtung von Ghettos....
163
Политика выраженная в цитированном меморандуме Розенберга не являлась изолированным примером и индивидуальным поступком. Этим выражалась государственная политика. Подсудимый фон Ширах сыграл свою роль в программе «геттоизации». Я приобщаю в качестве доказательства документ 3048-ПС, экземпляр номер США-274. У суда есть полный перевод того, что я цитирую. Подсудимый фон Ширах выступал перед Европейским конргрессом молодёжи в Вене 14 сентября 1942, и на странице 2, колонка 2 венского издания «Volkischer Beobachter» от 15 сентября, я цитирую:
The policies expressed in the quoted Rosenberg memoranda were not isolated instances nor the acts of one individual It was the expressed state policy. Defendant Von Schirach played his part in the program of «ghettoization.» I offer in evidence Document 3048-PS; Exhibit Number USA-274. Before the Court is a full translation of that which I wish to quote. The Defendant Von Schirach spoke before the European Youth Congress held in Vienna on 14 September 1942, and from Page 2, Column 2, of the Vienna edition of the Volkischer Beobachter of 15 September, I quote:
Les principes exprimés dans ce mémorandum Rosenberg prouvent que les mesures prises n’étaient pas le fait d’initiatives individuelles. C’était la politique expresse de l’État. L’accusé von Schirach joua son rôle dans le programme de généralisation de l’emploi des ghettos. Je dépose comme preuve le document PS-3048 (USA-274): le 14 septembre 1942, l’accusé von Schirach parla devant les membres du Congrès de la Jeunesse européenne, tenu à Vienne, et je cite la deuxième colonne de la page 2 de l’édition viennoise duVölkischer Beobachter du 15 septembre:
Die in den zitierten Memoranden Rosenbergs niedergelegte Politik beschränkt sich weder auf Einzelfälle, noch war sie die Handlung einer Einzelperson. Sie war die ausgesprochene Staatspolitik. Der Angeklagte von Schirach hatte am Programm der Errichtung der Ghettos seinen Anteil. Ich lege zum Beweis Dokument 3048-PS, US-274, vor. Dem Gerichtshof liegt eine vollständige Übersetzung des Stückes vor, aus dem ich zitieren möchte. Der Angeklagte von Schirach sprach vor dem Europäischen Jugendkongreß in Wien am 14. September 1942, und ich zitiere von Seite 2, Spalte 2 der Wiener Ausgabe des «Völkischen Beobachters» vom 15. September:
164
Всякий еврей который оказывает влияние в Европе является угрозой для европейской культуры. Если кто-то упрекает меня в том, я очищаю город, который был мегаполисом европейского еврейства, от десятков тысяч евреев направляя их в гетто Востока, я вынужден ответить: «Я понимаю это как действия во благо европейской культуры».
Every Jew who exerts influence in Europe is a danger to European culture. If anyone reproaches me with having driven from this city, which was once the European metropolis of Jewry, tens of thousands upon tens of Thousands of Jews into the ghetto of the East, I feel myself compelled to reply, 'I see in this an action contributing to European culture.»'
Tout Juif qui exerce une influence en Europe est un danger pour la culture européenne. Si quelqu’un me reproche d’avoir chassé de cette ville, qui fut jadis la grande capitale européenne de la juiverie, des dizaines et des dizaines de milliers de Juifs, vers les ghettos de l’Est, je me vois obligé de répondre: «Je considère cet acte comme une contribution à la culture européenne».
Jeder Jude, der in Europa wirkt, ist eine Gefahr für die europäische Kultur. Wenn man mir den Vorwurf machen wollte, daß ich aus dieser Stadt, die einst die europäische Metropole des Judentums gewesen ist, Zehntausende und aber Zehntausende von Juden ins östliche Ghetto abgeschoben habe, muß ich antworten: Ich sehe darin einen aktiven Beitrag zur europäischen Kultur.
165
Одно из самых больших гетто находилось в городе Варшаве. Подлинный рапорт генерала СС Штроопа озаглавлен: «Варшавское гетто больше не существует», документ ПС-1061, США-275.
One of the largest ghettos was within the City of Warsaw. The original report made by SS Major General Stroop concerning this ghetto is entitled, «The Warsaw Ghetto is no more.» I now offer this in evidence at this time, if the Court please, and request leave to refer to it later on in this presentation-Exhibit Number USA-276, 1061-PS, top of Page 3 of the translation, Document 1061-PS:
Un des plus grands ghettos était celui de Varsovie. Le rapport original fait par le général SS Stroop sur ce ghetto a pour titre: «Le ghetto de Varsovie n’est plus.» S’il plaît au Tribunal, je dépose cette pièce comme preuve et je demande la permission de m’y référer plus tard au cours de cet exposé. C’est le document USA-275 (PS-1061); la phrase que je cite maintenant se trouve en haut de la page 3 de la traduction:
Eines der größten Ghettos befand sich innerhalb der Stadt Warschau. Der Originalbericht des SS-Generalmajors Stroop über dieses Ghetto trägt den Titel: «Es gibt keinen jüdischen Wohnbezirk in Warschau mehr.» Ich lege nunmehr mit Genehmigung des Gerichtshofs Dokument 1061-PS, US-275, vor und möchte mir vorbehalten, später in meiner Darstellung darauf zurückzukommen. Der Beginn der Seite 3 der Übersetzung lautet:
166
В этом гетто, организованном в Варшаве, жили 400 тысяч евреев.
The ghetto thus established in Warsaw was inhabited by about 400,000 Jews.
Le ghetto qu’on avait créé à Varsovie était habité par 400.000 Juifs environ.
Der so gebildete jüdische Wohnbezirk in der Stadt Warschau wurde von etwa 400000 Juden bewohnt.
167
В нем было 27 тысяч квартир, примерно по две с половиной комнаты в каждой. Гетто было отделено от остального города стенами и заборами. Все открытые места, окна и двери были заколочены.
It contained 27,000 apartments with an average of two and a half rooms each. It was separated from the rest of the city by partitions and other walls and by walling-up of thoroughfares, windows, doors, open spaces, et cetera.
Il comprenait 27.000 appartements de deux pièces et demie en moyenne. Pour le séparer du reste de la ville, on avait élevé des murs et des cloisons, on avait bouché les rues, les fenêtres, les portes, les espaces libres, etc.
Es befanden sich in ihm 27000 Wohnungen mit einem Zimmerdurchschnitt von 2½ Zimmern. Er war von dem übrigen Stadtgebiet durch Brand — und Trennmauern und durch Vermauerung von Straßenzügen, Fenstern, Türen, Baulücken abgetrennt und so weiter.
168
Определенное представление об условиях жизни в этом гетто дает тот факт, что в каждой комнате жило в среднем по шесть человек. Некоторой иллюстрацией существовавших в гетто порядков служит полученный Гиммлером от бригадефюрера СС, группа «А», 15 октября 1941 г. рапорт. Это документ Л-180, США-276.
Some idea of the conditions within this ghetto can be gathered from the fact that an average of six persons lived in every room. Himmler received a report from the SS Brigadefuehrer Group A, dated 15 October 1941 which further illustrates the establishment and operation of the ghettos. I offer Document L-180 in evidence as Exhibit Number. USA-276. The translation, if the Tribunal please, is from the second paragraph from the bottom of Page 9:
On peut se faire une idée des conditions qui régnaient dans les ghettos si l’on pense que six personnes en moyenne vivaient dans chaque pièce. Un rapport du SS Brigadeführer de l’Einsatzgruppe A, daté du 15 octobre 1941 et envoyé à Himmler, donne de plus amples détails sur l’établissement et le fonctionnement des ghettos. Je le dépose comme preuve sous le nº USA-276; c’est le document L-180. Je cite le deuxième paragraphe à partir du bas de la page 9, dans la traduction:
Eine Vorstellung von den Verhältnissen innerhalb dieses Ghettos kann man durch die Tatsache erhalten, daß durchschnittlich sechs Personen in jedem Zimmer lebten. Himmler erhielt einen Bericht vom SS-Brigadeführer der Gruppe A vom 15. Oktober 1941, welcher die Errichtung und den Betrieb dieses Ghettos weiter beschreibt. Ich lege Dokument L-180, US-276, zum Beweis vor und zitiere aus der Übersetzung vom zweiten Absatz am Ende der neunten Seite:
169
Кроме организации и проведения мэр по уничтожению, о которых говорилось с первых дней наших операций, во многих больших городах стали создавать гетто. Это было особенно необходимо в Ковно (Каунас), потому что там было 30 тысяч евреев при населении в 152 400 человек».
Apart from organizing and carrying out measures of execution, the creation of ghettos was begun in the larger towns at once during the first days of operations. This was especially urgent in Kovno because there were 30,000 Jews in a total population of 152,400.
En plus des mesures d’organisation et d’exécution, la création de ghettos fut entreprise dans les grandes villes, dès les premiers jours de notre entrée en scène. Le besoin s’en faisait particulièrement sentir à Kovno, car sur un chiffre total de 152.400 habitants, il y avait 30.000 Juifs.
Neben der Organisierung und Durchführung der Exekutionsmaßnahmen wurde gleich in den ersten Tagen des Einsatzes in den größeren Städten auf die Schaffung von Ghettos hingewirkt. Besonders dringlich war dies in Kauen, da dort bei einer Gesamteinwohnerzahl von 152400 30000 Juden wohnten.
170
И последний абзац на странице 9 продолжающийся на странице 10, я цитирую:
And from the last paragraph on Page 9 continuing to page 10 I quote:
Je cite encore le dernier paragraphe de la page 9, qui se continue à la page 10:
Und vom letzten Absatz auf Seite 9 bis Seite 10 zitiere ich:
171
В Риге для гетто был назначен так называемый Московский пригород. Это самый плохой с точки зрения жилищных условий район Риги. В нем уже теперь в большинстве своем живут евреи. Перевод евреев в этот район был затруднительным, потому что латыши, живущие в этом районе, должны были быть эвакуированы, а квартирные условия в Риге очень тяжелые. На данном этапе из 28 тысяч евреев Риги 24 тысячи были переведены в гетто. При устройстве гетто полиция безопасности ограничивалась тем, что выполняла полицейские функции, в то время как устройство и управление гетто, так же как регулирование поставок продуктов питания для тех, кто жил в гетто, было предоставлено гражданской администрации, а управлению труда было поручено заниматься вопросом распределения еврейской рабочей силы. В других городах, где много евреев, гетто будут созданы аналогичным образом».
In Riga the so-called Moscow suburb' was designated as a ghetto: This is the worst dwelling district of Riga, already now mostly inhabited by Jews. The transfer of the Jews into the ghetto district proved rather difficult because the Latvian dwellings in that district had to be evacuated and residential space in Riga is very crowded. Of the 28,000 Jews living in Riga 24,000 have been transferred into the ghetto so far. In creating the ghetto the Security Police restricted themselves to mere policing duties, while the establishment and administration of the ghetto as well as the regulation of the food supply for the inmates of the ghetto was left to civil administration; the Labor Offices were left in charge of labor allocation. In the other towns with a larger Jewish population ghettos shall be established likewise.
À Riga, le faubourg dit «de Moscou» fut choisi comme ghetto. C’est le plus mauvais quartier de Riga; il était déjà habité presque uniquement par des Juifs. Le transfert des Juifs dans la zone du ghetto fut assez difficile, car les Lituaniens qui habitaient là durent être évacués, et il y a pénurie de logements à Riga. Sur les 28.000 Juifs environ restés à Riga, 24.000 sont dès à présent dans le ghetto. Au cours de l’établissement des ghettos, la Police de Sûreté s’est bornée à de simples actions de police, tandis que l’installation et l’administration, ainsi que l’organisation du ravitaillement, furent laissées aux fonctionnaires civils; le soin d’employer la main-d’œuvre juive fut laissé aux bureaux de placement.
In Riga wurde als Ghetto die sogenannte Moskauer Vorstadt bestimmt. Es handelt sich hier um das schlechteste Wohnviertel in Riga, das auch bisher schon im wesentlichen von Juden bewohnt war. Die Einweisung der Juden in den Ghettobezirk war ziemlich schwierig, weil die dort noch wohnenden Letten ausgesiedelt werden mußten und der Wohnraum Rigas sehr beengt ist. Von den in Riga verbliebenen Insgesamt rund 28000 Juden sind bisher 24000 im Ghetto untergebracht. Die Sicherheitspolizei beschränkte sich bei der Schaf fung der Ghettos auf rein polizeiliche Aufgaben, während die Einrichtung und Verwaltung der Ghettos sowie die Regelung der Verpflegung der Insassen der Zivilverwaltung und ihr Arbeitseinsatz den Arbeitsämtern überlassen wurden. Auch in den übrigen Städten, in denen noch eine größere Anzahl von Juden wohnt, werden Ghettos eingerichtet.
172
Евреев загоняли в гетто в польской провинции Галиция. У меня нет слов адекватно описать те условия, о которых идет речь в докладе Кацмана, генерал-лейтенанта полиции, Крюгеру, генералу полиции на восточных территориях. Доклад датирован 3 июня 1943 года и озаглавлен «Решение еврейского вопроса в Германии». Я представляю документ Л-18, США-277. Цитирую с конца страницы 11 перевода,со слов: «Абсолютно катастрофические условия выявлены в гетто Рава-Русской и Рогатина».
Jews were also forced into ghettos in the Polish Province of Galicia. No words in my vocabulary could describe quite so adequately the conditions as those contained in the report from Katzmann, Lieutenant General of Police, to Kruger, General of the Police East dated 3 June 1943, entitled «Solution of Jewish Question in Galicia.» I offer Document L-18 in evidence as Exhibit Number USA-277. From the translation, if the Court please, we will begin with the last three sentences on Page 11, that is, the last three sentences prior to the word «nothing» which is there on that page: «Nothing but catastrophical conditions were found in the ghettos of Rawa-Ruska and Rohatyn.»
«On établira également des ghettos dans les autres villes où habite un plus grand nombre de Juifs. Certains Juifs furent contraints aussi d’aller dans des ghettos de la province polonaise de Galicie. Dans la crainte d’être évacués, les Juifs de Rawa-Ruska avaient caché dans des souterrains ceux d’entre eux qui étaient atteints du typhus exanthématique. Lorsque l’évacuation dut commencer, la police découvrit que 3.000 Juifs environ étaient atteints de cette maladie dans ce ghetto. Pour détruire aussitôt ce foyer d’infection, on fit entrer en action tous les officiers de police vaccinés contre le typhus exanthématique. Nous parvînmes ainsi à crever cet abcès sans perdre plus d’un seul policier. Les mêmes conditions ou à peu près régnaient à Rothatyn. «
Auch in der polnischen Provinz Galizien wurden Juden in Ghettos gezwungen. Keine Worte aus meinem Wortschatz könnten die Verhältnisse annähernd so beschreiben, wie sie in dem Bericht des Generalleutnants der Polizei, Katzmann, an den General der Polizei Ost, Krüger, vom 3. Juni 1943 beschrieben sind. Der Bericht trägt den Titel «Lösung der Judenfrage in Galizien». Ich lege zum Beweis Dokument L-18, US-277, vor. Wir wollen von der Übersetzung der letzten drei Sätze auf Seite 11, beginnend mit dem Wort «Geradezu», vorlesen:
173
Евреи Равы-Русской, боясь эвакуации, спрятали тех, кто страдал сыпным тифом, в ямах. Когда должна была начаться эвакуация, полиция обнаружила, что три тысячи евреев, страдавших сыпным тифом, спрятаны в этом гетто. Для того чтобы уничтожить очаг инфекции, все полицейские офицеры, которым была сделана прививка против сыпного тифа, были призваны для этого дела. Таким образом нам удалось уничтожить этот источник заразы, потеряв при этом только одного офицера. Такие же примерно условия были обнаружены в Рогатине».
The Jews of Rawa-Ruska, fearing the evacuation, had concealed those who suffered from spotted fever in underground holes. When evacuation was to start it was found that 3,000 Jews suffering from spotted fever lay about in this ghetto. In order to destroy this center of pestilence at once, every police officer inoculated against spotted fever was called into action. Thus we succeeded in destroying this plague-boil, losing thereby only one officer. Almost the same conditions were found in Rohatyn.
Mon vocabulaire n’est pas assez riche pour donner une impression exacte des conditions de vie telles qu’elles sont exposées dans le rapport de Katzmann, Generalleutnant de la Police, en date du 3 juin 1943, adressé à Krüger, général de la Police dans l’Est et intitulé «Solution du problème juif en Galicie.» Je dépose le document L-18 sous le nº USA-277. Plaise au Tribunal. Je citerai d’abord les trois dernières phrases de la page 2 de la traduction, c’est-à-dire les trois dernières phrases précédant celle-ci: «Les conditions que nous avons trouvées dans les ghettos de Rawa-Ruska et Rothatyn étaient tout simplement catastrophiques.» C’est huit lignes environ avant le bas de la page:
Geradezu katastrophale Zustände wurden in den Judenwohnbezirken in Rawa-Ruska und in Rohatyn angetroffen. Die Juden in Rawa-Ruska hatten aus Furcht vor der Aussiedelung ihre Fleckfieberkranken verschwiegen und in Erdlöchern untergebracht. Als die Aussiedelungsaktion begonnen werden sollte, wurde festgestellt, daß 3000 fleckfiebererkrankte Juden in diesem Wohnbezirk herumlagen. Zur Vertilgung dieses Seuchenherdes mußten sofort alle auf Fleckfieber geimpften Poli zeibeamten herangezogen werden. Es gelang dann auch tatsächlich mit nur einem Mann Verlust, diese Pestbeule zu vernichten. Fast dieselben Zustände wurden in Rohatyn angetroffen.
174
Далее, страница 19:
On page 19 of the same document, L-18, the last paragraph, I wish to quote further.
Je désire encore faire une citation dans ce même document L-18, au dernier paragraphe de la page 19:
Von Seite 19 des gleichen Dokuments, L-18, letzter Absatz, möchte ich weiter zitieren.
44175
Председатель Да.
THE PRESIDENT Yes.
VORSITZENDER Ja.
45176
Уолш
MAJOR WALSH
MAJOR WALSH
MAJOR WALSH
177
Поскольку мы получаем все более и более тревожные сообщения о том что евреи вооружаются и что это продолжается все время, мначали две недели тому назад, в июне 1943 года, по всему району Галиции осуществлять мероприятия с применением самых решительных мер для того, чтобы уничтожить еврейские банды. Было обнаружено, что необходимы специальные меры для того, чтобы ликвидировать гетто во Львове, где были устроены землянки, о которых я уже говорил. Для того чтобы избежать потерь с нашей стороны, здесь с самого начала надо было действовать жестоко. Нам пришлось взорвать и сжечь несколько домов... К нашему удивлению, мы сумели схватить примерно 20 тысяч евреев вместо 12 тысяч, которых мы зарегистрировали. Мы должны были вытащить по крайней мере 3 тысячи трупов евреев из всякого род укромных мест, где они совершали самоубийства, приняв яд.
Since we received more and more alarming reports on the Jews becoming armed in an ever June 1943, an action throughout the whole of the District of Galicia with the intent to use strongest measures to destroy the Jewish gangsterdom. Special measures were found necessary during the action to dissolve the living quarters in Lvov where the dug-out mentioned above had been established. Here we had to act brutally from the beginning in order to avoid losses on our side; we had to blow up or to burn down several houses. On this occasion the surprising fact arose that we were able to catch about 20,000 Jews instead of 12,000 Jews who had registered. We had to pull at least 3,000 Jewish corpses out of every kind of hiding place; they had committed suicide by taking poison.
Ayant reçu des rapports de plus en plus alarmants aux termes desquels les Juifs s’armaient de plus en plus, nous passâmes à l’action pendant la dernière quinzaine de juin 1943, dans tout le district de Galicie, avec l’intention de prendre des mesures draconiennes pour mettre fin au gangstérisme juif. Des mesures spéciales s’avérèrent nécessaires pour dissoudre le ghetto de Lwow où avaient été transportés les malades trouvés dans les souterrains et dont nous avons parlé plus haut. Pour éviter des pertes de notre côté, nous avons dû agir brutalement dès le début; nous avons dû faire sauter ou brûler plusieurs maisons. Et nous avons eu la surprise de prendre 20.000 Juifs et non 12.000, ce dernier chiffre étant celui du recensement. Nous avons dû sortir environ 3.000 cadavres de Juifs des endroits où ils s’étaient cachés: ils s’étaient suicidés ou avaient avalé du poison.
Da immer mehr alarmierende Nachrichten eintrafen über die sich mehrende Bewaffnung der Juden, wurde in den letzten 14 Tagen des Monats Juni 1943 in allen Teilen des Distrikts Galizien gleichzeitig mit den schärfsten Mitteln gegen die Vernichtung des jüdischen Banditentums eingeschritten. Besondere Maßnahmen waren notwendig bei der Auflösung des jüdischen Wohnbezirks in Lemberg, wo die bereits demontierten Bunker eingerichtet waren. Hier mußte, um eigene Verluste zu vermeiden, von vornherein brutal eingeschritten werden, wobei mehrere Häuser gesprengt bzw. durch Feuer vernichtet werden mußten. Hierbei ergab sich die erstaunliche Tatsache, daß anstatt der gemeldeten 12000 Juden insgesamt 20000 Juden erfaßt werden konnten. Mindestens 3000 jüdische Leichen, die durch Einnehmen von Gift Selbstmord begingen, mußten bei den Aufräumungsarbeiten aus allen möglichen Verstecken geborgen werden.
178
На странице 20 документа, я читаю третий абзац:
On Page 20 of this document, the third paragraph I read:
Je lis le troisième paragraphe de la page 20 de ce document:
Auf Seite 20 dieses Dokuments heißt es im dritten Absatz:
179
Несмотря на небывалое бремя, павшее на каждого сотрудника СС и полиции в ходе этих акций, настроение и дух этих сотрудников был крайне высоким и похвальным до самого последнего дня.
Despite the extraordinary burden heaped upon every single SS and Police member during these actions, the mood and spirit of the men were extraordinarily good and praiseworthy from the first to the last day.
En dépit de la tâche extrêmement lourde dévolue à chaque officier de la Police et des SS au cours de ces actions, le moral des hommes fut extraordinairement bon et leur attitude digne d’éloges, du premier jusqu’au dernier jour.
Trotz der außerordentlichen Belastung, die jeder einzelne SS — und Polizeiangehörige während dieser Aktionen durchzumachen hatte, ist die Stimmung und der Geist der Männer vom ersten bis zum letzten Tage außerordentlich gut und lobenswert gewesen.
180
Эти действия и акты угона и убийства были не совсем бесполезными. Автор этого доклада, на девятой странице переведённой копии говорил и я цитирую последний абзац:
These acts and actions of removal and slaughter were not entirely without profit. The author of this report, on the ninth page of this translated copy stated, and I quote the last paragraph:
Ces actions, qui consistaient à déplacer et à massacrer les gens, n’étaient pas absolument dénuées de profit. L’auteur de ce rapport dit, au dernier paragraphe de la page 9 de cette traduction, et je cite:
Diese Taten und Handlungen der Beseitigung und Abschlachtung waren überdies auch noch gewinnbringend. Der Schreiber dieses Berichts führt auf Seite 9 der Übersetzung aus, und ich zitiere den letzten Absatz:
181
В период проведения эвакуации мы провели также конфискацию еврейской собственности. Очень большое количество этой собственности было конфисковано и передано специальному управлению «Рейнхард». Кроме мебели и текстильных товаров, следующие предметы были конфискованы и переданы управлению «Рейнхард».
Together with the evacuation action we executed the confiscation of Jewish property. Very high values were confiscated and handed over to the Special Staff 'Reinhard.' Apart from furniture and many textile goods, the following amounts were confiscated and turned over to Special Staff 'Reinhard.»'
En même temps que nous évacuions, nous avons confisqué les biens juifs, en grande quantité, et nous les avons remis à l’Einsatzkommando»Reinhard». Ont été confisqués et transmis à l’Einsatzkommando»Reinhard» en plus des objets textiles et d’ameublement...
Gleichzeitig mit den Aussiedelungsaktionen wurde die Erfassung der jüdischen Vermögenswerte durchgeführt. Außerordentliche Werte konnten sichergestellt und dem Sonderstab ›Reinhard‹ zur Verfügung gestellt werden. Außer den erfaßten Möbeln und großen Mengen von Textilien usw. wurden im einzelnen erfaßt und dem Sonderstab ›Reinhard‹ abgeführt:
182
Вот часть списка этих конфискованных предметов:
I would like to read a few of the many and assorted items listed under this confiscation:
(je voudrais lire quelques-uns des nombreux articles énumérés):
Ich möchte einige wenige dieser beschlagnahmten Gegenstände verlesen, so wie sie aufgeführt sind:
183
20 952 килограмма золотых обручальных колец, 7 полных коллекций печатей, полный чемодан перочинных ножей, ящик вечных ручек и автоматических карандашей, три мешка колец ненастоящего золота, 35 вагонов мехов.
20.952 kilograms of golden wedding rings; 7 stamp collections, complete; 1 suitcase with pocket knives; 1 basket of fountain pens and propelling pencils; 3 bags filled with rings -not genuine; 35 wagons of furs.
20,952 kgr. d’alliances en or, 7 collections de timbres au complet, 1 valise remplie de couteaux de poche, 1 panier rempli de stylos et stylomines, 3 sacs de bagues sans grande valeur, 35 wagons de fourrures.
20,952 kg Eheringe – Gold; 7 Briefmarkensammlungen kompl. 1 Koffer mit Taschenmessern. 1 Koffer mit Füllhaltern und Drehbleistiften. 3 Säcke mit unechten Ringen – Schmuck. 35 Waggons Pelze.
184
Я не обременяю суд подробным списком ценностей и конфискованных денежных средств; но цитированное показывает тщательность грабежа беззащитных людей, даже 11,73 килограмма золотых зубов и пломб.
I will not burden the Court with the detailed lists of objects of value and of the money confiscated; but the foregoing is cited to illustrate the thoroughness of the looting of a defenseless people, even to the 11.73 kilograms of gold teeth and inlays.
Je n’importunerai pas le Tribunal avec les listes détaillées d’objets de valeur et de sommes d’argent confisquées, mais j’ai cité celle-ci pour démontrer l’intégralité du pillage de ce peuple sans défense, pillage allant jusqu’à 11,730 kgr. de dents et de plombages en or.
Ich möchte den Gerichtshof nicht mit den detaillierten Listen von konfiszierten Wertgegenständen und Geld belasten; das Vorstehende wurde jedoch angeführt, um die Gründlichkeit der Plünderung eines wehrlosen Volkes zu schildern, die sogar «11,73 kg Zahngold – Zahnprothesen» in sich schließt.
185
К концу 1942 года евреи в польском генерал-губернаторстве были согнаны в 55 общин, в то время как до немецкого вторжения существовала примерно тысяча еврейских поселений на той же площади. Это сообщено в официальной газете генерал-губернаторства за 1 ноября 1942 года.
By the end of 1942 Jews in the Government General of Poland had been crowded into 55 localities whereas before the German invasion there had been approximately 1,000 Jewish settlements within this same area. This is reported in the 1942 official gazette for the Government General, Number 94, Page 665, 1 November 1942.
À la fin de 1942, les Juifs du Gouvernement Général de Pologne avaient été confinés dans cinquante-cinq localités, là où, avant l’invasion allemande, il y avait eu approximativement 1.000 colonies juives. Cette information est donnée par la Gazette officielle du Gouvernement Général, nº 94, page 665,er novembre 1942.
Ende 1942 waren die Juden im Generalgouvernement Polen in 55 Gemeinden zusammengepfercht, während vor dem deutschen Überfall im gleichen Gebiet ungefähr 1000 jüdische Siedlungen bestanden haben. Dies wird in der amtlichen Zeitung für das Generalgouvernement 1942 in Nummer 94, Seite 665, vom 1. November 1942 berichtet.
186
Евреи, поскольку их регистрировали и поселяли в гетто, стали фактически источником рабского труда. Полагаю, что следует подчеркнуть, какая разница существует между рабским трудом и трудовой повинностью. В последнем случае должна была существовать какая-то компенсация, установленные часы работы, какое-то медицинское обслуживание и другие элементарные меры социального обеспечения, но первая группа ничего этого не имела и фактически находилась на положении хуже рабов.
The Jews having been registered and confined within the ghettos, they now furnished a reservoir for slave labor. It is believed pertinent at this time to point out the difference between the slave labor and labor duty. The latter group were entitled to reasonable compensation, stated work hours, medical care and attention, and other social security measures, while the former were granted none of these advantages, being in fact on a level below a slave.
Les Juifs recensés et confinés dans les ghettos formaient une réserve de travailleurs esclaves. Il est important, je crois, de souligner ici quelle différence il y a entre la main-d’œuvre esclave et les travailleurs obligatoires: ces derniers étaient dédommagés de manière raisonnable, ils avaient des heures de travail fixes, ils étaient l’objet d’une surveillance médicale et bénéficiaient de mesures de sécurité, tandis que les premiers n’avaient aucun de ces avantages, étant en fait traités beaucoup plus durement que des esclaves.
Nachdem die Juden registriert und in die Ghettos gesperrt waren, bildeten sie nun ein Reservoir für Sklavenarbeit. Ich glaube, es gehört zur Sache, hier auf den Unterschied zwischen Sklavenarbeit und Arbeitspflicht hinzuweisen. Die letzte Gruppe hatte Anspruch auf mäßige Entschädigungen, auf festgesetzte Arbeitsstunden, ärztlichen Beistand und andere soziale Fürsorgemaßnahmen, während der erstgenannten Gruppe nichts von diesen Vorteilen gewährt wurde; tatsächlich war ihr Standard niedriger als der eines Sklaven.
187
Подсудимый Розенберг, в качестве рейхсминистра по делам оккупированных восточных территорий, учредил специальный отдел, который должен был разрешать еврейскую проблему путем организации рабского труда. Его планы содержались в документе, который мы представляем под номером ПС-1024, США-278.
Defendant Rosenberg, as Reich Minister for the Eastern Occupied Territories, set up within his organization a department which, among other things, was to seek a solution for the Jewish problem by means of forced labor. His plans are contained in another document, 1024-PS, which I now offer in evidence, Exhibit Number USA-278.
L’accusé Rosenberg, en sa qualité de ministre pour les Territoires occupés de l’Est, créa dans ses services un département chargé, entre autres choses, d’apporter au problème juif une solution basée sur le travail forcé. Ses plans nous sont fournis par le document PS-1024, que je dépose comme preuve sous le nº USA-278. Je cite ce document intitulé
Der Angeklagte Rosenberg errichtete als Reichsminister für die besetzten Ostgebiete innerhalb seiner Organisation eine Abteilung, die unter anderen Dingen eine Lösung des jüdischen Problems durch Zwangsarbeit suchte. Seine Pläne sind in einem anderen Dokument, 1024-PS, enthalten, das ich zum Beweis als US-278 vorlege.
188
Этот документ озаглавлен: «Общая организация и задачи управления по разрешению вопросов, связанных с восточными территориями». Он датирован 29 апреля 1941 г.
I quote the first part of Paragraph 3 of Page 1 of the document entitled, «General Organization and Tasks of Our Office for the General Handling of Problems in the Eastern Territory.» This is dated 29 April 1941. This brief excerpt reads as follows:
«Organisation et tâches d’un Service central pour l’examen des problèmes relatifs aux territoires européens de l’Est», daté du 29 avril 1941. Le bref extrait que je lis se trouve à la première partie du troisième paragraphe de la page 1:
Ich zitiere den ersten Teil des dritten Absatzes auf Seite 1 des Dokuments, das die Überschrift: «Allgemeiner Aufbau und Aufgaben einer Dienststelle für die zentrale Bearbeitung der Fragen des osteuropäischen Raumes» trägt. Es ist vom 29. April 1941 datiert. Ein kurzer Auszug lautet wie folgt:
189
Разрешение еврейской проблемы должно сейчас проводиться мерами временного характера.Принудительный труд для евреев, создание гетто и т. д. должны служить разрешению этой проблемы».
A general treatment is required for the Jewish problem for which a temporary solution will have to be determined (forced labor for the Jews, creation of ghettos, et cetera).
Il faut prendre des mesures générales afin de donner au problème juif une solution temporaire (travail forcé pour les Juifs, création de ghettos, etc.).
Eine allgemeine Behandlung erfordert die Judenfrage, deren zeitweilige Übergangslösung festgelegt werden muß (Arbeitszwang der Juden, eine Ghettosierung usw.).
190
После этого Розенберг издал инструкцию, документ ПС-212, США-272, о том, что еврейский труд должен применяться и использоваться главным образом в форме ручного труда.
Thereafter he issued instructions that Jewish forced labor should be effected and utilized for every manual labor; and I refer to Document 212-PS, already in evidence, Exhibit Number USA-272. From Page 3 of this document, Paragraph 5 and Paragraph 7, I quote Paragraph 5:
Là-dessus, il donna des instructions pour que les Juifs soient affectés et utilisés aux travaux manuels, et je me réfère au document PS-212, déjà déposé sous le nº USA-272; je cite, à la page 3 de ce document, les paragraphes 5 et 7:
Sodann gab er Anweisungen heraus, daß jüdische Zwangsarbeitskräfte für jede Handarbeit herangezogen und verwandt werden sollten. Ich verweise auf Dokument 212-PS, US-272. Ich lese von Seite 3, Absatz 5 und 7; ich zitiere Absatz 5:
191
...Еврейский труд, независимо от возраста, должен полностью и неограниченно использоваться для перестройки оккупированных восточных территорий. Таково действующее правило».
The standing rule for the Jewish labor employment is the complete and unyielding use of Jewish manpower regardless of age in the reconstruction of the Eastern Occupied Territories.
La règle de base pour l’emploi de la main-d’œuvre juive est celle-ci: tirer parti absolument et sans merci de cette main-d’œuvre, sans considération d’âge, pour les travaux de reconstruction dans les territoires occupés de l’Est.
Maßgebliches Gebot für den jüdischen Arbeitseinsatz wird allein die volle und unnachsichtliche Inanspruchnahme der jüdischen Arbeitskraft ohne irgendeine Altersbegrenzung zum Wiederaufbau der besetzten Ostgebiete sein.
192
И с абзаца 7 этой же страницы:
And from Paragraph 7 of the same page I read:
Und von Absatz 7 derselben Seite:
193
Нарушение указаний германских властей, в особенности нарушение распоряжений по использованию принудительного труда со стороны евреев, должно караться смертью.
Violations of German measures, especially evasions of the forced labor regulations, are to be punished by death in the case of the Jews.
Toute violation de mesures prises par les Allemands, en particulier des règlements du Travail obligatoire, doit être sanctionnée, lorsque le délinquant est Juif, par la peine de mort.
Verstöße gegen deutsche Maßnahmen, insbesondere die Entziehung vom Arbeitszwang, sind bei Juden grundsätzlich mit der Todesstrafe zu ahnden.
194
Из гетто еврейские рабочие отбирались и отправлялись на специальные пункты, где они концентрировались. Здесь отделялись евреи, которых можно было использовать для работы, причем их отделяли от тех, которые Казались ненужными. Например, из 45 тысяч евреев отбирались 10—15 тысяч таких, которые должны были работать.Моё подтверждение данному заявлению находится в телеграмме РСХА Гиммлеру, помеченной «срочно» и «секретно» от 16 декабря 1942.
From the ghettos Jewish labor was selected and sent to a concentration area. Here the usable Jews were screened from those considered worthless. For example, a contingent of 45,000 Jews would be expected to yield 10,000 to 15,000 usable laborers. My authority for this statement is contained in a RSHA telegram to Himmler, marked «urgent» and «secret,» dated 16 December 1942.
La main-d’œuvre juive était sélectionnée dans les ghettos, puis était envoyée dans un camp de concentration. Là, «les Juifs utilisables» étaient séparés de ceux que l’on considérait comme inutiles. Par exemple, un contingent de 45.000 Juifs donnait environ de 10.000 à 15.000 travailleurs utilisables. Je fonde cette affirmation sur le texte d’un télégramme envoyé à Himmler par le RSHA, le 16 décembre 1942, et qui porte les mentions» Urgent» et» Secret».
Aus den Ghettos wurden jüdische Arbeiter ausgesucht und in die Sammellager gebracht. Hier wurden die verwendbaren Juden von den als wertlos angesehenen Juden abgesondert. So wurde z. B. erwartet, daß ein Kontingent von 45000 Juden 10 bis 15000 verwendbare Arbeiter liefern würde. Die Quelle für meine Behauptung ist ein RSHA-Telegramm an Himmler, das mit «eilig» und «geheim» bezeichnet ist und das Datum des 16. Dezember 1942 trägt.
195
I offer this document, 1472-PS, in evidence, Exhibit Number USA-279; and from the translation before the Court I read the last four lines: .
Je dépose comme preuve ce document PS-1472 sous le numéro USA-279 et j’en cite les quatre dernières lignes, dans la traduction:
Ich lege dieses Dokument, 1472-PS, als Beweisstück US-279 vor. Von der dem Gerichtshof zur Verfügung stehenden Übersetzung lese ich die letzten vier Zeilen vor:
196
В общее количество 45 000 включаются физически немощные и остальные (старые евреи и дети). Проводя отбор для данной задачи, по крайней мере 10 000—15 000 рабочих будут доступны, когда в Аушвиц поступят евреи».
In the total of 45,000 are included physically handicapped and others (old Jews and children). In making a distribution for this purpose, at least 10,000 to 15,000 laborers will be available when the Jews arriving at Auschwitz are assigned.
Dans le total de 45.000 sont inclus les Juifs diminués physiquement (vieillards et enfants). En tenant compte de ces circonstances, 10.000 à 15.000 travailleurs au moins seront disponibles lorsque les Juifs arriveront à Auschwitz.
In der Zahl von 45000 ist der arbeitsunfähige Anhang (alte Juden und Kinder) mit inbegriffen. Bei Anlegung eines zweckmäßigen Maßstabes fallen bei der Ausmusterung der ankommenden Juden in Auschwitz mindestens 10000 bis 15000 Arbeitskräfte an.
197
Рапорт генерал-лейтенанта полиции Кацмана высшему руководителю СС и полиции на Востоке Крюгеру, документ Л-18, США-277, ясно показывает характер этого насильственного принуждения к труду и дает представление об условиях труда евреев в это время. Перевод на странице 2, начиная с абзаца 6:
From Document L-18, a report from the Lieutenant General of the Police, Katzmann, to General of the Police east, Kruger, already in evidence, Exhibit Number USA-277, we find the clearly outlined nature of the forced labor situation for the Jews. On Page 2 of the translation, starting with Paragraph 6, I read:
Dans le document L-18, un rapport du Generalleutnant de la Police, Katzmann, adressé au général de la Police dans l’Est, Krüger, déjà déposé comme preuve sous le nº USA-277, nous trouvons des indications très nettes sur la nature du travail forcé auquel étaient astreints les Juifs. Je commence à lire le paragraphe 6 de la page 2 de la traduction:
Aus Dokument L-18, dem Bericht des Generalleutnants der Polizei, Katzmann, an den General der Polizei Ost, Krüger, der als Beweisstück US-277 bereits vorliegt, finden wir die Art der Zwangsarbeit für die Juden deutlich geschildert. Ich lese von Seite 2 der Übersetzung, beginnend mit dem sechsten Absatz:
198
Лучшим выходом из положения будет организация трудовых лагерей с помощью СС и полицейских руководителей. Наиболее подходящий вид труда может заключаться в том, чтобы достроить дорогу «Dg-4», которая так необходима и важна для всей южной части фронта и которая сейчас находится в катастрофическом состоянии. 15 октября 1941 г. организация лагерей вдоль этой дороги была уже начата, и, несмотря на значительные трудности, которые существовали, через две недели уже было организовано семь лагерей, в которых содержалось 4 тысячи евреев».
The best remedy consisted in the formation of forced labor camps by the SS and Police Leader. The best opportunity for labor was offered by the necessity to complete the 'Dg. 4' road which was extremely important and necessary for the whole of the southern part of the front and which was in a catastrophically bad condition. On October 15, 1941, the establishment of camps along the road was commenced; and despite considerable difficulties there existed, after a few weeks only, seven camps containing 4,000 Jews.
Le meilleur remède consistait à faire établir par les chefs de la SS et de la Police, des camps de travail forcé. Une excellente occasion était fournie par la nécessité de terminer la route DG 4; c’était une route qui se trouvait en très mauvais état, extrêmement importante et indispensable à toute la partie sud du front. Le 15 octobre 1941, les premiers camps furent établis le long de la route, et, en dépit des difficultés considérables qui surgirent, après quelques semaines, il y avait 7 camps contenant 4.000 Juifs.
Die beste Handhabe hierzu bot die Bildung von Zwangsarbeitslagern durch den SS — und Polizeiführer. Arbeitsmöglichkeiten boten sich vor allen Dingen an dem äußerst wichtigen, für den gesamten Südabschnitt der Front notwendigen Ausbau der Dg. 4, die sich in einem katastrophalen Zustand befand. Am 15. Oktober 1941 wurde mit dem Ausbau der Lager an der Rollbahn begonnen, und schon nach wenigen Wochen entstanden trotz erheblicher Schwierigkeiten 7 Lager, die mit 4000 Juden belegt wurden.
199
Со страницы 2, абзац 7, я читаю:
Prom Page 2, Paragraph 7, I read:
Je lis maintenant le paragraphe 7 de la page 2:
Von Seite 2, Absatz 7, möchte ich wie folgt zitieren:
200
Вскоре за этими лагерями было организовано еще несколько лагерей. Таким образом, через некоторое время мы имели уже 15 лагерей и сообщили об этом руководящим работникам СС и полиции. За это время примерно 20 тысяч евреев прошли через эти лагеря, и, несмотря на невероятные трудности, я сегодня могу доложить, что 160 километров дороги уже построено».
Soon more camps followed these first ones, so that after a very short time the completion of 15 camps of this kind could be reported to the superior leader of SS and police. In the course of time about 20,000 Jewish laborers passed through these camps. Despite the hardly imaginable difficulties arising from this problem I can report today that about 160 kilometers of the road are completed.
Bientôt, d’autres camps suivirent les premiers, de telle sorte qu’après un délai très court, on put annoncer au chef suprême de la SS et de la Police que quinze de ces camps étaient établis. 20.000 travailleurs juifs environ y passèrent. En dépit de difficultés presque inimaginables, je puis annoncer aujourd’hui qu’environ 160 kilomètres de route ont été faits.
Diesen ersten Lagern folgten bald weitere, so daß in kürzester Frist 15 derartige Lager dem Höheren SS — und Polizeiführer gemeldet werden konnten. Durch diese Lager sind im Laufe der Zeit rd. 20000 jüdische Arbeitskräfte durchgelaufen. Trotz aller erdenklichen Schwierigkeiten, die bei diesem Problem auftauchten, können heute rd. 160 km Straße als fertiggestellt gemeldet werden.
201
И со страницы 2, абзац 8, я читаю:
And from Page 2, Paragraph 8, I read:
Je lis le paragraphe 8 de la page 2:
Und auf Seite 2, Absatz 8, heißt es:
202
В то же время все евреи пригодные к работе должны регистрироваться и направляться на работу трудовыми ведомствами».
At the same time all other Jews fit for work were registered and distributed for useful work by the labor agencies.
En même temps, tous les autres Juifs aptes au travail furent recensés et répartis par les bureaux de placement à des fins de travail utile.
Zu gleicher Zeit wurden alle anderen arbeitsfähigen Juden von den Arbeitsämtern registriert und einer nutzbringenden Arbeit zugeführt.
203
И на странице 5, последняя часть абзаца 1…
And on Page 5, last part of Paragraph 1...
La dernière partie du paragraphe 1 de la page 5…
Auf Seite 5 in dem letzten Teil des ersten Absatzes...
46204
ПредседательВам не нужна остальная часть этого абзаца на странице 2?
THE PRESIDENT Don't you want the remainder of that paragraph on Page 2?
LE PRÉSIDENT N’allez-vous pas lire le reste de ce paragraphe, page 2?
VORSITZENDER Wollen Sie nicht den restlichen Teil des Absatzes auf Seite 2 vorlesen?
47205
УолшЭто длинный документ, я воздержусь от того, чтобы вносить его в протокол, но я должен зачитать фрагмент под протокол.
MAJOR WALSH It is such a lengthy document, I hesitated to burden the record with so much of it, and had extracted certain portions therefrom, but I shall be very glad to read it into the record.
COMMANDANT WALSH C’est un document très long; je ne voulais pas en encombrer le procès-verbal et j’en ai extrait certains passages; mais je suis prêt à le lire si vous le désirez.
MAJOR WALSH Es ist ein so langes Dokument, daß ich zögerte, das Protokoll mit so viel zu belasten; ich habe nur gewisse Teile herausgezogen; doch bin ich gerne bereit, es in das Protokoll zu verlesen.
48206
Председатель«В данном случае, например, муниципальная администрация Львова не смогла заселить евреев в закрытом районе, который населяли бы только евреи. Вопрос был быстро разрешён руководителем СС и полиции и его подчинёнными».
THE PRESIDENT «Then, for instance, the Municipal Administration at Lvov had no success in their attempts to house the Jews within a closed district which would be inhabited only by Jews. This question, too, was solved quickly by the SS and Police Leader through his subordinate officials.»
LE PRÉSIDENT «Ainsi, par exemple, l’administration municipale de Lwow ne réussit pas dans ses efforts pour concentrer les Juifs dans une zone séparée qui serait habitée par eux seuls. Cette question, elle aussi, fut résolue rapidement par le chef de la SS et de la Police, qui fit agir ses subordonnés.»
VORSITZENDER «Da wiederholte Versuche der Stadtverwaltung Lemberg zum Beispiel, die Juden in einem abgeschlossenen jüdischen Wohnbezirk unterzubringen, scheiterten, wurde kurzerhand auch diese Frage vom SS — und Polizeiführer mit seinen Organen gelöst.»
49207
УолшС разрешения суда, я добавляю это в протокол: Читаю последний абзац на странице 2:
MAJOR WALSH With the Court's permission, I add that to the record. Reading the last paragraph of Page 2:
COMMANDANT WALSH Avec la permission du Tribunal, j’ajouterai ceci au procès-verbal. Je lis le dernier paragraphe de la page 2:
MAJOR WALSH Mit Genehmigung des Gerichtshofs füge ich dies dem Protokoll zu. Der letzte Absatz auf Seite 2 lautet wie folgt:
208
Когда евреев помечают звездой Давида, когда регистрируют трудовые ведомства, проявляются первые признаки их попыток саботировать приказы властей. Меры, которые вызываются этим приводят к тысячам арестов. Всё более и более очевидным становится, что гражданская администрация не в состоянии решить еврейскую проблему удовлетворительным образом. «В данном случае, например, муниципальная администрация Львова не смогла заселить евреев в закрытом районе, который населяли бы только евреи. Вопрос был быстро разрешён руководителем СС и полиции и его подчинёнными. Эти меры стали чрезвычайно срочными так как зимой 1941 во многих частях города появились крупные очаги сыпного тифа…
When the Jews were marked by the Star of David, as well as when they were registered by the labor agencies, the first symptoms appeared in their attempts to dodge the order of the authorities. The measures which were introduced thereupon led to thousands of arrests. It became more and more apparent that the civil administration was not in a position to solve the Jewish problem in an approximately satisfactory manner. Then, for instance, the municipal administration at Lvov had no success in their attempts to house the Jews within a closed district which would be inhabited only by Jews. This question, too, was solved quickly by the SS and Police Leader through his subordinate officials. This measure became the more urgent as in the winter of 1941 big centers of spotted fever were noted in many parts of the town....
Lorsque les Juifs furent marqués de l’étoile de David, et lorsqu’ils furent recencés par les bureaux de placement, ils essayèrent de tourner les ordres des autorités. Les mesures que ceci entraîna conduisirent à des milliers d’arrestations. Il devint de plus en plus évident que l’administration civile n’était pas en état de résoudre le problème juif d’une manière à peu près satisfaisante. Ainsi, par exemple, l’administration municipale de Lwow n’a pas réussi dans ses efforts pour concentrer les Juifs dans une zone séparée qui serait habitée par eux seuls. Cette question, elle aussi, fut résolue par le chef des SS et de la Police, qui fit agir ses subordonnés. Cette mesure devenait des plus urgentes car, au cours de l’hiver de 1941, des foyers importants de typhus exanthématique furent signalés en de nombreux points de la ville.
Sowohl bei der Kenntlichmachung der Juden mit dem Davidstern als auch bei der Registrierung durch die Arbeitsämter, machten sich schon die ersten Anzeichen bemerkbar, daß die Juden sich den behördlichen Anordnungen zu entziehen versuchten. Die darauf durchgeführten Kontrollmaßnahmen führten zu Tausenden von Festnahmen. Es zeigte sich immer mehr, daß die Zivilverwaltung nicht in der Lage war, das Judenproblem auch nur einer annähernd befriedigenden Lösung zuzuführen. Da wiederholte Versuche der Stadtverwaltung Lemberg zum Beispiel, die Juden in einem abgeschlossenen jüdischen Wohnbezirk unterzubringen, scheiterten, wurde kurzerhand auch diese Frage vom SS — und Polizeiführer mit seinen Organen gelöst. Diese Maßnahme wurde um so vordringlicher, da allenthalben im Stadtgebiet in den Wintermonaten 1941 große Fleckfieberzentren auftraten....
209
И на странице 5 этого документа, Л-18, последняя часть абзаца 1, я читаю:
And on Page 5 of this document, L-18, last half of Paragraph 1, I read:
Je lis maintenant la dernière moitié du paragraphe 1 de la page 5, du document L-18:
Sodann auf Seite 5 dieses Dokuments L-18 heißt es im letzten Teil des ersten Absatzes:
210
Во время вывода евреев в отдельный квартал города были построены несколько шлюзов в которых был выявлен весь безработный и асоциальный еврейский сброд. В связи со значимым фактом, что 90 процентов ремесленников в Галиции были евреями, задачу следовало решать шаг за шагом, поскольку немедленная эвакуация не могла послужить интересам военной экономики».
During the removal of the Jews into a certain quarter of the town several sluices were erected at which all the work-shy and asocial Jewish rabble were caught during the screening and treated in a special way. Owing to the peculiar fact that almost 90 percent of artisans working in Galicia were Jews, the task to be solved could be fulfilled only step by step, since an immediate evacuation would not have served the interest of war economy.
Au cours du transfert des Juifs dans un quartier spécial de la ville, on procéda à plusieurs filtrages qui permirent de saisir les Juifs inassimilables et ceux qui se refusaient au travail et de les soumettre à un traitement spécial. Étant donné que près de 90 % des artisans travaillant en Galicie étaient des Juifs, cette tâche ne put être accomplie que petit à petit, car une évacuation immédiate n’aurait pas servi les intérêts de l’Économie de Guerre.
Bei dieser Umsiedelung der Juden in ein bestimmtes Stadtviertel wurden mehrere Schleusen errichtet, an denen von vornherein bei der Durchschleusung das gesamte arbeitsscheue und asoziale jüdische Gesindel erfaßt und sonderbehandelt wurde. Durch die Eigenart, daß das Handwerkertum in Galizien fast zu 90 % aus jüdischen Arbeitskräften bestand, konnte die zu lösende Aufgabe nur Zug um Zug durchgeführt werden, da eine sofortige Entfernung nicht im Interesse der Kriegswirtschaft gelegen hätte.
211
И снова, на странице 5, абзаце 2, последняя часть, начиная со слов «выявлены случаи»:
And again, on Page 5, Paragraph 2, the latter part, beginning with «cases were discovered»:
Je cite encore, à la page 5, la fin du paragraphe 2:
Und wiederum auf Seite 5 im letzten Teil des zweiten Absatzes, beginnend mit: «Es wurden Fälle bekannt... «:
212
Выявлены случаи в которых евреи, для того, чтобы получить какой-либо сертификат о работе, не только отказывались от заработной платы, но и сами платили деньги. Более того, организация евреев в интересах их работодателей возросла в такой катастрофической степени, что оказалось необходимым от немецкого имени вмешаться самыми крайними мерами.
Cases were discovered where Jews, in order to acquire any certificate of labor, not only renounced all wages but even paid money themselves. Moreover, the organizing of Jews for the benefit of their employers grew to such catastrophical extent that it was deemed necessary to interfere in the most energetic manner for the benefit of the German name.
Dans certains cas, des Juifs, afin d’obtenir un certificat de travail, non seulement renonçaient à toucher un salaire, mais encore payaient une somme d’argent. Bien plus, les combinaisons juives profitables aux employeurs atteignirent des proportions tellement catastrophiques que l’on jugea nécessaire d’intervenir avec toute l’énergie voulue dans l’intérêt du nom allemand.
Es wurden Fälle bekannt, bei denen Juden zwecks Erlangung irgendeines Arbeitsausweises nicht allein keinen Lohn verlangten, sondern sogar noch laufend Geld zuzahlten. Darüber hinausnahm das ›Organisieren‹ der Juden für ihre ›Arbeitgeber‹ einen derartig katastrophalen Umfang an, daß im Interesse des Ansehens des Deutschtums energischst eingeschritten werden mußte.
213
Так как администрация не оказалась не в состоянии и показал себя слишком слабой, что управиться с этим хаосом, руководитель СС и полиции просто принял на себя назначение работы евреям. Все трудовые книжки выданные фирмами или административными ведомствами были признаны недействительными, карточки выданные евреям трудовыми ведомствам удостоверялись полицией путём их проштамповки. В ходе акции, снова, были пойманы тысячи евреев, которые имели поддельные трудовые книжки или которые получили трудовые книжки под разного рода предлогами. Этих евреев направили на особое обращение.
Since the administration was not in a position and showed itself too weak to master this chaos, the SS and Police leader simply took over the entire disposition of labor for Jews. The Jewish labor agencies, which were manned by hundreds of Jews, were dissolved. All certificates of labor given by firms or administrative offices were declared invalid, and the cards given to the Jews by the labor agencies were validated by the police offices by stamping them. In the course of this action, again, thousands of Jews were caught who were in possession of forged certificates or who had obtained, surreptitiously, certificates of labor by all kinds of pretexts. These Jews also were exposed to special treatment.
L’administration n’étant pas en état de venir à bout de ce chaos et s’étant avérée trop faible pour cela, le chef des SS et de la Police assuma entièrement la direction de la main-d’œuvre juive. Les bureaux de placement juifs, tenus par des centaines de Juifs, furent fermés. Tous les certificats de travail par des entreprises ou des administrations furent déclarés sans validité et les cartes données aux Juifs par les bureaux de placement furent validées à nouveau par les bureaux de la Police qui y apposèrent un tampon de contrôle. À cette occasion, de nouveau, on prit des milliers de Juifs qui étaient en possession de faux certificats ou qui avaient obtenu sous de faux prétextes, subrepticement, des certificats de travail. Ces Juifs subirent également un traitement spécial.
Da die Verwaltung nicht in der Lage war und sich zu schwach zeigte, dieses Chaos Herr zu werden, wurde kurzerhand der gesamte Arbeitseinsatz der Juden vom SS — und Polizeiführer übernommen. Die bestehenden jüdischen Arbeitsämter, die mit Hunderten von Juden besetzt waren, wurden aufgelöst, sämtliche Arbeitsbescheinigungen von Firmen und Dienststellen für ungültig erklärt und die von den Arbeitsämtern den Juden gegebenen Karten durch Abstempelung der Polizei-Dienststellen neu gültig gemacht. Im Zuge dieser Aktion wurden wiederum Tausende von Juden erfaßt, die sich im Besitze von gefälschten Ausweisen befanden, oder aber sich unter allen möglichen Vorwänden Arbeitsausweise erschlichen hatten. Auch diese Juden wurden einer Sonderbehandlung zugeführt.
214
С позволения суда, сейчас я хочу организовать показ очень коротко фильма, вероятно одного из самых необычных экземпляров которые будут приобщены на процессе. С разрешения суда я хочу попросить коммандера Донована помочь мне.
If the Court please, at this time I would like to arrange for the showing of a very short motion picture, perhaps one of the most unusual exhibits that will be presented during the Trial. With the Courts permission I would like to call upon Commander Donovan to assist.
Plaise au Tribunal. Je voudrais montrer maintenant un film très court, qui est peut-être une des pièces les plus extraordinaires de celles qui seront produites au cours de ce Procès, et demander au commandant Donovan de le commenter.
Wenn der Gerichtshof gestattet, möchte ich jetzt einen kurzen Film vorführen lassen, vielleicht eines der ungewöhnlichsten Beweisstücke, die dem Gerichtshof während dieser Verhandlungen vorgelegt werden. Mit Erlaubnis des Gerichtshofs bitte ich um die Unterstützung des Fregattenkapitäns Donovan.
50215
ПредседательНужно откладываться или нет?
THE PRESIDENT Need we adjourn for it or not?
LE PRÉSIDENT L’audience doit-elle être suspendue?
VORSITZENDER Müssen wir eine Vertagung einlegen?
51216
Уолш Нет, сэр. Фильм очень, очень короткий.
MAJOR WALSH No, Sir. The movie itself is very, very short, Sir.
COMMANDANT WALSH Non, Votre Honneur. Le film est extrêmement court.
MAJOR WALSH Nein, Herr Präsident, der Film selbst ist sehr, sehr kurz.
52217
Председатель Очень хорошо.
THE PRESIDENT Very well.
LE PRÉSIDENT Très bien.
VORSITZENDER Gut.
14
Помощник Главного обвинителя от США У. Уолш представляет сборник документов Т: «Преследование евреев».Продолжение от 13 декабря 1945 г. (14.12.1945)
3563
ПредседательИтак, я вызываю майора Уолша. Майор Уолш, вы присвоили букву документальной книге которую вы рассматриваете?
THE PRESIDENT Now, I call upon Major Walsh. Major Walsh, did you give a lettering to the document book with which you are dealing?
LE PRÉSIDENT Maintenant, je demande au Commandant Walsh de prendre la parole: Commandant Walsh, avez-vous donné un indicatif au livre de documents que vous présentez?
VORSITZENDER Ich erteile nun Major Walsh das Wort. Ist das Dokumentenbuch, über das Sie sprechen werden, mit einem Buchstaben gekennzeichnet?
3664
Уолш Да. С позволения вашей чести, это буква «Т». С позволения трибунала, в ходе последнего заседания обвинение кратко представило предварительные шаги ведущие к окончательной цели нацистской партии и нацистского государства, то есть, уничтожению евреев. Пропаганда, декреты, позорные нюренбергские законы, бойкоты, регистрация и «геттоизация» являлись первоначальными мерами данной программы. С разрешения Трибунала я продолжу рассмотрение методов уничтожения евреев.
MAJOR WALSH Yes. If Your Honor please, it is the letter "T." May it please the Tribunal, during the last session the Prosecution presented briefly the preliminary steps leading to the ultimate objective of the Nazi Party and the Nazi-controlled State, that is, the extermination of the Jews. Propaganda, decrees, the infamous Nuremberg Laws, boycotts, registration, and "ghettoization" were the initial measures in the program. I shall, with the Court's permission, continue with a discussion of the methods utilized for the annihilation of the Jewish people.
COMMANDANT WALSH Oui, Monsieur le Président, c’est la lettre «T». Plaise au Tribunal. Durant la dernière séance le Ministère Public a présenté brièvement les préliminaires conduisant au but suprême du parti nazi et de l’État sous le contrôle nazi: ce but est l’extermination des Juifs. La propagande, les décrets, les lois infâmes de Nuremberg, le boycottage, l’établissement de registres, le maintien des ghettos furent les mesures initiatrices de ce programme; je continuerai, avec la permission du Tribunal en exposant les méthodes utilisées pour annihiler le peuple juif.
MAJOR WALSH Ja, Herr Präsident, mit dem Buchstaben»T«. Während der letzten Sitzung legte die Anklagebehörde kurz die vorbereitenden Maßnahmen dar, die zu dem Endziel der Nazi-Partei und der durch die Nazis kontrollierten Staaten führten, nämlich zur Ausrottung der Juden. Propaganda, Erlasse, die berüchtigten Nürnberger Gesetze, Boykotts, Registrierung und Ghettoisierung waren die ersten Maßnahmen in diesem Programm. Mit Erlaubnis des Hohen Gerichtshofs werde ich die Erörterung der zur Vernichtung des jüdischen Volkes angewandten Methoden fortsetzen.
65
Сначала я хотел бы остановиться на доказательствах умерщвления евреев голодной смертью. Была разработана и осуществлялась политика лишения евреев самых необходимых средств существования. Подсудимый Ганс Франк записал в своем дневнике, что в польском гетто были установлены голодные нормы, и, ссылаясь на новые правила продовольственного снабжения, введенные в августе 1942 года, он бессердечно и, по-видимому, мимоходом указал, что, используя эти правила, он приговорил к голодной смерти более чем один миллион евреев. Я представляю в качестве доказательства дневник Ганса Франка, запись от 24 августа 1943 г. Документ ПС-2233(е), США-283.
I would like first to discuss starvation. Policies were designed and adopted to deprive the Jews of the most elemental necessities of life. Again the Defendant Hans Frank, then Governor General of Poland, wrote in his diary that hunger rations were introduced in the Warsaw ghetto; and referring to the new food regulations in August 1942, he callously, and perhaps casually, noted that by these food regulations he virtually condemned more than 1 million Jews to death. I offer in evidence that part of Document 2233(e)-PS, diary of Hans Frank, "Conference Volume," 24 August 1942, Exhibit USA-283. And I quote:
J’aimerais parler d’abord de la mort lente par la faim; une politique fut tracée et suivie pour priver les Juifs des nécessités les plus élémentaires de l’existence. L’accusé Hans Frank, alors Gouverneur Général de Pologne, écrivit dans son journal que des rations de famine furent allouées au ghetto de Varsovie et parlant du nouveau régime alimentaire d’août 1942, il note avec dureté, par hasard peut-être, que ces restrictions alimentaires ont de fait, condamné à mort plus d’un million de Juifs.
Zuerst möchte ich über Aushungerung sprechen. Richtlinien, die darauf abzielten, die Juden der elementarsten Lebensnotwendigkeiten zu berauben, wurden entworfen und ausgeführt. Der Angeklagte Hans Frank, damals Generalgouverneur von Polen, schrieb in seinem Tagebuch, daß Hungerrationen im Warschauer Ghetto eingeführt worden seien. Zu den neuen Ernährungsvorschriften vom August 1942 bemerkt er gefühllos und ziemlich beiläufig, daß er durch diese Vorschriften tatsächlich über eine Million Juden zum Tode verurteilt habe. Ich lege jenen Teil der Urkunde 2233(e)-PS, des Tagebuchs von Hans Frank, Korrespondenzband vom 24. August 1942, zum Beweis vor; es ist US-283. Ich zitiere:
66
Тот факт, что мы приговариваем 1 миллион 200 тысяч евреев к голодной смерти, следует отметить только мимоходом. Само собою разумеется, что если евреи не умрут с голоду, то мы надеемся, это приведет к активизации антиеврейских мероприятий.
That we sentence 1,200,000 Jews to die of hunger should be noted only marginally. It is a matter of course that should the Jews not starve to death it would, we hope, result in a speeding up of the anti-Jewish measures.
Je dépose comme preuve cette partie du document PS-2233 (e), journal de Hans Frank, volume de conférences du 24 août 1942, (USA-283). – Et je cite: «que nous condamnions 1.200.000 Juifs à mourir de faim devrait être noté seulement en marge. Il est bien entendu que si les Juifs ne meurent pas de faim, il en résultera, nous l’espérons, une aggravation des mesures anti-juives .
Daß wir 1,2 Millionen Juden zum Hungertod verurteilen, sei nur am Rande festgestellt. Es ist selbstverständlich, daß ein Nichtverhungern der Juden hoffentlich eine Beschleunigung der antijüdischen Maßnahmen zur Folge haben wird.
67
Дневник Франка не является единственным гидом в политике рукотворного голода евреев. Евреям запрещалось заниматься сельским хозяйством с целью закрыть им доступ к источникам продовольствия, о чем свидетельствует документ ПС-1138, США-284, озаглавленный: «Временные директивы относительно обращения с евреями». Директивы изданы рейхскомиссаром Остланда.
Frank's diary was not the only guide to the deliberate policy of starvation of the Jews. They were prohibited from pursuing agricultural activities in order to cut them off from access to the source of food. I offer Document 1138-PS in evidence, Exhibit USA-284. I refer the Court to Page 4 of the translation, marked with numeral V, Paragraphs a and b. The document is entitled "Provisional Directive on the Treatment of Jews..." and it was issued by the Reich Commissioner for the Ostland. I read:
Le journal de Frank n’est pas le seul guide qui nous renseigne sur la politique délibérée de destruction des Juifs par la faim. Il leur fut défendu d’exercer des professions agricoles de façon à leur interdire tout accès aux sources mêmes de la nourriture. Je dépose le document PS-1138 sous la cote USA-284 et je prie le Tribunal de se référer à la page 4 de la traduction marquée du chiffre romain V, paragraphes a et b; le document a pour titre «Directives provisoires pour le traitement des Juifs» et il émane du Reichskommissar des territoires de l’Est. Je lis:
Franks Tagebuch war nicht der einzige Hinweis auf die vorsätzliche, gegen die Juden angewandte Aushungerungspolitik. Sie durften keine landwirtschaftliche Tätigkeit verrichten, damit sie zu Nahrungsmittelquellen keinen Zutritt erhielten. Ich lege Dokument 1138-PS, Beweisstück US-284, vor. Ich verweise den Gerichtshof auf Seite 4 der Übersetzung, unter der römischen Zahl V, Absätze a und b. Die Urkunde trägt den Titel:»Vorläufige Richtlinien für die Behandlung der Juden...«; sie waren vom Reichskommissar für das Ostland herausgegeben. Ich lese vor:
68
Евреев надо изгонять из сельской местности. Евреев надо изгонять из всех областей торговли, особенно из торговли сельскохозяйственными продуктами и другими продуктами питания.
Jews must be cleaned out from the countryside. The Jews are to be removed from all trades, especially from trade with agricultural products and other foodstuffs.
Les Juifs doivent disparaître du pays. Ils doivent être expulsés de tous commerces, et spécialement des commerces de produits agricoles et produits alimentaires .
Das flache Land ist von Juden zu säubern. Die Juden sind aus dem gesamten Handel, vordringlich aber aus dem Handel mit landwirtschaftlichen Erzeugnissen und anderen Lebensmitteln zu entfernen.
69
Евреям запрещалось покупать основные продукты питания, такие, как мучные изделия, мясо, яйца и молоко.
Jews were excluded from the purchase of basic food, such as wheat products, meat, eggs, and milk.
Les Juifs furent exclus du commerce des produits alimentaires de base comme les farines, la viande, les œufs et le lait.
Juden wurden vom Einkauf lebenswichtiger Nahrungsmittel, wie Weizenprodukte, Fleisch, Eier und Milch ausgeschlossen.
70
Я приобщаю в качестве доказательства документ 1347-ПС, экземпляр США-285 и я цитирую из абзаца 2 на первой странице перевода суда. Это оригинал указа от 18 сентября 1942 из министерства сельского хозяйства. Я цитирую:
I offer in evidence Document 1347-PS, Exhibit USA-285, and I quote from Paragraph 2 on the first page of the translation before the Court. This is an original decree, dated 18 September 1942, from the Ministry of Agriculture. I quote:
Je présente comme preuve le document PS-1347 sous la cote USA-285 et je cite le paragraphe 2 de la première page de la traduction que le Tribunal a entre les mains. C’est un décret en date du 18 septembre 1942 du ministère de l’Agriculture. Je cite:
Ich lege die Urkunde 1347-PS, Beweisstück US-285, vor und zitiere aus dem zweiten Absatz der ersten Seite der Übersetzung. Dies ist ein Originalerlaß des Reichsministers für Ernährung und Landwirtschaft vom 18. September 1942. Ich zitiere:
71
Начиная с 42-го периода распределения (19 октября 1942) евреи не получают следующие продукты: мясо, мясные продукты, яйца, продукты из пшеницы (печенье, белый хлеб, пшеничные лепёшки, пшеничную муку, и т.д.), цельное молоко, сливки, а также продовольствие не распределяемое по единым продуктовым карточкам Рейха, а по местным сертификатам или особыми распоряжениями продовольственных отделов по дополнительным талонам к продуктовым карточкам. Еврейские дети и подростки старше 10 лет получают хлебные паёк обычного потребителя.
Jews will no longer receive the following foods, beginning with the 42d distribution period (19 October 1942): meat, meat products, eggs, wheat products (cake, white bread, wheat rolls, wheat Dour, et cetera), whole milk, fresh skimmed milk, as well as such food distributed not on food ration cards issued uniformly throughout the Reich but on local supply certificates or by special announcement of the nutrition office on extra coupons of the food cards. Jewish children and young people over 10 years of age will receive the bread ration of the normal consumer.
Les Juifs ne recevront plus les aliments suivants, à partir de la 42e distribution (19 octobre 1942): viande, aliments carnés, œufs, produits farineux (gâteaux, pain blanc, petits pains, farine de blé, etc.), lait entier, lait écrémé frais, aussi bien que les aliments distribués en dehors des cartes d’alimentation et délivrés uniformément dans tout le Reich mais avec des certificats de communes ou par avertissement spécial de l’office de nutrition ou sur coupons spéciaux des cartes d’alimentation. Les enfants juifs et les jeunes gens au-dessus de dix ans recevront la ration normale de pain.
Juden erhalten von der 42. Zuteilungsperiode (19. Oktober 1942) ab folgende Lebensmittel nicht mehr: Fleisch, Fleischwaren, Eier, Weizenerzeugnisse (Kuchen, Weißbrot, Weizenkleingebäck, Weizenmehl usw.), Vollmilch, entrahmte Frischmilch, desgleichen solche Lebensmittel, die nicht auf reichseinheitlich eingeführte Lebensmittelkarten, sondern auf örtliche Bezugsausweise oder durch Sonderaufrufe der Ernährungsämter auf freie Abschnitte der Lebensmittelkarten abgegeben werden. Jüdische Kinder und Jugendliche über 10 Jahre erhalten die Brotrationen der Normalverbraucher.
72
Больные, старые и беременные матери исключались из специальных продовольственных мер которые направляли неевреям. Государственной полиции было поручено изымать продовольственные посылки евреям, присылаемые из-за границы. Еврейские продовольственные карточки имели на лицевой стороне цветную надпись «еврей», чтобы владельцы магазинов могли сразу определить национальность владельца и ограничить закупки евреев.
The sick, the old, and the pregnant mothers were excluded from the special food concessions allotted to non-Jews. Seizure by the State Police of food shipments to Jews from abroad was authorized, and the Jewish ration cards were distinctly marked with "Jew," in color, across the face of the cards, so that the storekeepers could readily identify and discriminate against Jewish purchasers.
Les malades, les vieillards et les femmes enceintes n’avaient pas droit au régime spécial accordé aux non-juifs. Les envois de l’étranger pour les Juifs furent saisis et les cartes d’alimentation des Juifs furent marquées du mot «Juif» en couleur sur la couverture, afin que les commerçants puissent les identifier rapidement et ainsi faire la discrimination.
Die Kranken, Greise und schwangeren Mütter wurden von den besonderen Lebensmittelzuschüssen, die Nichtjuden zustanden, ausgeschlossen. Die Beschlagnahme von Lebensmittelsendungen aus dem Ausland an Juden durch die Staatspolizei war erlaubt. Die jüdischen Lebensmittelkarten trugen eine farbige, deutlich erkennbare, quer über die Karte gedruckte Aufschrift»Jude«, so daß der Kaufmann sofort den jüdischen Kunden erkennen und benachteiligen konnte.
73
Чехословацкое правительство в 1943 опубликовало официальный документ под названием «Чехословакия сражается». Я приобщаю эту книгу в качестве доказательства, документ 1689-ПС, экземпляр США-286.Подытоживая содержание на странице 110, она говорит, что поставки для евреев ограничивались отдельными районами и отдельными днями и часами. Как можно было ожидать, период закупок попадал на время, когда продовольственные склады пустели.
The Czechoslovakian Government published in 1943 an official document entitled "Czechoslovakia Fights Back;." I offer this book in evidence, Document 1689-PS, Exhibit USA-286. To summarize the contents of Page 110, it states that the Jewish food purchases were confined to certain areas and to certain days and hours. As might be expected, the period permitted for the purchases was during the time when food stocks were likely to be exhausted.
Le Gouvernement tchécoslovaque publia en 1943 un document officiel intitulé «La Tchécoslovaquie riposte» et je dépose ce livre comme preuve (document PS-1689, USA-286); en résumant le contenu de la page 110, nous voyons que les Juifs ne pouvaient faire d’achats alimentaires que dans une certaine zone, à jours et à heures spécifiés. Comme on peut s’y attendre, les heures autorisées étaient celles où les stocks de nourriture étaient vraisemblablement épuisés.
Die tschechische Regierung gab im Jahr 1943 ein offizielles Dokument unter dem Titel»Die Tschechoslowakei wehrt sich«heraus. Ich lege dieses Buch als Dokument 1689-PS, US-286, vor. Den Inhalt der Seite 110 will ich zusammenfassen. Es heißt dort, daß jüdische Käufer von Lebensmitteln nicht nur an bestimmte Bezirke, sondern auch an bestimmte Tage und Stunden gebunden waren. Wie man sich denken kann, waren die Stunden für den Einkauf von Lebensmitteln auf Tageszeiten festgesetzt, zu denen die Nahrungsmittel gewöhnlich ausverkauft waren.
74
По специальному приказу номер 44 от 4 ноября 1941 г. в восточных оккупированных областях для евреев устанавливались нормы, которые составляли лишь половину самых низших норм для остальной части населения. Министерству сельского хозяйства предписывалось целиком или частично лишать евреев возможности приобретать продовольствие. Таким образом, еврейское население обрекалось на голодную смерть.
By Special Order Number 44 for the Eastern Occupied Territories, dated 4 November 1991, the Jews were limited to rations as low as only one-half of the lowest basic category of other people; and the Ministry of Agriculture was empowered to exclude Jews entirely or partially from obtaining food, thus exposing the Jewish community to death by starvation.
Par ordonnance spéciale nº 44 des territoires occupés de l’Est, en date du 4 novembre 1941, fut allouée aux Juifs une ration qui était à peu près la moitié de la ration la plus faible prévue comme base et le ministre de l’Agriculture reçut le pouvoir d’exclure les Juifs totalement ou partiellement du rationnement alimentaire, réduisant ainsi la communauté juive à la mort par famine.
Auf Grund der besonderen Anordnung Nummer 44 für die besetzten Ostgebiete vom 4. November 1941 wurden die Rationen der Juden so herabgesetzt, daß sie sich nur auf die Hälfte der niedrigsten Rations-Kategorie anderer Leute beliefen, und das Landwirtschaftsministerium war ermächtigt, Juden ganz oder teilweise von der Nahrungsmittel-Belieferung auszuschließen und so die jüdische Einwohnerschaft dem Hungertode auszusetzen.
75
Я приобщаю в качестве доказательства документ Л-165.
I now offer in evidence Document L165.
Je dépose en preuve le document L-165.
Ich unterbreite nun als Beweis Dokument L-165.
3776
ПредседательВы, что-нибудь читали из 1689-ПС?
THE PRESIDENT Did you read anything from 1689-PS?
LE PRÉSIDENT Avez-vous lu un passage du document PS-1689?
VORSITZENDER Haben Sie etwas aus 1689-PS verlesen?
3877
Уолш Сэр, только подытожил, содержание страницы 110.
MAJOR WALSH to summarize, Sir, the contents of Page 110.
COMMANDANT WALSH Je n’ai fait que résumer le contenu de la page 110.
MAJOR WALSH Ich habe nur den Inhalt der Seite 110 zusammengefaßt.
3978
Председатель Я понял. Сейчас вы приобщаете Л…
TO PRESIDENT I see. Now you are offering L...
LE PRÉSIDENT D’accord. Maintenant vous présentez le document L.
VORSITZENDER Gut. Nun unterbreiten Sie L....
4079
Уолш Ваша честь, Л-165, экземпляр США-287. Я обращаюсь к последней половине первого абзаца перевода. Это информационный бюллетень издаваемый польским министерством информации от 15 ноября 1942. Польское министерство делает вывод о том, согласно характеру раздельных пайков и количества доступного для евреев продовольствия в гетто Варшавы и Кракова, была создана система ведущая к голоду; и я цитирую:
MAJOR WALSH L-165, Your Honor, Exhibit USA-287. I refer the Court to the last half of the first paragraph of the translation. This is a press bullet-in Issued by the Polish Ministry of Formation, dated 15 November 1942. The Polish Ministry concludes that, upon the basis of the nature of the separate rationing and the amount of food available to Jews in the Warsaw and Krakow ghettos, the system was designed to bring about starvation; and from the quotation I read:
COMMANDANT WALSH L-165, Monsieur le Président, (USA-287). Je demande au Tribunal de se reporter à la dernière moitié du premier paragraphe de la traduction; c’est un bulletin de presse du ministère polonais de l’Information en date du 15 novembre 1942; le ministre polonais conclut que les rations spéciales et les quantités de nourriture accordées aux Juifs dans les ghettos de Varsovie et de Cracovie, étaient calculées dans le dessein de produire la mort lente par la famine.
MAJOR WALSH L-165, Beweisstück US-287, Herr Präsident. Ich verweise den Gerichtshof auf die letzte Hälfte des ersten Absatzes der Übersetzung. Dies ist eine vom polnischen Informationsministerium am 15. November 1942 herausgegebene Presse-Bekanntmachung. Das polnische Ministerium kommt zu dem Schluß, daß das System der Sonder-Rationierung und der Kürzung der Nahrungsmittelzuweisung für Juden in den Ghettos von Warschau und Krakau den Hungertod bezweckte. Ich verlese aus dem Zitat:
80
В отношении поставок продовольствия они организованы полностью раздельно, что очевидно направлено на их лишение самых элементарных жизненных благ.
In regard to food supplies they are brought under a completely separate system, which is obviously aimed at depriving them of the most elemental necessities of life.
Je cite: «Pour ce qui est des rations de nourriture, elles sont calculées d’après un système tout à fait distinct qui a pour but évident de les priver des nécessités les plus élémentaires de l’existence.
Es gibt ein besonderes System für ihre Nahrungsmittelversorgung, und dieses zielt offensichtlich darauf hin, ihnen die elementarsten Notwendigkeiten des Lebens zu verweigern.
81
Перехожу к теме уничтожения евреев в пределах гетто. Господин Джексон в своей вступительной речи уже ссылался на документ ПС-1061, США-275, озаглавленный: «Варшавское гетто больше не существует».
I would now like to discuss annihilation within the ghettos. Justice Jackson in his opening address to the Tribunal made reference to Document 1061-PS, "The Warsaw Ghetto Is No More," marked Exhibit USA-275.
Je voudrais maintenant parler de la suppression des Juifs à l’intérieur des ghettos. M. Justice Jackson, dans son discours d’ouverture, se réfère à un document PS-1061, intitulé «Le ghetto de Varsovie n’est plus» et qui porte la cote USA-Exhibit 275,
Ich möchte nunmehr von der Ausrottung innerhalb der Ghettos sprechen. Justice Jackson erwähnte in seiner Eröffnungsrede das Dokument 1061-PS.»Es gibt keinen jüdischen Wohnbezirk in Warschau mehr!«; es ist als Beweisstück US-275 bezeichnet.
82
Это — целый том, переплетенный в кожу, со многими иллюстрациями и отпечатанный на бумаге высшего качества. Он представляет собой образец германского ремесленнического мастерства. Это отчет о деятельности генерал — майора полиции Штроопа. Генерал Штрооп вначале превозносит «мужество» и «героизм» германских вооруженных сил, участвовавших в жестоких и беспощадных действиях против беззащитных евреев, которых насчитывалось 56065 человек, в том числе, конечно, женщин и детей. Затем в этом документе он описывает день за днем, как производилось выполнение его задания — разрушить и смести с лица земли варшавское гетто.
This finest example of ornate German craftsmanship, leather bound, profusely illustrated, typed on heavy bond paper, is the almost unbelievable recital of a proud accomplishment by Major General of the Police Stroop, who signed the report with a bold hand. General Stroop in this report first pays tribute to the bravery and heroism of the German forces who participated in the ruthless and merciless action against a helpless, defenseless group of Jews, numbering, to be exact, 56,065, including, of course, the infants and the women. In this document he proceeds to relate the day-by-day account of the ultimate accomplishment of his mission-to destroy and to obliterate the Warsaw ghetto.
C’est un superbe exemple du travail soigné de l’artisan allemand, relié en cuir, contenant de nombreuses illustrations, imprimé sur papier épais et c’est le récit presque incroyable des hauts faits du Generalmajor de la Police, Stroop qui a bravement signé de sa main. Dans ce rapport, il rend hommage à la bravoure et à l’héroïsme des Forces allemandes qui participèrent à l’action impitoyable exécutée contre un groupe de Juifs sans défense, comptant exactement 56.065 personnes, y compris naturellement femmes et enfants. Dans ce document, il entreprend de raconter le compte rendu au jour le jour de l’accomplissement définitif de sa mission: détruire et effacer de la carte le ghetto de Varsovie.
Dieses ausgezeichnete Probestück deutscher Handwerkskunst, in Leder eingebunden, reich illustriert, auf schwerem Büttenpapier gedruckt, enthält einen beinahe unglaublich klingenden Bericht einer stolzen Leistung des Generalmajors der Polizei, Stroop, der diesen Bericht mit kühner Hand unterzeichnet hat. General Stroop rühmt in diesem Bericht erst die Tapferkeit und das Heldentum der deutschen Streitkräfte, die an der rücksichtslosen und erbarmungslosen Aktion gegen eine hilf- und schutzlose Gruppe von Juden teilnahmen, die sich, um genau zu sein, auf 56065 Personen belief, selbstverständlich einschließlich Frauen und Kinder. In diesem Dokument gibt er eine tägliche, fortlaufende Beschreibung der schließlichen Erfüllung seiner Aufgabe: Der Zerstörung und Vernichtung des Warschauer Ghettos.
83
Согласно этому отчету, в гетто, созданном в Варшаве в ноябре 1940 года, находилось около 400 тысяч евреев. До начала операции по разрушению этого гетто около 316 тысяч евреев было уже вывезено оттуда.Суд заметит, что данный доклад занимает приблизительно 75 страниц, и обвинению кажется, что содержание имеющее настолько шокирующую доказательственную ценность нельзя опустить для целей материалов дела и что трибунал должен рассмотреть весь доклад, чтобы судить о виновности подсудимых.
According to this report, the ghetto, which was established in Warsaw in November 1940, was inhabited by about 400,000 Jews; and prior to the action for the destruction of this ghetto, some 316,000 had already been deported. The Court will note that this report is approximately 75 pages in length, and the Prosecution believes that the contents are of such striking evidentiary value that no part should be omitted from the permanent records of the Tribunal and that the Tribunal should consider the entire report in judging the guilt of these defendants.
Selon ce récit, le ghetto, tel qu’il existait en novembre 1940, comprenait environ 400.000 Juifs et, avant l’exécution du plan de destruction, quelque 316.000 Juifs avaient déjà été déportés. Le Tribunal remarquera que ce rapport a environ 75 pages et le Ministère Public pense que son contenu est d’une telle force probante qu’aucune partie ne peut être omise des archives permanentes du Tribunal et qu’il devrait envisager le contenu entier du rapport lorsqu’il établira la culpabilité des accusés.
Nach diesem Bericht lebten in dem Ghetto, das im November 1940 in Warschau errichtet wurde, ungefähr 400000 Juden. Vor der Aktion zur Zerstörung dieses Ghettos waren schon etwa 316000 Juden von dort deportiert worden. Der Gerichtshof wird bemerken, daß dieser Bericht etwa 75 Seiten lang ist; die Anklagebehörde ist der Ansicht, daß sein Inhalt solch schlagenden Beweiswert hat, daß keiner seiner Teile in dem Gerichtsprotokoll fehlen und daß der Gerichtshof den ganzen Bericht bei der Beurteilung der Schuld dieser Angeklagten in Betracht ziehen sollte.
84
Для подсудимых подготовили несколько фотокопий полного доклада почти 20 дней назад и они имели время, изучить его в деталях. Если суд, при вынесении приговора, установит, что весь доклад может быть принят целиком, обвинению кажется, что оглашение своего рода итога, вместе с краткими выдержками из ежедневных докладов по телетайпу будет достаточным для устного протокола. Я прошу суд изучить его; и я представляю его суду, вместе с дубликатом оригинала, и прошу суд распорядится о том, что был принят весь документ.
The defendants were furnished with several photostatic copies of the entire document at least 20 days ago and have had ample time, I am sure, to scrutinize it in detail. If the Court, in the exercise of its judgment, determines that the entire report may be accepted in tote, the Prosecution believes that the reading of a portion of the summary, together with brief excerpts from the daily teletype reports, will suffice for the oral record. I would like the Court to examine it; and I present it to the Court, together with the duplicate original thereof, and ask that the Court rule that the entire document may be accepted.
On a remis aux accusés plusieurs photocopies du document il y a au moins vingt jours et ils ont eu amplement le temps, je suis sûr, de l’étudier en détail. Si le Tribunal, dans l’exercice de ses pouvoirs, décide que le rapport peut être accepté en entier, le Ministère Public estime que la lecture d’une partie du résumé, avec de brefs extraits des rapports journaliers télétypés, suffira pour le procès-verbal. Je voudrais que le Tribunal l’examine, je présente ce livre au Tribunal avec un double et lui demande d’accepter le document en entier.
Die Angeklagten haben verschiedene Photokopien dieses Gesamtdokuments vor mindestens 20 Tagen erhalten; ich bin sicher, daß sie genügend Zeit hatten, es gründlich zu prüfen. Falls der Gerichtshof nach seinem Ermessen findet, daß der ganze Bericht vollständig angenommen werden kann, dann glaubt die Anklagebehörde, daß die Verlesung eines Teils der Zusammenfassung, zusammen mit kurzen Auszügen aus den täglichen Fernschreibberichten für das mündliche Protokoll genügen würde. Ich bitte den Gerichtshof, den Bericht zu prüfen, und überreiche ihn zusammen mit dem Duplikat des Originals. Ich bitte um die Entscheidung des Gerichtshofs, dahingehend, daß das ganze Dokument als Beweismittel angenommen wird.
4185
ПредседательМайор Уолш, суд так и сделает, предусмотрев, чтобы обвинение обеспечило как можно быстрее советскую и французскую делегации копиями на русском и французском языках полного документа.
THE PRESIDENT Major Walsh, the Court will take that course, provided that the Prosecution supplies as soon as possible, both to the Soviet and to the French members of the Tribunal, copies in Russian and French of the whole document.
LE PRÉSIDENT Commandant Walsh, le Tribunal accepte pourvu que le Ministère Public fasse parvenir aussi vite que possible aux membres russes et français du Tribunal, des exemplaires en russe et en français du document entier.
VORSITZENDER Major Walsh, der Gerichtshof wird so verfahren, vorausgesetzt, daß die Anklagebehörde sobald wie möglich den sowjetischen sowohl als auch den französischen Mitgliedern des Gerichtshofs Kopien des ganzen Dokuments in russischer und französischer Sprache zur Verfügung stellt.
4286
УолшДа, сэр. Могу я проконсультироваться с…
MAJOR WALSH Yes, Sir; may I consult with . .
COMMANDANT WALSH Oui, Monsieur le Président. Puis-je consulter…?
MAJOR WALSH Ja, Herr Vorsitzender; darf ich mich besprechen mit...
4387
Председатель Я не говорю немедленно, но как можно быстрее.
THE PRESIDENT I do not say present immediately, but present as soon as possible.
LE PRÉSIDENT Je ne dis pas de faire parvenir immédiatement, mais aussitôt que possible.
VORSITZENDER Das braucht nicht sofort zu geschehen, aber so bald wie möglich.
4488
Уолш Да.
MAJOR WALSH Yes.
COMMANDANT WALSH Oui.
MAJOR WALSH Jawohl.
4589
Председатель Вы собираетесь читать отрывки, которые считаете необходимыми?
THE PRESIDENT You are going to read the passages that you think necessary?
LE PRÉSIDENT Vous allez lire les passages que vous jugez indispensables?
VORSITZENDER Werden Sie jetzt die Teile, die Sie für wichtig erachten, verlesen?
4690
УолшДа. Со страницы 6 перевода суда, документ 1061-ПС, я хочу огласить звериный, но вместе с тем откровенный отчёт о некоторых безжалостных акциях в Варшавском гетто. Я цитирую второй абзац, страница 6:
MAJOR WALSH Yes. From Page 6 of the translation before the Court of Document 1061-PS I would like to read the boastful but nonetheless vivid account of some of this ruthless action within the Warsaw ghetto. I quote, second paragraph, Page 6:
COMMANDANT WALSH Oui. De la page 6 de la traduction du document PS-1061 que possède le Tribunal, j’aimerais lire le récit, vantard mais néanmoins expressif, de cette action militaire à l’intérieur du ghetto de Varsovie. Je cite le second paragraphe page 6:
MAJOR WALSH Jawohl. Von Seite 6 der dem Gerichtshof vorliegenden Übersetzung des Dokuments 1061-PS möchte ich die prahlerische, aber dennoch lebhafte Schilderung der rücksichtslosen Aktion im Warschauer Ghetto verlesen. Ich zitiere den zweiten Absatz auf Seite 6:
91
Сопротивление, оказанное евреями и бандитами было сломлено только беспощадным и энергичным использованием наших штурмовых подразделений днем и ночью. 23 апреля 1943 рейхсфюрер СС через высшего руководителя СС и полиции Востока в Кракове отдал свой приказ завершить прочёсывание Варшавского гетто с величайшей жестокостью и беспощадностью. Таким образом, я решил уничтожить весь еврейский жилой район, поджигая каждый квартал, включая кварталы жилых зданий в районе предприятий вооружений. Дом за домом систематически эвакуировались и уничтожались огнём. Евреи стали выходить из своих укрытий почти во всех случаях. Нечасто евреи оставались в горящих зданиях, до тех пор, пока жара и страх сгореть заживо не заставлял их прыгать из окон на матрасы и другие вещи. Переломав свои кости они пытались ползти в те кварталы, которые ещё не горели. Часто евреи меняли свои укрытия ночью пробираясь по сгоревшим зданиям, до тех пор пока их не ловили наши патрули. Их пребывание в канализации перестало быть приятным спустя неделю. Часто на улице мы слышали громкие голоса доносящиеся из канализационных люков. Тогда военнослужащие Ваффен-СС, полиции или инженеры Вермахта отважно спускались в люки, чтобы достать евреев и нередко они оказывались среди мертвых или застрелившихся евреев. Всегда приходилось использовать дымовые гранаты, чтобы выкурить евреев. Так, однажды, мы открыли 183 люка и бросили в них в установленное время дымовые гранаты. В результате бандиты стали бежать, как они думали от газа в центр бывшего гетто, что привело к тому, что они стали вылезать там из канализационных люков. Нельзя определить количество евреев уничтоженных подрывом канализации и укрытий.
The resistance put up by the Jews and bandits could be broken only by the relentless and energetic use of our shocktroops by day and night. On 23 April 1943 the Reichsfuehrer SS issued through the Higher SS and Police Leader East at Krakow his order to complete the combing out of the Warsaw ghetto with the greatest severity and relentless tenacity. I therefore decided to destroy the entire Jewish residential area by setting every block on fire, including the blocks of residential buildings near the armament works. One building after the other was systematically evacuated and subsequently destroyed by fire. The Jews then emerged from their hiding places and dugouts in almost every case. Not infrequently the Jews stayed in the burning buildings until, because of the heat and the fear of being burned alive, they preferred to jump down from the upper floor after having thrown mattresses and other upholstered articles into the street from the burning buildings. With their bones broken they still tried to crawl across the street into blocks of buildings which had not yet been set on fire or were only partially in flames. Often the Jews changed their hiding places during the night by moving into the rooms of burnt-out buildings, taking refuge there until they were found by our patrols. Their stay in the sewers also ceased to be pleasant after the first week. Frequently from the street we could hear loud voices coming through the sewer shafts. Then the men of the Waffen-SS, the Police, or the Wehrmacht Engineers courageously climbed down the shafts to bring out the Jews and not infrequently they then stumbled over Jews already dead or were shot at. It was always necessary to use smoke candles to drive out the Jews Thus one day we opened 183 sewer entrance holes and at a fixed time lowered smoke candles into them, with the result that the bandits fled from what they believed to be gas into the center of the former ghetto, where they could then be pulled out of the sewer holes there. A great number of Jews who could not be counted were exterminated by blowing up sewers and dugouts.
La résistance opposée par les Juifs et les bandits ne put être brisée que par l’emploi sans répit, nuit et jour, de toutes nos troupes de choc. Le 23 avril 1943, le Reichsführer SS donna l’ordre aux chefs SS et au chef de la Police de l’Est à Cracovie, d’accomplir le nettoyage du ghetto de Varsovie avec la plus grande sévérité et la ténacité la plus ferme. C’est pourquoi je décidai de détruire entièrement les maisons juives, en incendiant chaque bloc de maisons, y compris les blocs qui avoisinent les usines d’armement. On évacua systématiquement et on incendia tous les bâtiments, l’un après l’autre. Les Juifs sortirent de leurs abris, il y en avait partout; il n’était pas rare que les Juifs restassent dans les bâtiments en flammes jusqu’à ce que la chaleur et la crainte d’être brûlés vifs les fissent sauter des étages supérieurs après avoir jeté dans la rue les matelas et autres objets rembourrés que pouvaient contenir les bâtiments incendiés. Malgré leurs membres brisés, ils essayaient encore de ramper dans la rue pour atteindre les blocs de maisons qui n’avaient pas encore été atteints par les flammes ou qui n’étaient que partiellement brûlés. Souvent les Juifs changeaient de cachette durant la nuit, en se faufilant à travers les ruines des bâtiments brûlés où ils se cachaient jusqu’à ce qu’ils soient pris par les patrouilles. Ils restaient dans les égouts mais ils trouvèrent cela moins drôle après la première semaine, car fréquemment, de la rue, nous pouvions entendre des voix qui venaient jusqu’à nous par les bouches d’égout. Ainsi, les Waffen SS, les hommes de la police ou ceux du corps du Génie descendaient courageusement par les bouches d’égout pour en faire sortir les Juifs. Assez fréquemment, ils trébuchaient sur des Juifs déjà morts, ou ils fusillaient les vivants sur place. Il fallait toujours se servir de bombes fumigènes pour les faire sortir de leur cachette; ainsi, un jour, nous ouvrîmes 183 bouches d’égout et à une heure fixée nous y lançâmes une bombe fumigène. Il en résulta que les bandits, croyant que c’étaient des gaz, s’enfuirent jusque dans le centre du vieux ghetto où ils purent être expulsés des bouches d’égout. Un grand nombre de Juifs que nous n’avons pu compter furent tués par l’explosion des égouts et des tranchées.
Der von den Juden und Banditen geleistete Widerstand konnte nur durch energischen unermüdlichen Tag- und Nacht-Einsatz der Stoßtrupps gebrochen werden. Am 23. April 1943 erging vom Reichsführer-SS über den höheren SS- und Polizeiführer Ost in Krakau der Befehl, die Durchkämmung des Ghettos in Warschau mit größter Härte und unnachsichtlicher Zähigkeit zu vollziehen. Ich entschloß mich deshalb, nunmehr die totale Vernichtung des jüdischen Wohnbezirks durch Abbrennen sämtlicher Wohnblocks, auch der Wohnblocks bei den Rüstungsbetrieben, vorzunehmen. Es wurde systematisch ein Betrieb nach dem anderen geräumt und anschließend durch Feuer vernichtet. Fast immer kamen dann die Juden aus ihren Verstecken und Bunkern heraus. Es war nicht selten, daß die Juden in den brennenden Häusern sich so lange aufhielten, bis sie es wegen der Hitze und aus Angst vor dem Verbrennungstod vorzogen, aus den Stockwerken herauszuspringen, nachdem sie vorher Matratzen und andere Polstersachen aus den brennenden Häusern auf die Straße geworfen hatten. Mit gebrochenen Knochen versuchten sie dann noch über die Straße in Häuserblocks zu kriechen, die noch nicht oder nur teilweise in Flammen standen. Oft wechselten die Juden auch ihre Verstecke während der Nacht, indem sie sich in bereits abgebrannte Ruinen verzogen und dort so lange Unterschlupf fanden, bis sie von den einzelnen Stoßtrupps aufgefunden wurden. Auch der Aufenthalt in den Kanälen war schon nach den ersten 8 Tagen kein angenehmer mehr. Häufig konnten auf der Straße durch die Schächte laute Stimmen aus den Kanälen heraus gehört werden. Mutig kletterten dann die Männer der Waffen- SS oder der Polizei oder Pioniere der Wehrmacht in die Schächte hinein, um die Juden herauszuholen, und nicht selten stolperten sie dann über bereits verendete Juden oder wurden beschossen. Immer mußten Nebelkerzen in Anwendung gebracht werden, um die Juden herauszutreiben. So wurden an einem Tag 183 Kanaleinsteiglöcher geöffnet und in diese zu einer festgelegten Zeit Nebelkerzen herabgelassen mit dem Erfolg, daß die Banditen vor dem angeblichen Gas flüchtend im Zentrum des ehemaligen jüdischen Wohnbezirks zusammenliefen und aus den dort befindlichen Kanalöffnungen herausgeholt werden konnten. Zahlreiche Juden, die nicht gezählt werden konnten, wurden in Kanälen und Bunkern durch Sprengungen erledigt.
92
Чем дольше длилось сопротивление, тем мужественнее становились люди из Ваффен-СС, полиции и Вермахта. Они без устали выполняли свой долг как верные товарищи, являясь образцовыми солдатами. Зачастую они несли службу с раннего утра до поздней ночи. Ночные патрули с перебинтованными ногами, оставались на посту и не давали евреям передышки. Нередко они ловили и убивали евреев, которые использовали ночное время для пополнения своих запасов и связи с соседними группами или обмена информацией.
The longer the resistance lasted, the tougher the men of the Waffen-SS, Police, and Wehrmacht became. They fulfilled their duty indefatigably in faithful comradeship and stood together as models and examples of soldiers. Their duty hours often lasted from early morning until late at night. At night search patrols, with rags wound around their feet, remained at the heels of the Jews and gave them no respite. Not infrequently they caught and killed Jews who used the night hours for supplementing their stores from abandoned dugouts and for contacting neighboring groups or exchanging news with then.
Plus la résistance se prolongeait, plus les Waffen SS, la Police et la Wehrmacht s’exaspéraient. Ils remplirent leur devoir sans relâche dans une camaraderie fraternelle et furent tous des modèles et des exemples de soldats. Leur activité commençait au petit jour et durait jusqu’à une heure avancée de la nuit. La nuit, des patrouilles, les pieds entourés de chiffons talonnaient les Juifs sans répit. Fréquemment, ils attrapaient et tuaient des Juifs qui profitaient des heures d’obscurité pour se ravitailler en sortant des égouts ou pour aller trouver des groupes voisins et échanger des nouvelles.
Je länger der Widerstand andauerte, desto härter wurden die Männer der Waffen-SS, Polizei und der Wehrmacht, die auch hier in treuer Waffenbrüderschaft unermüdlich an die Erfüllung ihrer Aufgabe herangingen und stets beispielhaft und vorbildlich ihren Mann standen. Der Einsatz ging oft vom frühen Morgen bis in die späten Nachtstunden. Nächtliche Spähtrupps, mit Lappen um die Füße gewickelt, blieben den Juden auf, den Fersen und hielten sie ohne Unterbrechung unter Druck. Nicht selten wurden Juden, welche die Nacht benutzten, um aus verlassenen Bunkern ihre Lebensmittelvorräte zu ergänzen oder mit Nachbargruppen Verbindung aufzunehmen, beziehungsweise Nachrichten auszutauschen, gestellt und erledigt.
93
Учитывая то, что большая часть людей Ваффен-СС прошла подготовку только на протяжении 3–4 недель до начала акции, следует отдать должное смелости, отваге и самопожертвованию которые они продемонстрировали. Следует сказать, что инженеры Вермахта, без устали и с величайшей самоотдачей проводили подрывы укрытий, канализации и бетонных зданий. Офицеры и сотрудники полиции, большая часть которых уже была на фронте имели превосходных боевой дух.
Considering that the greater part of the men of the Waffen-SS had only been trained for 3 to 4 weeks before being assigned to this action, high credit should be given to the pluck, courage, and devotion to duty which they showed. It must be stated that the Wehrmacht Engineers, too, executed the blowing up of dugouts, sewers, and concrete buildings with indefatigability and great devotion to duty. Officers and men of the Police, a large part of whom had already been at the front, again excelled by their dashing spirit.
Si l’on considère que la plus grande partie des hommes de la Waffen SS n’avaient été entraînés que pendant trois ou quatre semaines avant de participer à cette action, il faut reconnaître qu’ils ont témoigné d’un rare cran, d’un courage et d’un goût du risque émérites. On doit également souligner l’infatigable dévouement et le sens du devoir des soldats du Génie, qui firent sauter les égouts, les abris, et les maisons; les officiers et les hommes appartenant à la Police, dont beaucoup avaient été déjà au front, manifestèrent à nouveau un esprit combatif exemplaire.
Wenn man berücksichtigt, daß die Männer der Waffen-SS zum größten Teil vor ihrem Einsatz nur eine drei- bis vierwöchentliche Ausbildung hinter sich hatten, so muß der von ihnen gezeigte Schneid, Mut und die Einsatzfreudigkeit besonders anerkannt werden. Es ist festzustellen, daß auch die Pioniere der Wehrmacht, die von ihnen vorgenommenen Sprengungen von Bunkern, Kanälen und Betonhäusern in unermüdlicher, einsatzfreudiger Arbeit vollbrachten. Offiziere und Män ner der Polizei, die zu einem großen Teil bereits Fronterfahrung hatten, bewährten sich erneut durch beispielhaftes Draufgängertum.
94
Только с помощью продолжительной и безустанной работы всех кто участвовал, мы смогли установить 56065 ликвидированных евреев. К этому следует добавить ряд евреев которых убили взрывы и пожары, количество которых установить невозможно».
Only through the continuous and untiring work of all involved did we succeed in catching a total of 56,065 Jews whose extermination can be proved. To this should be added the number of Jews who lost their lives In explosions or fires but whose number could not be ascertained.
C’est seulement par un travail ininterrompu et infatigable de forces combinées que nous avons réussi à attraper 56.065 Juifs dont nous pouvons prouver l’extermination; à ce nombre il y a lieu d’ajouter les Juifs qui perdirent la vie dans les explosions ou les incendies et dont le nombre n’a pu être évalué.
Nur durch den ununterbrochenen und unermüdlichen Einsatz sämtlicher Kräfte ist es gelungen, insgesamt 56065 Juden zu erfassen und nachweislich zu vernichten. Dieser Zahl hinzuzusetzen sind noch die Juden, die durch Sprengungen, Brände usw. ums Leben gekommen sind, aber nicht zahlenmäßig erfaßt werden konnten.«
4795
ПредседательМайор Уолш, в разделе с которым вы сейчас работаете, вам не следует прочитать вступительные абзацы этого документа, которые излагают потери германских войск?
THE PRESIDENT Major Walsh, in the section that you are just upon now, ought you not to read the opening paragraphs of this document, which set out the amount of the losses of the German troops?
LE PRÉSIDENT Commandant Walsh, dans la partie dont vous vous occupez en ce moment, n’y aurait-il pas lieu de lire la préface de ce document qui établit le total des pertes des troupes allemandes?
VORSITZENDER Major Walsh, sollten Sie nicht aus dem Teil, mit dem Sie sich gerade befassen, die einleitenden Absätze dieses Dokuments verlesen, die über die Verluste der deutschen Truppen sprechen?
4896
УолшСделаю сэр. На странице 1 перевода, я цитирую. Заголовок: «Варшавского гетто больше нет».
MAJOR WALSH I will do so, Sir. On Page 1 of the translation, I quote. The title: "The Warsaw Ghetto is no more."
COMMANDANT WALSH Je le ferai, Monsieur le Président; à la page 1 de la traduction, je cite le titre: «Le ghetto de Varsovie n’existe plus.» Suivent quinze noms.
MAJOR WALSH Ich will es gerne tun, Herr Präsident. Ich zitiere von Seite 1 der Übersetzung. Überschrift:»Es gibt keinen jüdischen Wohnbezirk in Warschau mehr.«
97
За фюрера и страну пали в бою с евреями и бандитами в бывшем еврейском жилом районе Варшавы» — приводятся пятнадцать фамилий.
For the Fuehrer and their country the following fell in the battle for the destruction of Jews and bandits in the former Jewish residential area of Warsaw." — Fifteen are thereafter listed.
Pour le Führer et pour leur pays, ceux dont les noms suivent sont tombés dans la bataille pour l’extermination des Juifs et des bandits dans l’ancien ghetto de Varsovie.
Für den Führer und für ihr Vaterland sind im Kampf bei der Vernichtung von Juden und Banditen im ehemaligen jüdischen Wohnbezirk in Warschau gefallen:...» und dann folgen 15 Namen.
98
Кроме того сержант польской полиции Юлиан Зелински, родившийся 13 ноября 1891, 8-й комиссариат, пал 19 апреля 1943 при выполнении долга. Они приложили все силы, отдав свою жизнь. Мы не никогда не забудем.
Furthermore, the Polish Police Sergeant Julian Zielenski, born 13 November 1891, 8th Commissariat, fell on 19 April i943 while fulfilling his duty. They gave their utmost, their life. We shall never forget them.
Tout d’abord, le sergent de police polonais Julian Zelinski, né le 13 novembre 1891, 8e commissariat, tombé le 19 avril 1943 en accomplissant son devoir. Ils donnèrent le maximum: leur vie. Nous ne les oublierons jamais.
Ferner fiel in Ausübung seines Dienstes am 19. April 1943 der polnische Polizei-Wachtmeister Julian Zielinski, geboren am 13. November 1891, 8. Kommissariat. Sie setzten ihr Höchstes, ihr Leben, ein. Wir wer den sie nie vergessen.
99
«Ранены следующие…» Далее следуют фамилии 60 военнослужащих Ваффен-СС, 11 часовых из лагерей подготовки (вероятно литовцы), 12 офицеров полиции безопасности в подразделениях СС, 5 сотрудников польской полиции и 2 солдат среди инженеров Вермахта.
"The following were wounded.... Then follow the names of 60 Waffen-SS personnel, 11 watchmen from training camps (probably Lithuanians), 12 Security Police officers in SS units, 5 men of the Polish Police, and 2 soldiers of the Wehrmacht Engineers.
«Ceux dont les noms suivent ont été blessés.» Viennent alors les noms de 60 Waffen SS, 11 surveillants de camps d’entraînement, probablement Lituaniens, 12 officiers de la Police de sûreté dans les SS, 5 hommes de la police polonaise et 2 membres du Génie de la Wehrmacht.
Es wurden verwundet:...» Dann folgen die Namen von 60 Waffen-SS-Männern, 11 Wachtposten von Schulungslagern, wahrscheinlich Litauer, 12 Sicherheitspolizei-Offiziere in SS-Einheiten, 5 Männer der polnischen Polizei und 2 Soldaten der Wehrmacht-Pioniere.
100
Позвольте зачитать краткие выдержки из ежедневных докладов по телетайпу. Страница 13 перевода, из сообщения телетайпом от 22 апреля 1943, я читаю:
Permit me to read some brief excerpts of the daily teletype reports. Page 13 of the translation, from the teletype message of 22 April 1943, I read:
Permettez-moi de lire quelques extraits de comptes rendus télétypés à la page 13 de la traduction, du message télétypé du 22 avril 1943, je lis:
Ich möchte nunmehr einige kurze Auszüge der täglichen Fernschreibeberichte verlesen. Seite 13 der Übersetzung aus dem Fernschreiben vom 22. April 1943 lautet:
101
Поджог нами квартала дал результат ночью, когда те евреи которых мы не смогли обнаружить, несмотря на все наши поисковые операции, покинули свои укрытия — на крышах, в подвалах и стали выходить из домов, пытаясь как-нибудь спастись от пожаров. Массами, целыми семьями, уже охваченные огнём они выпрыгивали из окон или пытались спустится по простыням. Эти евреи, как и остальные сразу же ликвидировались».
Our setting the block on fire achieved the result in the course of the night that those Jews whom we had not been able to find despite all our search operations left their hideouts under the roofs, in the cellars; and elsewhere and appeared on the outside of the building, trying to escape the flames anyhow. Masses of them-entire families were already aflame and jumped from the windows or endeavored to let themselves down by means of sheets tied together or the like. Steps had been taken so that these Jews as well as the remaining ones were liquidated at once.
Nous incendiâmes tout un bloc de bâtiments ce qui eut pour résultat, pendant la nuit, de faire sortir les Juifs que nous n’avions pu trouver, malgré toutes nos opérations de recherches, de leurs cachettes sous les toits, dans les caves ou ailleurs et nous les vîmes sur les façades des maisons, essayant d’échapper aux flammes; des groupes importants, des familles entières avaient déjà leurs vêtements en flammes et sautaient des fenêtres et essayaient d’atteindre la rue au moyen de draps noués les uns aux autres, mais des mesures avaient été prises pour que ces Juifs, aussi bien que ceux qui restaient dans les bâtiments, fussent aussitôt supprimés.
Die Anlegung des Brandes hatte im Laufe der Nacht das Ergebnis, daß die unter den Dächern, beziehungsweise in den Kellern und sonstigen Schlupfwinkeln sich trotz aller Durchsuchungsaktionen noch verborgenen Juden an den äußeren Fronten des Häuserblocks zeigten, um dem Feuer irgendwie zu entgehen. In Massen – ganze Familien – sprangen die Juden, schon vom Feuer erfaßt, aus dem Fenster oder versuchten, sich durch aneinandergeknüpfte Bettlaken usw. herabzulassen. Es war Vorsorge getroffen, daß diese sowohl als auch die anderen Juden sofort liquidiert wurden.
102
И на странице 28 перевода, последняя часть первого абзаца, я читаю:
And from Page 28 of the translation, the last part of the first paragraph, I read:
À la page 28 de la traduction, dernière partie du premier paragraphe, je cite:
Von Seite 28 der Übersetzung lese ich den letzten Teil des Absatzes:
103
Когда вышеуказанные кварталы были уничтожены, были пойманы 120 евреев и множество евреев было уничтожено, когда они прыгали с чердаков во дворы пытаясь спастись от пожара. Ещё больше евреев было убито в пожарах или были уничтожены при подрыве укрытий или канализации».
When the blocks of buildings mentioned above were destroyed, 120 Jews were caught and numerous Jews were destroyed when they jumped from the attics to the inner courtyards, trying to escape the flames. Many more Jews perished in the flames or were destroyed when the dugouts and sewer entrances were blown up.'
Quand les blocs de bâtiments mentionnés plus haut furent détruits, 120 Juifs furent pris et plusieurs furent tués en sautant des greniers dans les cours des maisons, en essayant d’échapper aux flammes. Un plus grand nombre encore périt dans les flammes ou fut tué par l’explosion des tranchées et des égouts.
Bei der Vernichtung des vorgenannten Häuserblocks wurden 120 Juden erfaßt und ungezählte Juden, die infolge des Brandes aus dem Dachgeschoß auf die inneren Hofe absprangen, vernichtet. Weiter sind viele Juden in den Flammen umgekommen, beziehungsweise wurde eine weitere Anzahl von Juden durch vorgenommene Sprengungen von Bunkern und Kanalöffnungen ebenfalls vernichtet.
104
И на странице 30, вторая половина второго абзаца, я читаю:
And on Page 30, second half of the second paragraph, I read:
À la page 30, seconde moitié du second paragraphe, je lis:
Und ich lese nun von Seite 30 die zweite Hälfte des zweiten Absatzes:
105
Евреи не появлялись пока кварталы не стали гореть сильно, а здания начали рушиться. Снова и снова евреи пытались спастись даже с помощью горящих зданий. Бесчисленные евреи которых мы видели на крышах во время поджога исчезали в огне. Другие появлялись на верхних этажах в последний момент и могли избежать смерти только прыгнув вниз. Сегодня мы поймали 2283 еврея из которых расстреляно 204; бесчисленное количество евреев уничтожено в укрытиях и пожарах».
Not unto the blocks of buildings were well aflame and were about to collapse did a considerable number of Jews emerge, forced to do so by the flames and the smoke. Time and again the Jews tried to escape even through burning buildings. Innumerable Jews whom we saw on the roofs during the conflagration perished in the flames. Others emerged from the upper stories in the last possible moment and were only able to escape death from the flames by jumping down Today we caught a total of 2,283 Jews of whom 204 were shot; and innumerable Jews were destroyed in dugouts and in the flames.
Ce n’est que lorsque les blocs de bâtiments furent complètement en flammes et près de s’effondrer, qu’un nombre plus considérable encore de Juifs sortit pour éviter les flammes et la fumée. À plusieurs reprises, ils essayèrent d’échapper des bâtiments en flammes. D’innombrables Juifs que nous avons vus sur les toits pendant l’incendie y périrent. D’autres sortirent des étages supérieurs au dernier moment et purent échapper, mais en sautant dans la rue. Aujourd’hui, nous avons attrapé en tout 2283 Juifs dont 204 furent fusillés et d’innombrables autres furent exterminés par le feu dans les tranchées.
Erst nachdem die Häuserblocks durch Feuer der Vernichtung entgegengingen, kamen eine erhebliche Zahl von Juden, durch das Feuer und den Rauch gezwungen, zum Vorschein. Immer wieder versuchten die Juden, selbst durch brennende Gebäude hindurchzuwechseln. Ungezählte Juden, die sich während der Feuersbrunst auf den Dächern zeigten, sind in den Flammen umgekommen. Andere kamen erst im letzten Augenblick in den höchsten Stockwerken zum Vorschein und konnten sich nur durch Abspringen vor dem Verbrennungstod retten. Es wurden am heutigen Tage insgesamt 2283 Juden erfaßt, davon 204 erschossen, ungezählte Juden wurden in Bunkern und durch Feuer vernichtet.
106
И на странице 34, второй абзац, я читаю, начиная с второй строки:
And from Page 34, the second paragraph, I read, beginning the second line:
À la page 34, second paragraphe, je lis en commençant par la seconde ligne:
Und nun lese ich von Seite 34, Absatz 2, beginnend mit der zweiten Zeile:
107
Евреи говорят о том, что они появляются снаружи, чтобы подышать свежим воздухом, так как невыносимо постоянно находится в укрытиях из-за длительности операции. В среднем поисковые отряды расстреливают от 30 до 50 евреев каждую ночь. Из этих заявлений, можно сделать вывод, что евреи находятся под землей в гетто. Сегодня мы подорвали бетонное здание, которое не смог уничтожить пожар. В ходе этой операции мы поняли, что подрыв здания это очень долгий процесс, требующий огромное количество взрывчатки. Таким образом лучшим и единственным методом уничтожения евреев является поджог».
The Jews testify that they emerge at night to get fresh air, since it is unbearable to stay permanently within the dugouts owing to the long duration of the operation. On the average the raiding parties shoot 30 to 50 Jews each night. From these statements it was to be inferred that a considerable number of Jews are still underground in the ghetto. Today we blew up a concrete building which we had not been able to destroy by fire. In this operation we learned that the blowing up of a building is a very lengthy process and takes an enormous amount of explosives. The best and only method for destroying the Jews therefore still remains the setting of fires.''
Les Juifs certifient qu’ils sortent la nuit de leurs abris pour avoir un peu d’air frais, car le séjour permanent dans les abris finit par devenir insupportable. En moyenne, les patrouilles abattent de 30 à 40 Juifs chaque nuit; de cette constatation, il ressort qu’un nombre considérable de Juifs séjourne encore dans le sous-sol du ghetto. Aujourd’hui, nous avons fait sauter un bâtiment en béton que nous n’avions pu détruire par le feu et par cette opération, nous avons établi que c’est une opération très longue que de faire sauter un bâtiment à la dynamite et qu’elle demande une quantité énorme d’explosifs. C’est pourquoi la meilleure et la seule méthode pour exterminer les Juifs est encore l’emploi de produits incendiaires.
Die Juden sagen aus, daß sie nachts an die frische Luft kommen, da ein ununterbrochener Aufenthalt in ihren Bunkern für sie durch die längere Dauer der Aktion unerträglich wird. Durchschnittlich werden durch die Stoßtrupps in jeder Nacht 30-50 Juden erschossen. Nach diesen Aussagen muß an genommen werden, daß immer noch eine größere Zahl von Juden sich unterirdisch im Ghetto aufhält. Heute wurde ein Betongebäude, das durch Feuer nicht zu vernichten war, gesprengt. Hierbei wurde festgestellt, daß das Sprengen von Häusern ungemein langwierig ist und eine Riesenmenge an Munition erfordert. Die einzige und beste Methode zur Vernichtung der Juden ist daher immer noch die Anlegung von Bränden.
108
И на странице 35, последняя часть второго абзаца, я читаю:
And from Page 35, the last part of the second paragraph, I read:
À la page 35, dernière partie du second paragraphe, je lis:
Ich lese jetzt den letzten Teil des zweiten Absatzes auf Seite 35:
109
Отдельные показания говорят о трёх — четырёх тысячах евреев остающихся в подземных норах, канализации, и укрытиях. Нижеподписавшийся решился не завершать крупномасштабную операцию пока не будет уничтожен последний еврей».
Some depositions speak of three to four thousand Jews still remaining in underground holes, sewers, and dugouts. The undersigned is resolved not to terminate the large-scale operation until the last Jew has been destroyed.
Quelques dépositions indiquent que 3 ou 4.000 Juifs restent encore terrés dans des trous, des abris, des égouts; le soussigné est résolu à ne pas abandonner une opération faite sur une aussi grande échelle avant que les derniers Juifs aient été anéantis.
Nach gemachten Aussagen sollen sich noch etwa 3-4000 Juden in den unterirdischen Löchern, Kanälen und Bunkern aufhalten. Der Unterzeichnete ist entschlossen, die Großaktion nicht eher zu beenden, bis auch der letzte Jude vernichtet ist.
110
И в сообщении по телетайпу от 15 мая 1943 на странице 44, мы видим, что операция находится на завершающем этапе. Я читаю конец первого абзаца на странице 44:
And from the teletype message of 15 May 1943 on Page 44, we gather that the operation is in its last stage. I read the end of the first paragraph on Page 44:
Et du message télétypé du 15 mai 1943, page 44, nous constatons que l’opération est à sa dernière phase. Je lis la fin du premier paragraphe, page 44:
Aus dem Fernschreiben vom 15. Mai 1943 auf Seite 44 müssen wir entnehmen, daß das Unternehmen seinem Ende entgegengeht. Ich verlese den Schluß des ersten Absatzes auf Seite 44:
111
Специальное подразделение снова обыскало последний квартал, который оставался нетронутым и уничтожило его. Вечером, часовня, морг и все другие здания еврейского кладбища были уничтожены огнём».
A special unit once more searched the last block of buildings, which was still intact, in the ghetto and subsequently destroyed it. In the evening the chapel, mortuary, and all other buildings in the Jewish cemetery there blown up or destroyed by fire.
Des unités spéciales ont fouillé une fois de plus le dernier bloc de bâtiment qui était encore intact dans le ghetto et l’ont ensuite détruit. Le soir, la chapelle, le dépôt mortuaire et tous les autres édifices du cimetière juif ont été dynamités ou détruits par le feu.
Durch ein Sonderkommando wurde der letzte noch vorhandene unversehrte Gebäudekomplex des Ghettos nochmals durchsucht und anschließend vernichtet. Am Abend wurden auf dem jüdischen Friedhof die Kapelle, Leichenhalle und sämtliche Nebengebäude gesprengt, beziehungsweise durch Feuer vernichtet.
112
В донесении от 24 мая 1943 г. генерал-майор Штрооп подвел последние итоги.
On 24 May 1943 the final figures have been compiled by Major General Stroop. He reports on Page 45, last paragraph:
Le 24 mai 1943, le major général Stroop fait le bilan (page 45, dernier paragraphe):
Am 24. Mai 1943 hat Generalmajor Stroop die Bilanz gezogen. Er berichtet auf Seite 45, letzter Absatz:
113
Из общего числа 56 065 человек пойманных евреев примерно 7 тысяч человек были убиты на территории бывшего гетто во время операции широких масштабов, 6929 евреев были уничтожены после доставки их в Т-11, что как кажется Треблинка, лагерь номер 2 — к этому относится «общее число уничтоженных евреев составляет 13 929 человек. Помимо названных 56 065 человек, примерно 5 или 6 тысяч евреев были уничтожены во время взрывов или же погибли в пламени».
Of the total of 56,065 caught, about 7,000 were destroyed in the former Jewish residential area during large-scale operations; 6,929 Jews were destroyed by transporting them to T. II"-which we believe to be Treblinka, Camp Number 2, which will later be referred "the sum total of Jews destroyed is therefore 13,929. Beyond the number of 56,065 an estimated number of 5,000 to 6,000 Jews were destroyed by being blown up or by perishing in the flames.
Sur un total de 56.065 Juifs pris, environ 7.000 furent tués dans l’ancien ghetto durant l’opération de grande envergure, 6.929 Juifs furent mis à mort quand ils furent transportés au T. II – qui est je crois le camp Treblinka nº 2 –, qui sera mentionné plus tard. Le total des Juifs tués est de 13.929. Dans ce chiffre de 56.065, un nombre approximatif de 5 à 6.000 moururent dans l’explosion des bâtiments ou périrent dans les incendies.
Von den 56065 insgesamt erfaßten Juden sind ca. 7000 im Zuge der Großaktion im ehemaligen jüdischen Wohnbezirk selbst vernichtet. Durch Transport nach T. II«– wir glauben, das ist Treblinka, Lager Nr. 2, auf das wir später zurückkommen werden, –»wurden 6929 Juden vernichtet, so daß insgesamt 13929 Juden vernichtet wurden. Über die Zahl 65065 hinaus sind schätzungsweise 5-6000 Juden bei Sprengungen und durch Feuer vernichtet worden.
114
Суд заметил в докладе 1061-ПС ряд фотографий; и с разрешения суда я хочу показать некоторые из этих фотографий, снимков на экране, если суд не считает, что ссылки на оригинальный текст не достаточно для суда.
The Court has noted within the report 1061-PS a number of photographs; and with the Court's permission I should like to show a few of these photographs, still pictures, on the screen, unless the Court believes that reference to the original text will be sufficient for the Court's purpose.
Le Tribunal a remarqué dans le document PS-1061 un certain nombre de photographies et avec sa permission j’aimerais en montrer quelques-unes sur l’écran à moins que le Tribunal ne décide qu’il suffise de se référer au texte original.
Der Gerichtshof hat in dem Bericht 1061-PS eine Anzahl von Photographien bemerkt, und ich möchte einige dieser Lichtbilder auf der Leinwand zeigen, falls ein Hinweis auf den Originaltext dem Gerichtshof nicht ausreichend erscheint.
49115
ПредседательНет; если вы хотите показать их на экране, можете это сделать. Вероятно, будет удобно сейчас прерваться и вы сможете показать их на экране.
THE PRESIDENT No; if you want to put them on the screen, you may do so. Perhaps it would be convenient to adjourn now and you can put them on the screen afterwards.
LE PRÉSIDENT Non, si vous désirez faire passer ces photographies devant le Tribunal, vous le pouvez. Peut-être conviendrait-il de suspendre maintenant et vous montrerez ces photographies sur l’écran à la reprise de l’audience?
VORSITZENDER Nein. Wenn Sie diese Bilder auf der Leinwand zeigen wollen, sollten Sie das tun. Vielleicht können wir eine Pause einschalten und die Bilder nachher sehen.
Объявлен перерыв
A Recess was taken.
L’audience est suspendue.
Pause von 10 Minuten.
[Фотографии проектируются на экран в зале суда
Still pictures were projected on the screen in the courtroom.7
Présentation des photographies à l’écran.
Lichtbilder werden im Gerichtssaal auf der Leinwand gezeigt. Bild auf der Leinwand.
50116
УолшПервая фотография [указывая на фотографию] показанная на странице 27 из фотографий документа 1061-ПС. Она называется «Уничтожение квартала». Суд вспомнит, те фрагменты сообщений по телетайпу который ссылались на поджоги для того, чтобы выгнать евреев. Эта фотография, изображает эту сцену.
MAJOR WALSH This first picture [pointing to a picture on the screen] is shown on Page 27 of the photographs in Document 1061-PS. It is entitled "The Destruction of a Block of Buildings" The Court will recall those portions of the teletype messages that referred to the setting of fires for the purpose of driving out the Jews. This picture, taken from the record, portrays such a scene.
COMMANDANT WALSH 1. Cette première photographie figure à la page 27 des photographies comme pièce justificative du document PS-1061; son titre est: «Destruction d’un bloc de bâtiments». Le Tribunal se rappellera le passage des messages télétypés qui se réfère au fait de déchaîner un incendie pour forcer les Juifs à sortir de leur abri.
MAJOR WALSH Das erste Bild ist auf Seite 27 der Photographien in Dokument 1061-PS zu finden. Es hat den Titel»Vernichtung eines Häuserblocks«. Der Gerichtshof wird sich der Teile der Fernschreibenachrichten erinnern, die sich auf die Brandlegung bezogen, um die Juden aus den Häusern zu treiben. Dieses Bild, das der Urkunde entnommen ist, zeigt solch eine Szene.[Bild auf der Leinwand.]
117
Эта фотография [показывая на фотографию] со страницы 21 фотографий в экземпляре, и подпись «Выкуривание евреев и бандитов». Фрагменты из сообщений по телетайпу относятся к использованию дыма, чтобы вынудить евреев покинуть свои укрытия.
This picture [pointing to a picture on the screen] is from Page 21 of the photographs contained in the exhibit, and the caption is "Smoking out of the Jews and Bandits." Excerpts from the teletype messages read in the record relate to the use of smoke as a means of forcing Jews out of the hiding places.
2. Page 21 des photographies contenues dans le document; le sous-titre est: «Asphyxie de Juifs et de bandits». Les extraits des messages télétypés lus au procès-verbal parlent de l’usage des bombes fumigènes comme moyen de forcer les Juifs à sortir de leurs cachettes.
Dieses Bild mit der Überschrift:»Ausräucherung der Juden und Banditen«ist von Seite 21 der im Beweisstück enthaltenen Bilder. Auszüge aus den Fernschreibenachrichten, die in das Protokoll verlesen wurden, sprechen von der Rauchanwendung als einem Mittel, die Juden aus ihren Verstecken herauszutreiben. Bild auf der Leinwand.]
118
Эта фотография [показывая на фотографию] со страницы 36, фотография в экземпляре и она называется «Сражение в логове сопротивления». Очевидно это снимок подрыва, использованного для уничтожения одного из домов и суд может вспомнить сообщение от 7 мая 1943, которое относится к подрыву зданий как долгому процессу требующему огромного количества взрывчатки. Это же сообщение говорит о том, что лучшим методом уничтожения евреев является поджог.
This picture [pointing to a picture on the screen] is from Page 36 of the photographs in the exhibit and it is called "Fighting a Nest of Resistance." It is obviously a picture of an explosive blast being used to destroy one of the buildings, and the Court may recall the message of 7 May 1943 that related to the blowing up of buildings as a lengthy process requiring an enormous amount of explosive. The same message reported that the best method for destroying the Jews was the setting of fires.
3. Page 36 des photographies, le sous-titre est: «Lutte contre un centre de résistants». Il s’agit évidemment d’une explosion faite pour démolir un bâtiment; je rappelle le message du 7 mai 1943 qui disait que l’explosion des bâtiments prenait beaucoup de temps et exigeait beaucoup d’explosifs. Le même message indiquait que la meilleure méthode pour exterminer les Juifs était l’incendie.
Dieses Bild,»Bekämpfung eines Widerstandsnestes«, ist von Seite 36 der Photographien des Beweisstücks. Es zeigt eine Explosion, die offenbar bei der Zerstörung eines dieser Gebäude stattfand. Der Gerichtshof wird sich erinnern, daß die Fernschreibe nachricht vom 7. Mai 1943 besagte, daß die Sprengung von Häusern langwierig ist und eine Riesenmenge Sprengstoff erfordert. Derselbe Bericht besagte, daß die beste Methode zur Vernichtung der Juden die Anlegung von Bränden ist.[Bild auf der Leinwand.]
119
Эта фотография [показывая на фотографию] со страницы 36. Внимание суда обращается на фигуру человека в воздухе, которая на фотографии между центром и верхним правым углом. Он выпрыгнул с верхнего этажа горящего здания. Подробное изучение этой фотографии судом на оригинале фотографии раскроет другие фигуры в окнах верхнего этажа, которые видимо последовали за ним. Сообщение по телетайпу от 22 апреля сообщало о целых семьях выпрыгивающих из горящих зданий и сразу же ликвидируемых.
This picture [pointing to a picture on the screen] is taken from Page 36 of the photographs. The Court's attention is invited to the figure of a man in mid-air who appears in the picture about halfway between the center and the upper right-hand corner. He has jumped from one of the upper floors of the burning building. A close examination of this picture by the Court in the original photograph will disclose other figures, in the upper floor windows, who apparently are about to follow him. The teletype message of 22 April reported that entire families jumped from burning buildings and were liquidated at once.
4. Cette vue est à la page 36 des photographies. Le Tribunal peut observer à droite, à la partie supérieure de l’écran un homme qui paraît sauter d’une fenêtre d’un étage supérieur du bâtiment en flammes; un examen attentif de la photographie originale révélera d’autres personnes aux fenêtres des étages supérieurs qui apparemment s’apprêtent à le suivre. Le message télétypé du 22 avril rapporte que des familles entières sautèrent des fenêtres de bâtiments en flammes et furent liquidées aussitôt.
Dieses Bild stammt von Seite 36 der Photographien. Ich möchte den Gerichtshof besonders auf einen Mann in der Luft aufmerksam machen, ungefähr in der Mitte zwischen dem Zentrum und der oberen rechten Ecke des Bildes. Er ist von einem der oberen Stockwerke des brennenden Hauses heruntergesprungen. Wenn der Gerichtshof eine nähere Untersuchung der Original-Photographie vornehmen will, wird er noch andere Personen in den Fenstern der oberen Stockwerke erkennen, die anscheinend im Begriff sind, ihm zu folgen. Die Fernschreibenachricht vom 22. April betont, daß ganze Familien aus den brennenden Häusern heraussprangen und sofort vernichtet wurden.[Bild auf der Leinwand.]
120
Эта фотография [показывая на экран] со страницы 39. «Руководитель крупномасштабной акции». Нацистом назначенным руководить этой акцией был генерал-майор Штрооп, который вероятно является центральной фигурой на этой фотографии. Я не могу воздержаться от того, что не прокомментировать улыбающиеся лица в эпицентре насилия и разрушений.
This picture [pointing to a picture on the screen] is from Page 39 of the photographs. It is entitled "The Leader of the Large-scale Action." The Nazi-appointed commander of this action was SS Major General Stroop, who probably is the central figure in this picture. I cannot refrain from commenting at this point on the smiting faces of the group shown there, in the midst of the violence and destruction.
5. Cette photographie se trouve à la page 59 des photographies; son sous-titre est: «Chef d’une opération de grande envergure» et c’est probablement le général SS Stroop qui est au centre de la photographie chef nazi de l’opération. Je ne peux m’empêcher de remarquer que les Allemands sourient dans ce groupe, devant cette destruction et cette violence.
Dieses Bild»Der Führer der Groß-Aktion«ist von Seite 39 der Photographien. Der von den Nazis eingesetzte Führer dieser Aktion war SS-Generalmajor Stroop, der wahrscheinlich im Mittelpunkt dieses Bildes steht. Ich kann nicht umhin, an dieser Stelle auf die lächelnden Gesichter dieser Gruppe inmitten der Gewalttätigkeit und Vernichtung hinzuweisen.
51121
Председатель Вы прошли этот документ?
THE PRESIDENT Are you passing from that document now?
LE PRÉSIDENT Allez-vous laisser de côté ce document maintenant?
VORSITZENDER Verlassen Sie jetzt dieses Dokument?
52122
Уолш Да, сэр.
MAJOR WALSH Yes, Sir.
COMMANDANT WALSH Oui, Monsieur le Président.
MAJOR WALSH Ja.
53123
ПредседательВы скажете трибуналу где был обнаружен документ?
THE PRESIDENT you tell the Tribunal where the document was found?
LE PRÉSIDENT Voulez-vous dire au Tribunal où ce document a été trouvé?
VORSITZENDER Wollen Sie dem Gerichtshof bitte mitteilen, wo dieses Dokument gefunden wurde?
54124
Уолш Это захваченный документ, сэр. Я не знаю его историю, но я с радостью представлю суду его происхождение и историю в начале вечернего заседания.
MAJ0R WALSH It is a captured document, Sir. I do not have the history, but I shall be very pleased to submit the background and history to the Court at the beginning of the afternoon session.
COMMANDANT WALSH C’est un document saisi, Monsieur le Président; je n’en connais pas l’histoire et je serais heureux de soumettre au Tribunal les circonstances de sa découverte au début de l’audience de cet après-midi.
MAJOR WALSH Das Dokument wurde erbeutet. Seine Geschichte weiß ich nicht, aber ich werde seine Geschichte und Herkunft dem Gerichtshof gern zu Beginn der Nachmittagssitzung vortragen.
55125
Председатель Я думаю, трибунал захочет узнать где его обнаружили и кому он был направлен.
THE PRESENT The Tribunal, I think, would like to know where it was found and to whom it was submitted.
LE PRÉSIDENT Le Tribunal aimerait savoir où il fut trouvé et à qui il fut remis.
VORSITZENDER Der Gerichtshof möchte gern wissen, wo es erbeutet wurde, und an wen es gerichtet war.
56126
УолшУ меня это есть. Мне кажется это есть в документе. Сэр, сообщения по телетайпу которые содержатся в этом экземпляре были адресованы высшему руководителю СС и полиции, обергруппенфюреру и генералу полиции Крюгеру или его заместителю. Для уничтожения евреев не всегда было необходимо и возможно, начинать со стадии помещения их в гетто. В прибалтийских государствах это осуществлялось более простым путем. Об этом свидетельствует отчет бригадефюрера СС Шталекера от 15 октября 1941 г., адресованный Гиммлеру и обнаруженный нами в личном архиве Гиммлера. Документ Л-180, США-276. Он озаглавлен: "Эйнзатцгруппа «А»". В отчете докладывалось, что 135 567 человек, почти все евреи, были убиты в соответствии с приказами о полном уничтожении евреев. Этотобъёмный документ дает нам возможность ознакомиться со следующими заявлениями того же бригадефюрера СС;
MAJOR WALSH I have that. I believe that is contained in the document. The teletype messages, Sir, that are contained in this exhibit were all addressed to the Higher SS and Police Fuehrer, SS Obergruppenfuehrer and General of the Police Kruger or his deputy. It was not always necessary, or perhaps desirable, first to place me Jews within the ghettos to eject the elimination. In the Baltic States a more direct course of action was followed. I refer to Document L-180, now in evidence, which is Exhibit USA-276. This is a report by SS Brigadefuehrer Stahlecker to Himmler, dated 15 October 1941, enabled "Action Group A," found in Himmler's private files. He reported that 135,567 persons, nearly all Jews, were murdered in accordance with basic orders directing the complete annihilation of the Jews. This voluminous document provides me with the following statement by the same SS Brigade Fuehrer, and from the translation at the bottom of Page 6, the second sentence of the last paragraph, I read:
COMMANDANT WALSH Cette indication est je crois dans le document: les messages télétypés qui se trouvent dans cette pièce justificative sont tous adressés au SS Obergruppenführer et général de la Police, Krüger ou à son adjoint. Il n’était pas toujours nécessaire ou même désirable de mettre d’abord les Juifs dans les ghettos pour les exterminer. Dans les États Baltes, une manière d’agir plus rapide fut employée et je me réfère au document L-180 qui devint USA-276; c’est un rapport du SS Brigadeführer Stahlecker adressé à Himmler, daté du 15 octobre 1941 et intitulé «Groupe d’action A», il a été trouvé dans les archives privées de Himmler; il y est dit que 135.567 personnes, presque toutes Juives, furent massacrées en exécution d’ordres fondamentaux concernant l’extermination des Juifs. Ce volumineux document me fournit la déclaration suivante du SS Brigadeführer et je cite la traduction de la page 6, 2e phrase, dernier paragraphe:
MAJOR WALSH Es fällt mir jetzt ein. Ich glaube, daß es sich aus dem Dokument selbst ergibt. Die Fernschreibenachrichten, die in diesem Beweisstück enthalten sind, waren alle an den höheren SS- und Polizeiführer, SS-Obergruppenführer und Polizeigeneral Krüger oder seinen Vertreter gerichtet. Es war vielleicht nicht immer notwendig oder wünschenswert, die Juden erst in Ghettos zu bringen, um sie zu vernichten. In den baltischen Staaten wurde ein kürzerer Weg eingeschlagen. Ich weise auf Dokument L-180 hin und lege es jetzt als Beweisstück US-276 vor. Es ist ein Bericht des SS-Brigadeführers Stahlecker an Himmler vom 15. Oktober 1941, mit der Überschrift:»Einsatzgruppe A.«Das Dokument wurde in Himmlers Privatarchiv gefunden. Er meldete, daß 135567 Personen, beinahe alles Juden, im Einklang mit den grundlegenden Befehlen, welche die vollständige Vernichtung der Juden anordneten, ermordet wurden. Dieses umfangreiche Dokument gibt mir Material für die folgende Erklärung desselben SS-Brigadeführers. Ich lese unten von Seite 6 der Übersetzung den zweiten Satz des letzten Absatzes:
127
К нашему удивлению было не просто создать широкий погром против евреев. Климатис, руководитель вышеуказанного партизанского подразделения, которого первоначально использовали для этой задачи, добился начала погрома по рекомендациинебольшого передового подразделения действующего в Ковно и таким образом ни одного германского приказа или германской инициативы не было видно извне. В течение первого погрома в ночь с 25 по 26 июня литовские партизаны устранили 1500 евреев, подожгли несколько синагог или уничтожили их иными средствами и подожгли еврейский район включающий около 60 домов. В течение ночи около 2300 евреев были обезврежены похожим образом».
To our surprise it was not easy, at first, to set in motion an extensive pogrom against the Jews. Klimatis, the leader of the partisan unit. mentioned above, who was used for this purpose primarily, succeeded in starting a pogrom on the basis of advice given to him by a small advanced detachment acting in Kovno and in such a way that no German order or German instigation was noticed from the outside. During the first pogrom in the night from 25 to 26 June the Lithuanian partisans did away with more than 1,500 Jews, setting fire to several synagogues or destroying them by other means and burning down a Jewish dwelling district consisting of about 60 houses. During the following nights 2,300 Jews were eliminated in a similar way.
«À notre étonnement, il était difficile au début de déclencher un vaste pogrom contre les Juifs; Klimatis, chef susnommé de la bande de partisans qu’on a utilisée surtout pour cette besogne, réussit à commencer un pogrom en se basant sur les conseils d’un petit détachement d’avant-garde qu’on lui avait adjoint à Kovno, de telle façon qu’aucun ordre ou aucune incitation n’était reconnaissable comme venant des Allemands. Au cours du premier pogrom, dans la nuit du 25 au 26 juin, les partisans lituaniens exterminèrent plus de 1.500 Juifs; ils incendièrent plusieurs synagogues ou les détruisirent par d’autres moyens et ils mirent le feu à une agglomération d’environ soixante habitations juives. Au cours des nuits suivantes, environ 2.300 Juifs furent mis hors d’état de nuire, d’une façon analogue.»
Es war überraschenderweise zunächst nicht einfach, dort ein Judenpogrom größeren Ausmaßes in Gang zu setzen. Dem Führer der oben bereits erwähnten Partisanengruppe, Klimatis, der hierbei in erster Linie herangezogen wurde, gelang es, auf Grund der ihm von dem in Kauen eingesetzten kleinen Vorkommando gegebenen Hinweise ein Pogrom einzuleiten, ohne daß nach außen irgendein deutscher Auftrag oder eine deutsche Anregung erkennbar wurde. Im Verlaufe des ersten Pogroms in der Nacht vom 25. zum 26. Juni wurden über 1500 Juden von den litauischen Partisanen beseitigt, mehrere Synagogen angezündet oder anderweitig zerstört und ein jüdisches Wohnviertel mit rund 60 Häusern niedergebrannt. In den folgenden Nächten wurden in derselben Weise 2300 Juden unschädlich gemacht.
128
Из последней части параграфа 3, страница 7, я читаю:
From the last part of Paragraph 3, Page 7, I read:
Dans la dernière partie du paragraphe 3 de la page 7, je cite:
Vom letzten Teil des Absatzes 3, Seite 7, lese ich:
129
При этом, было возможно, путём похожего влияния, латвийской вспомогательной полиции начать погром против евреев в Риге. В течение этого погрома все синагоги были разрушены и около 400 евреев убито».
It was possible, though, through similar influences on the Latvian auxiliary to set in motion a pogrom against the Jews also in Riga. During this pogrom all synagogues were destroyed and about 400 Jews were killed.
II a été possible, en usant d’influences analogues sur les auxiliaires lettons, de déclencher un pogrom aussi à Riga. Au cours de ce pogrom, les synagogues furent toutes détruites et environ 400 Juifs tués.
Es gelang zwar, auch in Riga durch entsprechende Einwirkung auf die lettische Hilfspolizei ein Judenpogrom in Gang zu setzen, in dessen Verlauf sämtliche Synagogen zerstört und etwa 400 Juden getötet wurden.
130
В качестве средства массового уничтожения евреев были использованы газовые автомашины. Описание этих машин ужаса и смерти и их действия подробно изложено в совершенно секретном отчете от 16 мая 1942 г., который был адресован оберштурмбаннфюреру СС Рауфу в Берлин, Принцальбрехтштрассе, 8, доктором Бекером, унтерштурмфюрером СС. Документ ПС-501, США-288.
Nazi ingenuity reached a new high mark with the construction and operation of the gas van as a means of mass annihilation of the Jews. A description of these vehicles of horror and death and the operation of them is fully set forth in a captured top-secret document, dated 16 May 1942, addressed to SS Obersturmbannfuehrer Rauff, 8 Prinz-Albrecht-Strasse, Berlin, from Dr. Becker, SS Untersturmfuehrer. I offer this document, 501-PS;, Exhibit USA-288. I quote:
L’ingéniosité nazie atteignit un nouvel étiage avec la construction des camions à gaz comme mode d’extermination en masse des Juifs. La description de ces véhicules de terreur et de mort et leur maniement sont exposés en détail dans un document très secret daté du 16 mai 1942. Ce document a été envoyé au SS Obersturmbannführer Rauff, 8 Prinz Albrechtstrasse à Berlin, par le Dr Becker, SS Untersturmführer.Je présente ce document PS-501 qui devient USA-288. Je cite:
Die Erfindungsgabe der Nazis erreichte einen neuen Höhepunkt mit der Konstruktion und dem Betrieb von Gaswagen als Mittel für die Massenvernichtung von Juden. Eine vollständige Beschreibung dieser Schreckens- und Todeswagen und ihrer Verwendung finden wir in einem erbeuteten geheimen Dokument vom 16. Mai 1942, das an SS-Obersturmbannführer Rauff, Berlin, Prinz-Albrecht-Straße 8, von SS-Untersturmführer Dr. Becker gerichtet war. Ich unterbreite dieses Dokument 501-PS als Beweisstück US-288. Ich zitiere:
131
Осмотр газовых автомобилей группы «D» и «С» окончен. В то время как газовые автомобили первой серии могут использоваться и в плохую погоду, автомобили второй серии «Заурер» совершенно выходят из строя в дождливую погоду. Например, если они пробыли под дождем в течение получаса, то их нельзя использовать потому, что они буксуют. Эти автомобили можно использовать только при абсолютно сухой погоде. Вопрос заключается теперь в том, можно ли использовать газовый автомобиль тогда, когда он стоит на месте казни. Во-первых, его следует доставить к этому месту, что возможно только при хорошей погоде. Во-вторых, место казни обычно находится в 10–15 километрах от шоссе и к нему трудно добираться. В грязь или в сырую погоду к нему вообще нельзя проехать. Если лиц, подлежащих уничтожению, перевозят или ведут к этому месту, то они быстро начинают соображать, что происходит, и начинают беспокоиться, чего следует избегать, насколько это возможно. Остается один только способ — грузить их на автомашины в одном пункте, а затем перевозить к месту казни.
The overhauling of vans by groups D and C is finished. While the vans in the first series can also be put into action if the weather is not too bad, the vans of the second series (Saurer) stop completely in rainy weather. If it has rained for instance for only one-half hour, the van cannot be used because it simply skids away. It can only be used in absolutely dry weather. It is a question now of whether the van can be used only when it stands at the place of execution. First the van has to be brought to that place, which is possible only in good weather. The place of execution is usually 10 to 15 kilometers away from the highway and is difficult of access because of its location; in damp or wet weather it is not accessible at all If the persons to be executed are driven or led to that place, then they realize immediately what is going on and get restless, which is to be avoided as far as possible. There is only one way left: to load them at the collecting point and to drive them to the spot.
La révision des camions par les groupes D et C est terminée. Les camions de la première série peuvent être utilisés si le temps n’est pas trop mauvais. Les camions de la deuxième série s’arrêtent complètement par temps de pluie. Si par exemple il a plu pendant une demi-heure seulement, le camion ne peut être utilisé car il dérape. On ne peut l’utiliser que par temps absolument sec. Il est seulement question maintenant de savoir si le camion peut être utilisé seulement sur place à l’endroit de l’exécution. D’abord le camion doit arriver à cet endroit et ceci ne peut avoir lieu que par beau temps. Le lieu d’exécution se trouve d’habitude à 10 ou 15 kilomètres des grandes routes et ne peut être atteint facilement à cause de son emplacement. Par temps humide ou mouillé on ne peut pas y parvenir du tout. Si les personnes qui doivent être exécutées sont conduites ou amenées à cet endroit, elles se rendent compte tout de suite de ce qui se passe et s’inquiètent et nous devons éviter ceci autant que possible. Il n’y a qu’un seul moyen: les conduire à un point de rassemblement et ensuite, les amener à l’endroit voulu.
Die Überholung der Wagen bei der Gruppe D und C ist beendet. Während die Wagen der ersten Serie auch bei nicht allzu schlechter Wetterlage eingesetzt werden können, liegen die Wagen der zweiten Serie (Saurer) bei Regenwetter vollkommen fest. Wenn es zum Beispiel nur eine halbe Stunde geregnet hat, kann der Wagen nicht eingesetzt werden, weil er glatt wegrutscht. Benutzbar ist er nur bei ganz trockenem Wetter. Es tritt nun die Frage auf, ob man den Wagen nur am Ort der Exekution im Stand benutzen kann. Erstens muß der Wagen an diesen Ort gebracht werden, was nur bei guter Wetterlage möglich ist. Der Ort der Exekution befindet sich aber meistens 10-15 km abseits der Verkehrswege und ist durch seine Lage schon schwer zugänglich, bei feuchtem oder nassem Wetter überhaupt nicht. Fährt oder führt man die zu Exekutierenden an diesen Ort, so merken sie sofort, was los ist und werden unruhig, was nach Möglichkeit vermieden werden soll. Es bleibt nur der eine Weg übrig, sie am Sammelorte einzuladen und dann hinauszufahren.
132
Я приказал замаскировать автомашины «D» под прицепы, приспособленные под жилье, приделав по одной паре оконных ставен с каждой стороны малого газового автомобиля и по две пары ставен на каждой стороне больших автомобилей так, чтобы это походило на крестьянские домики, какие можно видеть в сельской местности. Эти машины стали настолько хорошо известными, что не только власти, но и гражданское население называют эти автомобили «душегубками». Я думаю, что невозможно даже в течение короткого промежутка времени не только держать в секрете газовый автомооиль, но и замаскировать его».
I ordered the vans of group D to be camouflaged as house-trailers by putting one set of window shutters on each side of the small van and two on each side of the larger vans, such as one often sees on farm houses in the country. The vans became so well-known that not only the authorities but also the civilian population called the van 'death van' as soon as one of the vehicles appeared. It is my opinion the van cannot be kept secret for any length of time, not even camouflaged.
J’ai ordonné que les camions des groupes D soient camouflés en roulottes en mettant des volets de chaque côté, un de chaque côté des petits et deux de chaque côté des grands, comme on en voit souvent dans les campagnes aux maisons de paysans. Ces camions sont tellement connus que les autorités civiles et la population les appellent les camions de la mort, dès qu’ils apparaissent. À mon avis, on ne peut garder le secret très longtemps, même avec du camouflage.
Die Wagen der Gruppe D habe ich als Wohnwagen tarnen lassen, indem ich an den kleinen Wagen auf jeder Seite einen, an den großen Wagen auf jeder Seite zwei Fensterläden anbringen ließ, wie man sie oft an den Bauernhäusern auf dem Lande sieht. Die Wagen waren so bekannt geworden, daß nicht nur die Behörden, sondern auch die Zivilbevölkerung den Wagen als»Todeswagen«bezeichneten, sobald eines dieser Fahrzeuge auftauchte. Nach meiner Meinung kann er auch getarnt nicht auf die Dauer verheimlicht werden.
133
Далее я читаю четвёртый абзац на этой странице:
And then I read the fourth paragraph on this page:
Au paragraphe 4 de la même page, je lis:
Und dann der vierte Absatz auf dieser Seite:
134
Ввиду пересеченной местности, ужасных дорог и шоссе заклепки со временем отходят и появляются щели. Меня просили, чтобы в таких случаях можно было не направлять автомобили на ремонт в Берлин. Перевозка в Берлин была бы слишком дорога и требовала бы слишком много горючего. С целью избежать этих затрат я приказал запаивать небольшие щели, а если это, в конце концов, окажется невозможным, то немедленно информировать Берлин по радио. Затем я приказал, чтобы во время выпуска газа служебный персонал находился на возможно большем расстоянии от автомашин с тем, чтобы их здоровье не страдало от газа, который может иногда выходить наружу. Я хотел бы воспользоваться этим случаем, чтобы обратить Ваше внимание на тот факт, что некоторые команды должны были производить разгрузку после применения газа своими силами. Я обратил внимание командиров специальных подразделений на огромный психологический ущерб для здоровья, который эта работа может иметь для этих людей, если не сразу, то, по крайней мере, позднее. Люди жалуются мне на головные боли, которые появляются после каждой разгрузки. Тем не менее они не хотят уклоняться от выполнения приказа потому, что заключенные, привлеченные к этой работе, могут использовать удобный момент для того, чтобы совершить побег. Я прошу Вас отдать соответствующие приказы для того, чтобы предохранить служебный персонал от ущерба здоровью... Газ не всегда применяется правильным образом. С целью кончить как можно скорее шофер нажимает акселератор до отказа. Таким образом, люди умирают от удушья, а не от отравления, как это было запланировано. Выполнение моих инструкций показало, что при правильном положении рычага управления заключенные мирно впадают в глубокий сон. Больше не приходится наблюдать искаженные лица и испражнения, как это было прежде.
Because of the rough terrain and the indescribable road and highway conditions the caulkings and rivets loosen in the course of time. I was asked if in such cases the vans should not be brought to Berlin for repairs. Transportation to Berlin would be much too expensive and would demand too much fuel. In order to save these expenses I ordered them to have smaller leaks soldered and, if that should no longer be possible, to notify Berlin immediately by radio, that License Number . . . is out of order. Besides that I ordered that during application of gas all the men were to be kept as far away from the vans as possible, so that they should not suffer damage to their health by the gas which eventually would escape. I should like to take this opportunity to bring the following to your attention: Several commands have had the unloading, after the application of gas, done by their own men. I brought to the attention of the commanders of these special detachments concerned the immense psychological injury and damage to their health which that work can have for those men, even if not immediately, at least later on. The men complained to me about headaches which appeared after each unloading. Nevertheless they don't want to change the orders, because they are afraid prisoners called for that work could use an opportune moment to flee. To protect the men from such damage, I request orders be issued accordingly The application of gas usually is not undertaken correctly, In order to come to an end as fast as possible, the driver presses the accelerator to the fullest extent. By doing that the persons to be executed suffer death from suffocation and not death by dozing off as was planned. My directions now have proved that by correct adjustment of the levers death comes faster and the prisoners fall asleep peacefully. Distorted faces and excretions, such as could be seen before, are no longer noticed.
À cause du terrain inégal et des conditions de routes indescriptibles, les rivets et le calfeutrage ne tiennent pas. On m’a demandé d’envoyer les camions à Berlin pour les réparer. Les envoyer à Berlin serait trop cher et exigerait trop de carburant. Afin d’éviter ces dépenses, j’ai ordonné que les petites fuites soient soudées sur place et quand on ne pourrait plus continuer, de prévenir Berlin par radio que le camion POL-NR… ne fonctionne plus. En plus, j’ai demandé que pendant que les gaz seraient utilisés, les hommes soient éloignés le plus possible des camions, afin que leur santé ne soit pas éprouvée par l’émanation des gaz. Je voudrais attirer votre attention sur cette question: très souvent on a fait décharger le camion par les hommes, après l’opération, et j’attire l’attention du chef des SK, sur le mal que cela peut leur faire, tant au point de vue santé qu’au point de vue psychologique, sinon de suite tout au moins plus tard. Les hommes se sont plaints qu’ils avaient mal à la tête chaque fois qu’ils déchargeaient les camions. Néanmoins, on ne peut changer les ordres, car les prisonniers qu’on emploierait pour ce travail pourraient profiter d’un moment opportun pour se sauver. Pour protéger les hommes, je demande que des ordres à cet effet soient donnés. L’opération des gaz n’est pas accomplie correctement. Afin d’en finir le plus rapidement possible, le chauffeur appuie sur l’accélérateur, donne les pleins gaz, et les personnes qu’on doit exécuter sont suffoquées et ne s’éteignent pas doucement comme prévu. Les directives ont montré qu’en ajustant bien les leviers, la mort est beaucoup plus rapide et les prisonniers s’endorment paisiblement. On ne remarque plus de visages défigurés et d’excréments comme on en a vus précédemment.
Durch das unebene Gelände und die kaum zu beschreibenden Wege- und Straßenverhältnisse lockern sich im Laufe der Zeit die Abdichtungen und Nietstellen. Ich wurde gefragt, ob in solchen Fällen der Wagen zur Reparatur nach Berlin überführt werden soll. Eine Überführung nach Berlin käme viel zu teuer und würde zu viel Betriebsstoff erfordern. Um diese Ausgaben zu sparen, gab ich die Anordnung, kleinere undichte Stellen selbst zu löten und, wenn das nicht mehr zu machen wäre, sofort Berlin durch Funk zu benachrichtigen, daß der Wagen Pol. Nr.... ausgefallen sei. Außerdem ordnete ich an, bei den Vergasungen alle Männer vom Wagen möglichst fernzuhalten, damit sie durch eventuell ausströmende Gase gesundheitlich nicht geschädigt werden. Bei dieser Gelegenheit möchte ich auf folgendes aufmerksam machen: Verschiedene Kommandos lassen nach der Vergasung durch die eigenen Männer ausladen. Die Kommandeure der betreffenden S.K. habe ich darauf aufmerksam gemacht, welch ungeheure seelische und gesundheitliche Schäden diese Arbeit auf die Männer, wenn auch nicht sofort, so doch später haben kann. Die Männer beklagten sich bei mir über Kopfschmerzen, die nach jeder Ausladung auftreten. Trotzdem will man von dieser Anordnung nicht abgehen, weil man befürchtet, daß die für die Arbeit herangezogenen Häftlinge einen günstigen Augenblick zur Flucht benutzen könnten. Um die Männer vor diesen Schäden zu bewahren, bitte ich, dementsprechende Anordnungen herauszugeben. Die Vergasung wird durchweg nicht richtig vorgenommen. Um die Aktion möglichst schnell zu beenden, geben die Fahrer durchweg Vollgas. Durch diese Maßnahme erleiden die zu Exekutierenden den Erstickungstod und nicht, wie vorgesehen, den Einschläferungstod. Meine Anleitungen haben nun ergeben, daß bei richtiger Einstellung der Hebel der Tod schneller eintritt und die Häftlinge friedlich einschlafen. Verzerrte Gesichter und Ausscheidungen, wie sie seither gesehen wurden, konnten nicht mehr bemerkt werden.
135
Сегодня я продолжу мою инспекционную поездку и посещу группу «В», где я смогу получить дальнейшие сведения. Доктор Векер, унтерштурмфюрер СС».
Today I shall continue my journey to group B. where I can be reached with further news. Signed, Doctor Becker; SS Untersturmfuehrer.
Je poursuivrai mon voyage jusqu’au groupe B où d’autres nouvelles peuvent me parvenir. — Signé: SS Untersturmführer Dr Becker.
Im Laufe des heutigen Tages erfolgt meine Weiterreise nach der Gruppe B, wo mich weitere Nachrichten erreichen können.«Unterschrift:»Dr. Becker, SS Untersturmführer.
136
На странице 3 документа пС-501 имеется письмо гауптштурмфюрера Трюге по вопросу газовых автомобилей, адресованное в главное управление имперской безопасности в Берлине, в комнату номер 2-D-3-A. Это совершенно секретное письмо подтверждает использование газовых автомобилей для уничтожения евреев.
On Page 3 in Document 501-PS we find a letter signed by Hauptsturmfuehrer Truehess on the subject of S-Vans, addressed to the Reich Security Main Office, Room II-D-3-A, Berlin, marked "top secret." This letter establishes that the vans were used for the annihilation of the Jews. I read this top-secret message; subject, "S-Vans":
À la page 3 du document PS-501, nous trouvons une lettre signée du Hauptsturmführer Trühess concernant les camions S, adressée à l’Office principal de Sûreté du Reich, Bureau II – D-3-A Berlin. Très secret. Cette lettre prouve que les camions servaient à l’annihilation des Juifs. Je lis ce message «très secret»:
Auf Seite 3 des Dokuments 501-PS finden wir einen vom Hauptsturmführer Trühess unterzeichneten Brief betreffs S-Wagen, gerichtet an das Reichssicherheitshauptamt Berlin II D 3 A, als»Geheime Reichssache«gekennzeichnet. Dieser Brief zeigt, daß jene Wagen für die Vernichtung von Juden gebraucht wurden. Ich zitiere diese»Geheime Reichssache«mit dem Betreff»S-Wagen«:
137
Транспорт с евреями, которые должны быть подвергнуты особому обращению, прибывает еженедельно в управление начальника полиции безопасности и СД Белоруссии.
A transport of Jews, which has to be treated in a special way, arrives weekly at the office of the commandant of the Security Police and the Security Service of White Ruthenia.
Objet: Camions S. Un contingent de Juifs devant recevoir un traitement spécial arrive toutes les semaines au bureau du commandant de la Police de sûreté et du service de sûreté de la Ruthénie blanche.
Beim Kommandeur der Sipo und des SD Weißruthenien trifft wöchentlich ein Judentransport ein, der einer Sonderbehandlung zu unterziehen ist.
138
Три газовых автомобиля, которые имеются там, недостаточны для этой цели. Я прошу, чтобы прислали еще один газовый автомобиль (пятитонку). В то же время я прошу, чтобы переправили 20 газовых труб для трех автомашин, которыми мы располагаем (две «Диамонд», одна «Заурер»), так как имеющиеся трубы уже пропускают газ. Начальник полиции безопасности и СД Остланда». Подпись».
The three S-vans which are there are not sufficient for that purpose. I request assignment of another S-van (5 tons). At the same time I request the shipment of 20 gas hoses for the three S-vans on hand (two Diamond, one Saurer), since the ones on hand are legally already." Signed-"the Commandant of the Security Police and the Security Service, Ostland."
Les trois camions S qui sont ici n’y suffisent plus. Je demande l’affectation d’un autre camion S de 5 tonnes. En même temps je demande qu’on m’envoie 20 conduites de gaz pour les trois camions S que je possède, un Saurer et deux Diamond, car celles dont on se sert ont déjà des fuites. Signé: Commandant de la Police de sûreté et du service de la sûreté, territoires de l’Est.
Die drei dort vorhandenen S-Wagen reichen für diesen Zweck nicht aus! Ich bitte um Zuweisung eines weiteren S-Wagens (5 Tonner). Gleichzeitig wird gebeten, für die vorhandenen drei S-Wagen (2 Diamond, 1 Saurer) noch 20 Abgasschläuche mitzusenden, da die vorhandenen bereits undicht sind.«Unterschrift:»Der Befehlshaber der Sipo und des SD Ostland.
139
Из документальных доказательств видно, что среди чиновников германского правительства существовали определенные разногласия по поводу того, какие средства и методы следует использовать при осуществлении программы уничтожения.Секретный доклад от 18 июня 1943, адресованный подсудимому Розенбергу, жаловался на то, что 5000 евреев пригодных к работе убиты полицией и СС и упрекал их за то, что те не захоронили тела убитых. Я приобщаю в качестве доказательства документ номер Р-135, экземпляр США-289.
It would appear from the documentary evidence that a certain amount of discord existed between the officials of the German Government as to the proper means and methods used in connection with the program of extermination. A secret report dated 18 June 1943, addressed to Defendant Rosenberg, complained that 5,000 Jews killed by the police and SS might have been used for forced labor and chided them for failing to bury the bodies of those liquidated. I offer in evidence this file, Document Number R-135, Exhibit USA-289.
Il semble, d’après les preuves documentaires, qu’un certain, désaccord existait entre les fonctionnaires du Gouvernement allemand, concernant la meilleure méthode à utiliser pour ce programme d’extermination. Un compte rendu secret en date du 18 juin 1943, destiné à l’accusé Rosenberg se plaignait que 5.000 Juifs tués par la Police et les SS auraient pu être utilisés pour le travail forcé, et les réprimandait pour n’avoir pas enterré les corps de ceux qui ont été exterminés. Je présente comme preuve R-135, qui devient USA-289.
Nach dem dokumentarischen Beweismaterial könnte der Eindruck entstehen, daß gewisse Meinungsverschiedenheiten zwischen den Vertretern der Deutschen Regierung über die geeigneten Mittel und Methoden, die in Verbindung mit dem Programm der Ausrottung angewandt wurden, bestanden haben. Ein geheimer, an den Angeklagten Rosenberg gerichteter Bericht vom 18. Juni 1943 führt Klage darüber, daß 5000 von der Polizei und SS getötete Juden für Zwangsarbeit hätten verwendet werden sollen, und wirft ihnen vor, daß die Leichen der von ihnen Ermordeten nicht begraben wurden. Ich lege zum Beweis diese Akte, Dokument R-135, Beweisstück US-289, vor.
57140
ПредседательЭто находится в этих томах, майор Уолш?
THE PRESIDENT Is it in these volumes, Major Walsh?
LE PRÉSIDENT Est-ce que cela figure dans ces volumes?
VORSITZENDER Ist das in diesen Bänden enthalten, Major Walsh?
58141
УолшСэр, я думаю, что это находится в собрании нашей документальной книги по нашему делу; это находится перед Р-124. Я цитирую письмо адресованное имперскому министру по делам оккупированных восточных территорий, документ Р-124.
MAJOR WALSH I think, Sir, that will be found in the assembly of the document book in our case; that has been placed in front of R-124. I quote from the letter referred to, addressed to the Reich Minister for the Occupied Eastern Territories, the first paragraph of the translation:
COMMANDANT WALSH Je crois que vous le trouverez dans le livre de documents juste avant R-124. Je cite une lettre adressée au ministre du Reich pour les territoires de l’Est, paragraphe 1 de la traduction:
MAJOR WALSH Ich glaube, Herr Vorsitzender, daß es sich in der Sammlung des Dokumentenbuchs unseres Falles befindet; es kommt vor R-124. Ich zitiere ans diesem erwähnten Briefe, der an den Reichsminister für die besetzten Ostgebiete gerichtet ist, den ersten Absatz der Übersetzung:
142
Тот факт, что к евреям применяется особое обращение, не нуждается в дальнейшей дискуссии. Однако с трудом можно поверить, чтобы это совершалось таким образом, как об этом говорится в отчете генерального уполномоченного от 1 июня 1943 г. Что представляет собой Катынь по сравнению с этим? Подумать только, что события станут известны противнику, который может это использовать. Хотя, вероятнее всего, такая пропаганда не будет иметь никакого успеха потому, что люди, которые будут слушать или читать об этом, просто не поверят, что это правда».
The fact that Jews receive special treatment requires no further discussion. However, it appears hardly believable that this was done in the way described in the report of the General Commissioner of 1 June 1943. What is Katyn against that? Imagine only that these occurrences might become known to the other side and be exploited by them! Most likely such propaganda would have no effect, only because people who hear and read about it simply would not be ready to believe it.
Le fait que les Juifs reçoivent un traitement spécial n’a pas à être discuté davantage. Néanmoins, il semble presque incroyable que ceci ait été fait de la façon signalée dans le compte rendu du Commissaire général. 1er juin 1943. Qu’est-ce que Katyn comparé à cela? Imaginez seulement que ceci soit connu de l’autre côté et exploité par eux? Cette propagande n’aurait aucun effet parce que ceux qui en entendraient parler ou qui le liraient ne voudraient pas le croire.
Daß die Juden sonderbehandelt werden, bedarf keiner weiteren Erörterung. Daß dabei aber Dinge vorgehen, wie sie in dem Bericht des Generalkommissars vom 1. Juni 1943 vorgetragen werden, erscheint kaum glaubhaft. Was ist dagegen Katyn? Man stelle sich nur einmal vor, solche Vorkommnisse würden auf der Gegenseite bekannt und dort ausgeschlachtet! Wahrscheinlich würde eine solche Propaganda einfach nur deshalb wirkungslos bleiben, weil Hörer und Leser nicht bereit wären, derselben Glauben zu schenken.
143
Последняя часть абзаца 3 на этой странице гласит:
The last part of Paragraph 3 on this page reads:
La dernière partie du paragraphe 3 de la même page:
Der letzte Teil von Absatz 3 auf dieser Seite lautet:
144
Запирать мужчин, детей и женщин в сараях и поджигать эти сараи не является подходящим методом для борьбы с бандами, даже если это преследует цели истребления населения.Этот метод не служит немецкой идее и сильно вредит нашей репутации
To lock men, women, and children into barns and to set fire to them does not appear to be a suitable method for combatting bands, even if it is desired to exterminate the population. His method is not worthy of the German cause and hurts our reputation severely.
Enfermer des hommes, des femmes et des enfants dans une grange et les incendier ne semble pas une méthode pratique pour combattre des bandes, même si on désire exterminer la population. Cette méthode n’est pas digne de la cause allemande et fait beaucoup de tort à notre réputation.
Männer, Frauen und Kinder in Scheunen zu sperren und diese anzuzünden, scheint mir selbst dann keine geeignete Methode der Bandenbekämpfung zu sein, wenn man die Bevölkerung ausrotten will. Diese Methode ist der deutschen Sache nicht würdig und tut unserem Ansehen stärksten Abbruch.
145
Начальник тюрьмы города Минска Гюнтер в письме от 31 мая 1943 г., которое является частью документа Р-135 и адресовано генеральному комиссару Белоруссии, тема«Акция против евреев» сообщал:
Gunther, the prison warden at Minsk, in a letter dated 31 May 1943, addressed to the General Commissioner for White Ruthenia, subject: "Action against Jews," was critical by implication. With the Court's permission I would like to read this entire letter, part of Document R-135, Page 5, subject: "Action Against Jews":
Günther, gardien de prison de Minsk, dans une lettre datée du 31 mai 1943, adressée au Commissaire général pour la Ruthénie blanche, formule implicitement une critique. Avec la permission du Tribunal, je lirai toute la lettre, qui fait partie du document R-135, page 5, sujet: «Action contre les Juifs.»:
Der Strafanstaltsverwalter von Minsk, Günther, übte in einem an den Generalkommissar für Weißruthenien gerichteten Brief vom 31. Mal 1943 unter dem Betreff»Judenaktion«verhüllt Kritik. Wenn der Gerichtshof gestattet, möchte ich den gesamten Brief verlesen, der einen Teil des Dokuments R-135 bildet, Seite 5. Betreff:»Judenaktion.«
146
13 апреля 1943 г. бывший германский зубной врач Эрнест Израель Тихауер и его жена Эльза Сарра Тихауер, урожденная Розенталь, службой безопасности были помещены в судебную тюрьму… С этого времени все немецкие и русские евреи, которые привозились к нам, подвергались операции по выдергиванию или выламыванию золотых зубов, мостов и коронок. Это производилось за один-два часа до соответствующих действий.
On 13 April 1943 the former German dentist Ernst Israel Tichauer and his wife, Elisa Sara Tichauer, nee Rosenthal, were committed to the court prison by the Security Service.... Since that time all German and Russian Jews who were turned over to us had their gold bridgework, crowns, and fillings pulled or broken out. His happens always 1 to 2 hours before the respective action.
Le 13 avril 1943, l’ancien dentiste allemand, Ernst Israel Tichauer et sa femme, Elisa Sarah Tichauer, née Rosenthal, furent amenés à la prison par le service de sûreté. Depuis ce temps-là tous les Juifs allemands et russes qui nous furent confiés furent dépouillés de toutes leurs couronnes, bridges ou plombages en or.
Am 13. April 1943 wurde der deutsche ehemalige Zahnarzt Ernst Israel Tischauer und seine Frau Elisa Sara Tischauer, geb. Rosenthal durch den SD... ins Gerichtsgefängnis eingeliefert. Seit dieser Zeit wurden bei den eingelieferten deutschen und russischen Juden die Goldbrücken, Kronen und Plomben ausgezogen, beziehungsweise ausgebrochen. Dieses geschieht jedesmal 1-2 Stunden vor der betreffenden Aktion.
147
Начиная с 13 апреля 1943 г. было убито 516 немецких и русских евреев. Точно установлено, что золотые предметы изо рта были извлечены в течение только двух операций: в первый раз 14 апреля 1943 г. у 172 евреев и во второй раз 27 апреля 1943 г. у 164 евреев. Примерно 50 процентов евреев имеют золотые зубы, мосты и пломбы. При этом всегда лично присутствовал гауптшарфюрер СД Рубе из службы безопасности, который и забирал золото.
Since 13 April 1943, 516 German and Russian hews have been finished off. On the basis of a definite investigation gold was taken only in two actions on 14 April 1943, from 172, and on 27 April 1943, from 164 Jews. About 50 percent of the Jews had gold teeth, bridgework, or fillings. Hauptscharfuehrer Ruebe of the Security Service was always personally present, and he took the gold along, too.
Ceci se produisait une heure ou deux avant que leur sort ne soit réglé (action spéciale). 516 Juifs allemands et russes ont été tués depuis avril 1943. Nous n’avons pris de l’or qu’au cours de deux opérations: le 14 avril 1943 sur 172 Juifs, et le 27 avril 1943 sur 164 Juifs. Environ 50 % des Juifs avaient des dents en or et des bridges ou des plombages. Le Hauptscharführer Rübe, du service de sûreté était toujours personnellement présent, et il emmenait l’or saisi.
Es wurden seit dem 13. April 1943 516 deutsche und russische Juden erledigt. Nach genauer Feststellung wurden aber nur bei 2 Aktionen die Goldsachen abgenommen, und zwar am 14. April 1943 bei 172 und am 27. April 1943 bei 164 Juden. Ungefähr 50 % der Juden hatten Goldzähne, Brücken oder Plomben. Hauptscharführer Rübe vom SD war jedesmal persönlich zugegen und hat auch die Goldsachen mitgenommen.
148
До 13 апреля 1943 г. этого не делалось. Подпись: Гюнтер, начальник тюрьмы».
Before 13 April 1943 this was not done. Signed, Gunther, Prison Warden.
Avant le 13 avril 1943 nous n’avions jamais fait ceci. Signé: Günther, gardien de prison.
Vor dem 13. April 1943 ist dies nicht gemacht worden. Gezeichnet: Günther, Strafanstaltsverwalter.
149
Это письмо было направлено подсудимому Розенбергу как имперскому министру по делам оккупированных восточных территорий в июне 1943 года.Я зачитаю сопроводительное письмо, часть документа Р-124, страница 4, рейхсминистру оккупированных восточных территорий, Берлин, через рейхскомиссара Остланда, Рига; предмет: «Акция против евреев в тюрьме Минска»:
This letter was forwarded to the Defendant Rosenberg as Reich Minister for the Occupied Eastern Territories on 1 June 1943. I will read the covering letter, part of Document R-135, Page 4, to the Reich Minister of the Occupied Eastern Territories, Berlin, through the Reich Commissioner for the Ostland, Riga; Subject, "Actions against Jews in the Prison of Minsk":
Cette lettre fut envoyée à l’accusé Rosenberg, ministre du Reich pour les territoires occupés de l’Est, le 1er juin 1943. Je lirai la lettre d’envoi qui fait partie du document R-135, page 4, au ministre du Reich pour les territoires occupés de l’Est, Berlin, par l’intermédiaire du Commissaire de l’Est, Riga. Sujet: «Opération contre les Juifs dans la prison de Minsk.»
Dieser Brief wurde an den Angeklagten Rosenberg als Reichsminister für die besetzten Ostgebiete am 1. Juni 1943 weitergeleitet. Ich will nun den Begleitbrief, der ein Teil des Dokuments R-135, Seite 4, ist, verlesen. Er ist an den Reichsminister für die besetzten Ostgebiete, Berlin, über den Reichskommissar für das Ostland, Riga, gerichtet und betrifft:»Judenaktionen im Gefängnis von Minsk«.
150
Настоящим прилагается официальный отчёт надзирателя тюрьмы в Минске, представляемый для сведения рейхсминистру и рейхскомиссару для сведения» — подписано — «генеральный комиссар в Минске».
The enclosed official report from the warden of the prison in Minsk is submitted to the Reich Minister and the Reich Commissioner for Information." — Signed — "the General Commissioner in Minsk.
Le compte rendu officiel ci-joint du gardien de prison de Minsk est soumis au ministre du Reich et commissaire du Reich pour information. Signé: Le Commissaire général de Minsk.
Die beigefügte dienstliche Meldung des Strafanstaltsverwalters vom Gefängnis in Minsk überreiche ich dem Herrn Reichsminister und dem Reichskommissar zur Kenntnis. Unterschrift:»Der Generalkommissar in Minsk.«>
59151
Председатель«Соответствующая акция» на которую указано в письме от 31 мая 1943 означает казнь?
THE PRESIDENT Does "respective action," as indicated in the letter dated the 31st of May 1943, mean execution?
LE PRÉSIDENT Est-ce que «action spéciale» signifie exécution?
VORSITZENDER Bedeutet»betreffende Aktion«, wie im Brief vom 31. Mai 1943 angeführt,»Exekution«?
60152
УолшДа, сэр; мы так это интерпретируем. Суд вспомнит, что перевозка евреев в газовых фургонах очень тесно связана с вторым письмом о транспорте евреев прибывших с этой целью.
MAJOR WALSH Yes, Sir; we so interpret it. The Court will recall that the ridding of the Jews via gas vans ties in very closely with the second letter of the transport of Jews arriving for that purpose.
COMMANDANT WALSH Oui, on l’interprète ainsi. Le Tribunal se souvient que l’extermination des Juifs au moyen des camions à gaz a un rapport très étroit avec la seconde lettre qui traite du transport des Juifs effectué dans ce but.
MAJOR WALSH Ja, Herr Vorsitzender, wir legen es so aus. Der Gerichtshof wird sich erinnern, daß die Vernichtung von Juden in Gaswagen sehr eng mit dem zweiten Brief über den Transport von Juden, der zu diesem Zweck ankam, zusammenhängt.
61153
ПредседательДокумент был обнаружен в материалах Розенберга?
THE PRESIDENT Was this document found in Rosenberg's file?
LE PRÉSIDENT Est-ce que ce document était dans les dossiers de Rosenberg?
VORSITZENDER Wurde dieses Dokument im Archiv Rosenbergs gefunden?
62154
УолшЯ так проинформирован, сэр. Следующая жалоба находится в секретном письме начальнику управления военной экономики и вооружения генералу Томасу от 2 декабря 1941. Следует заметить, что обнаруженный автор этого письма заявил о том, что не послал сообщение по официальным каналам. Я приобщаю в качестве доказательства документ ПС-3257, экземпляр США-290.
MAJOR WALSH I am so informed, Sir. A further complaint is contained in a secret letter addressed to General of the Infantry Thomas, chief of the industrial armament department, dated 2 December 1941. It might be noted with interest that the apprehensive writer of this letter stated that he did not forward the communication through official channels. I offer in evidence captured Document 3257-PS; and I quote from the first paragraph. This is Exhibit USA-290:
COMMANDANT WALSH D’après ce qu’on m’a dit, oui, Monsieur le Président. Une autre plainte figure dans une lettre secrète adressée au général d’infanterie Thomas, chef du département industriel de l’Armement, datée du 2 décembre 1941. On peut noter que l’auteur timoré de cette lettre dit qu’il ne l’a pas fait parvenir par la voie officielle. Je présente comme preuve le document PS-3257 (USA-290) et je cite le paragraphe 1:
MAJOR WALSH Ich wurde dahin unterrichtet, Herr Vorsitzender. Eine weitere Beschwerde befindet sich in einem geheimen Brief an den General der Infanterie Thomas, den Chef des Wirtschaftsrüstungsamts vom 2. Dezember 1941. Es kann daraus die interessante Wahrnehmung entnommen werden, daß der besorgte Schreiber dieses Briefes bemerkt, daß er die Weiterleitung dieses Briefes auf dem Dienstwege nicht wünsche. Ich lege das erbeutete Dokument 3257-PS als Beweisstück vor und zitiere den ersten Absatz. Es ist Beweisstück US-290:
155
Я направляю полный отчет о нынешнем положении в рейхскомиссариате Украины для личного сведения начальнику управления военной экономики и вооружения. В этом отчете с бесспорной ясностью описываются трудности и напряженная обстановка, которые имелись до сих пор, а также проблемы, которые могут вызвать серьезное беспокойство.
For the personal information of the chief of the industrial armament department, I am forwarding a total account of the present situation in the Reichskommissariat Ukraine in which the difficulties and tensions encountered so far and the problems which give rise to serious anxiety are stated with unmistakable clarity.
Pour l’information personnelle du chef du département de l’armement industriel, j’envoie au commissariat du Reich pour l’Ukraine un compte rendu de la situation actuelle dans lequel les difficultés rencontrées jusqu’à présent et cet inquiétant problème sont exposés avec une clarté indiscutable.
Zur persönlichen Unterrichtung des Herrn Chefs des Wirtschaftsrüstungsamtes übergebe ich einen Gesamtbericht über die derzeitige Lage im Reichskommissariat Ukraine, in welchem die bisher aufgetretenen Schwierigkeiten und Spannungen, sowie die zu ernsten Besorgnissen Anlaß gebenden Fragen mit vollster Offenheit und unmißverständlicher Deutlichkeit niedergelegt sind.
156
Я умышленно решил отказаться от передачи этого отчета через официальные каналы или же сделать известным его содержание другим ведомствам, так как не жду от этого никаких результатов. Наоборот, я сознаю, что ввиду сложности ситуации могут только увеличиться трудности и напряженность обстановки и усилиться расхождения во взглядах.
Intentionally I have desisted from submitting such a report through official channels or from making it known to other departments interested in it because I do not expect any results that way, but on the contrary am apprehensive that the difficulties and tensions and also the divergent opinions might only be increased due to the peculiarity of the situation.
Intentionnellement je n’ai pas soumis ce compte rendu par les voies officielles et je ne l’ai pas fait connaître aux autres départements intéressés, parce que je n’attendais aucun résultat de ce procédé, et que je prévoyais au contraire que les difficultés et les divergences d’opinions augmenteraient à cause de ces circonstances toutes spéciales.
Ich habe bewußt davon Abstand genommen, einen solchen Bericht auf dem Dienstwege vorzulegen oder ihn anderen interessierten Stellen zur Kenntnis zu bringen, da ich mir keinen Erfolg davon verspreche, vielmehr befürchte, daß die Schwierigkeiten und Spannungen sowie die unterschiedlichen Auffassungen bei der besonderen Art der Verhältnisse sich nur vergrößern würden.
157
Еврейская проблема.
Jewish problem"-Paragraph c, Page 1:
Problème juif (paragraphe C, page 1).
Und nun Absatz c) auf Seite 1:»Judenfrage.
158
Разрешение еврейского вопроса на Украине было трудной проблемой, так как евреи составляли значительную часть городского населения... Во мно-гих городах еврейское население превышало 50 процентов. Только богатые евреи сумели ускользнуть от германских войск. Большинство евреев осталось под властью немцев. Германская администрация оказалась перед трудной проблемой, так как почти все эти евреи представляют собой ремесленное сословие и они используются также на небольших промышленных предприятиях. Кроме того, евреи использовались в учреждениях, которые частично стали излишними в результате прямого или косвенного влияния войны. Поэтому истребление должно было вызвать далеко идущие экономические последствия и даже прямые последствия для производства вооружения».
Regulation of the Jewish question in the Ukraine was a difficult problem because the Jews constituted a large part of the urban population. We therefore have to deal just as in the Government General-with a mass problem of policy concerning the population. Many cities had a percentage of Jews exceeding 50 percent. Only the rich Jews had fled from the German troops. The majority of Jews remained under German administration. The latter found the problem more complicated through the fact that these Jews represented almost entire trade and even a part of the manpower in small and medium industries, besides business, which had in part become superfluous as a direct or indirect result of the war. The elimination therefore necessarily had far-reaching economic consequences and even direct consequences for the armament industry (production for supplying the troops).
Le règlement de la question juive en Ukraine est un problème difficile, car les Juifs constituent une grande partie de la population des villes. Donc, comme dans le cas du Gouvernement Général, nous avons à nous occuper d’un problème général concernant la population. Beaucoup de villes ont un pourcentage de Juifs dépassant 50 %. Seuls les Juifs riches se sont enfuis devant les troupes allemandes. La majorité reste sous l’administration allemande, qui a beaucoup de mal à résoudre ce problème, car ces Juifs représentent presque tout le commerce et même une partie de la main-d’œuvre dans les petites et les moyennes industries que la guerre a supprimées directement ou indirectement. L’élimination produira des contrecoups importants qui atteindront directement l’économie et l’industrie de l’armement (production d’équipement pour les troupes).
Die Ordnung der Judenfrage in der Ukraine war schon deshalb ein schwieriges Problem, weil die Juden in den Städten einen Großteil der Bevölkerung ausmachten. Es handelt sich also – ebenso wie im Generalgouvernement – um ein bevölkerungspolitisches Massenproblem. Viele Städte wiesen einen Judenteil von über 50 % auf. Vor den deutschen Truppen geflohen waren nur die reichen Juden. Das Gros der Judenheit verblieb der deutschen Verwaltung. Für diese komplizierte sich die Frage dadurch, daß diese Juden fast das gesamte Handwerk, sogar einen Teil der Arbeiterschaft der Klein- und Mittel-Industriellen erfüllten, abgesehen vom Handel, der z. T. infolge der direkten oder indirekten Kriegseinwirkung überflüssig geworden war. Die Beseitigung mußte mithin tiefgreifende wirtschaftliche, ja direkt wehrwirtschaftliche Rückwirkungen (Fertigungen für Truppenbedarf) haben.
159
Абзац 1 на странице 2:
Paragraph 1 on Page 2:
Absatz 1 auf Seite 2:
160
Еврейское население относилось к ситуации с беспокойством — с самого начала евреи пытались подчиняться. Они пытались избегать всего, что могло бы вызвать неудовольствие германской администрации. То, что евреи внутренне ненавидели германскую администрацию, само собой понятно и неудивительно. Однако нет доказательства того, что все еврейское население или даже большое число его занималось актами саботажа… Безусловно, среди них были террористы и саботажники, так же как и среди украинцев. Но нельзя сказать, что евреи, представляли собой угрозу для германских вооруженных сил. Производство, в котором были заняты евреи, ускорялось не чем иным, как их страхом, и удовлетворяло как войска, так и германскую администрацию.
The attitude of the Jewish population was anxious-obliging from the beginning. They tried to avoid everything that might displease the German administration. That they hated the German administration and army inwardly goes without saying and cannot be surprising. However, there is no proof that Jewry as a whole or even to a greater part was implicated in acts of sabotage.... Surely there were some terrorists or saboteurs among them, just as among the Ukrainians. But it cannot be said that the Jews as such represented a danger to the German Armed Forces. The output produced by Jews who, of course, were prompted by nothing but the feeling of fear, was satisfactory to the troops and the German administration.
Au début, les Juifs firent preuve d’une craintive soumission. Ils voulaient éviter de faire quoi que ce soit pouvant déplaire à l’administration allemande. Ils détestaient l’administration et l’armée allemandes, cela va sans dire et ne peut surprendre personne, néanmoins il n’est en rien établi que les Juifs en totalité, ou même en majorité, fussent mêlés aux actes de sabotage. Évidemment il y avait quelques terroristes et saboteurs parmi eux comme parmi les Ukrainiens, mais on ne peut dire que les Juifs, comme tels, représentaient un danger pour les Forces armées allemandes. La production juive qui ne marchait bien entendu que par la crainte, était satisfaisante pour les troupes et pour l’administration allemande.
Die Haltung der jüdischen Bevölkerung war von vornherein ängstlich-willig. Sie suchten alles zu vermeiden, um der deutschen Verwaltung zu mißfallen. Daß sie die deutsche Verwaltung und Armee im Inneren haßten, ist selbstverständlich und kann nicht wunder nehmen. Es ist aber nicht beweisbar, daß die Juden geschlossen oder auch nur in größerem Umfang an Sabotageakten... beteiligt waren. Sicher hat es unter ihnen – genau so wie unter den Ukrainern – einige Terroristen oder Saboteure gegeben. Daß die Juden als solche aber irgendeine Gefahr für die deutsche Wehrmacht darstellen, kann nicht behauptet werden. Mit der Arbeitsleistung der Juden, die selbstverständlich durch kein anderes Gefühl als die Angst angetrieben werden, ist Truppe und deutsche Verwaltung zufrieden gewesen.
161
Некоторое время после окончания военных действии еврейское население не трогали. Только лишь спустя несколько недель, иногда месяцев, специально выделенные части полиции начали производить планомерные расстрелы евреев. Эти действия происходили, как правило, в направлении с востока на запад. Это производилось открыто с использованием украинской милиции, и, к несчастью, в некоторых случаях военнослужащие германской армии также принимали в этом добровольное участие. Эти действия распространялись на мужчин, стариков, женщин и детей всех возрастов и проводились ужасным образом. Учитывая огромное количество казненных, эта акция оказывается куда более массовой, чем любая аналогичная по мерам, предпринимаемая до сего времени в Советском Союзе. Примерно от 150 до 200 тысяч евреев было уничтожено в той части Украины, которая входила в рейхскомиссариат. Никакого внимания не обращалось на интересы экономики.
The Jewish population remained temporarily unmolested shortly after the fighting. Only weeks, sometimes months later, specially detached formations of police executed a planned shooting of Jews. This action as a rule proceeded from east to west. It was done entirely in public with the use of the Ukrainian militia; and unfortunately, in many instances also with members of the Armed Forces taking part voluntarily. The way these actions, which included men and old men, women, and children of all ages, were carried out was horrible. The great masses executed make this action more gigantic than any similar measure taken so far in the Soviet union. So far about 150,000 to 200,000 Jews may have been executed in the part of the Ukraine belonging to the Reichskommissariat; no consideration was given to the interests of economy.
La population juive ne fut pas inquiétée tout de suite après les combats. C’est seulement des semaines, parfois des mois après, que des formations spéciales de police fusillèrent les Juifs, d’après un plan précis. Cette opération commençait à l’Est et s’étendait vers l’Ouest. Cela se faisait en public en utilisant la milice ukrainienne et dans beaucoup de cas, malheureusement, des membres des Forces armées y prenaient part aussi en tant que volontaires. Cette action était dirigée contre des hommes, des vieillards, des femmes, des enfants de tout âge qui étaient exécutés de façon horrible. Ces exécutions en masse donnent à cette action un caractère plus terrible que toute mesure semblable prise en Union Soviétique. 150.000 à 200.000 Juifs ont été exécutés dans la partie de l’Ukraine appartenant au Commissariat du Reich, sans prendre les intérêts de l’économie en considération.
Die jüdische Bevölkerung ist im unmittelbaren Anschluß an die Kampfhandlungen zunächst unbehelligt geblieben. Erst Wochen, zum Teil Monate später wurde eine planmäßige Erschießung der Juden durch dazu eigens abgestellte Formationen der Ordnungspolizei durchgeführt. Diese Aktion ging im wesentlichen von Osten nach Westen. Sie erfolgte durchaus öffentlich unter Hinzuziehung ukrainischer Miliz, vielfach leider auch unter freiwilliger Beteiligung von Wehrmachtsangehörigen. Die Art der Durchführung der Aktionen, die sich auf Männer und Greise, Frauen und Kinder jedes Alters erstreckte, war grauenhaft. Die Aktion ist in der Massenhaftigkeit der Einrichtungen so gigantisch wie bisher keine in der Sowjetunion vorgenommene gleichartige Maßnahme. Insgesamt dürften bisher etwa 150000 bis 200000 Juden in dem zum Reichskommissariat gehörigen Teil der Ukraine exekutiert»worden sein«.
162
Суммируя все это, можно сказать, что тот способ разрешения еврейской проблемы, который применялся на Украине и который в принципиальном отношении основывался на идеологических теориях, имел следующие результаты:
Summarizing, it can be said that the kind of solution of the Jewish problem applied to the Ukraine, which obviously was based on the ideological theories as a matter of principle, had the following results:
En résumé, on peut dire que la solution du problème juif appliquée en Ukraine, qui était basée sur des théories idéologiques érigées en principe, eut les résultats suivants:
Insgesamt kann gesagt werden, daß die in der Ukraine durchgeführte Art der Lösung der Judenfrage, offenbar von prinzipiell weltanschaulichen Gedankengängen getragen, nachstehende Folgen gehabt hat:>
163
а) уничтожение отчасти излишней категории едоков среди городского населения;
(a) Elimination of a part of partly superfluous eaters in the cities;
a) Élimination d’une surpopulation des villes;
a) Beseitigung eines Teils zum Teil überflüssiger Esser in den Städten,>
164
б) уничтожение части населения, которая, вне всякого сомнения, нас ненавидела;
(b) Elimination of a part of the population which undoubtedly hated us; .
b) Élimination d’une partie de la population qui nous haïssait sans conteste;
b) Beseitigung eines Bevölkerungsteils, der uns zweifellos haßte,>
165
в) уничтожение ремесленников, в которых имелась острая нужда и которые необходимы даже в интересах вооруженных сил;
(c) Elimination of badly needed tradesmen who were in many instances indispensable even in the interests of the Armed Forces;
c) Élimination de commerçants dont l’absence se fait cruellement sentir et qui étaient souvent même indispensables aux intérêts des Forces armées;
c) Beseitigung dringend notwendiger Handwerker, die auch für Wehrmachtsbelange vielfach unentbehrlich waren,>
166
г) результаты, сказавшиеся на внешней политике, — пропаганда, которая проводится вне всякого сомнения; д) отрицательное воздействие на войска, которые во всяком случае имели косвенное отношение к экзекуциям;
(d) Consequences as to foreign policy propaganda which are obvious;
d) Conséquences évidentes concernant la politique extérieure et la propagande;
d) Außenpolitisch-propagandistische Folgen, die auf der Hand liegen,>
167
е) дурное влияние на части регулярной полиции, которые производят казни».
(e) Bad effects on the troops which in any case get indirect contact with the execution;
e) Effets fâcheux sur les troupes qui prennent part aux exécutions;
e) Nachteilige Wirkungen auf die jedenfalls mittelbar mit den Exekutionen in Berührung kommende Truppe,>
168
(f) Brutalizing effect on the formations which carry out the execution-regular police.
f) Effet abrutissant sur les formations directement chargées des exécutions, la police régulière.
f) Verrohende Wirkung auf die die Exekutionen durchführenden Formationen (Ordnungspolizei).
169
Все это имело место не только на Востоке. То же самое об обращении с евреями на Западе можно видеть из официального отчета голландского правительства, составленного комиссаром по репатриации.
Lest the Court be persuaded to the belief that these conditions related, existed only in the East, I invite attention to the official Netherlands Government report by the Commissioner for Repatriation as indicative of the treatment of the Jews in the West.
Ces conditions n’existaient pas seulement à l’Est et j’attire l’attention du Tribunal sur un compte rendu officiel du Gouvernement des Pays-Bas, rédigé par le Commissaire au Rapatriement qui donne une idée du traitement infligé aux Juifs à l’Ouest.
Damit der Gerichtshof nicht glaubt, daß die geschilderten Zustände nur im Osten bestanden, möchte ich seine Aufmerksamkeit auf den offiziellen holländischen Regierungsbericht des Kommissars für Rückwanderung lenken, der bezeichnend für die Behandlung der Juden im Westen ist.
170
Этот документ повествует о германских мероприятиях в Нидерландах против голландских евреев: декреты, антисемитские демонстрации, сжигание синагог, изгнание евреев из экономической жизни страны, ограничения в снабжении продовольствием, рабский труд, заключение в концентрационные лагеря, насильственный вывоз и смерть — все это следовало той же программе, которая проводилась в жизнь на территории всей оккупированной фашистами Европы.
This document is a recital of the German measures taken in the Netherlands against the Dutch Jews. The decrees, the anti-Semitic demonstrations, the burning of synagogues, the purging of Jews from the economic life of their country, the food restrictions against them, forced labor, concentration camp confinement, deportation, and death-all follow the same pattern that was effected throughout Nazi-occupied Europe.
Ce document décrit les mesures que les Allemands ont prises contre les Juifs hollandais dans les Pays-Bas: les décrets, les démonstrations antisémites, l’incendie des synagogues, l’exclusion des Juifs de la vie économique de leur pays, les restrictions alimentaires, les travaux forcés, l’internement dans les camps de concentration, la déportation, la mort, tout ceci était la même chose dans toute l’Europe occupée par les nazis.
Dieses Schriftstück ist eine Schilderung der deutschen Maßnahmen in den Niederlanden gegen die holländischen Juden. Die Erlasse, die antisemitischen Demonstrationen, das Verbrennen von Synagogen, das Ausstoßen von Juden aus dem Wirtschaftsleben ihres Landes, die ihnen auferlegten Nahrungsmitteleinschränkungen, Zwangsarbeit, Haft in Konzentrationslagern, Verschleppung und Tod, – all das folgt derselben Schablone, die im ganzen nazi-besetzten Europa angewendet wurde.
171
В распоряжении Трибунала имеется документ ПС-1726, США-195. Я не намереваюсь оглашать его полностью, но следует обратить внимание Трибунала на ту часть отчета, где говорится о насильственном вывозе голландских евреев. Это на пятой странице перевода. Здесь говорится о том, что общее число евреев, подлежащих вывозу, составляло 140 тысяч. Общее число насильственно вывезенных лиц достигало 117 тысяч, что составляет более чем 83 процента всех евреев в Нидерландах. Из этих насильственно вывезенных лиц, согласно голландскому отчету, 115 тысяч было направлено в Польшу для принудительного труда, и после этого все сведения о них теряются. Вне зависимости от того, победит ли Германия или потерпит поражение, евреи были обречены. Нацистское руководство провозгласило свое намерение — какова бы ни была судьба Германии, евреи не выживут.
I now refer to Document 1726-PS, Exhibit USA-195, already in evidence. It is not intended to read this document in evidence, but it is deemed important to invite the Court's attention to that portion of the report relating to the deportation of Dutch Jews shown on Page 5 of the translation. There the Court will note that full Jews being liable to deportation number 140,000. The Court will also note that the total number of deportees was 117,000, representing more than 83 percent of all the Jews in the Netherlands. Of these 115,000 were deported to Poland for slave labor, according to the Netherlands report, and after departure all trace of them was lost. Regardless of victory or defeat to Germany, the Jew was doomed. It was the expressed intent of the Nazi State that, whatever the German fate might be, the Jew would not survive.
Je me réfère maintenant au document PS-1726 (USA-195), déjà présenté comme preuve. Je ne le lirai pas comme preuve, mais il est important d’attirer l’attention du Tribunal sur la partie du rapport traitant de la déportation des Juifs hollandais, page 5 de la traduction. Le Tribunal notera que le nombre de Juifs susceptibles d’être déportés, peut être évalué à 140.000. Il remarquera aussi que le nombre total des déportés Juifs hollandais fut 117.000, représentant 83 % de la totalité des Juifs aux Pays-Bas. 115.000 d’entre eux furent déportés en Pologne pour le travail forcé, d’après le rapport hollandais, et après leur départ on a perdu leur trace. Que la guerre soit gagnée ou perdue pour l’Allemagne, les Juifs étaient perdus. C’était l’intention de l’État nazi que, quoiqu’il advînt de l’Allemagne, le Juif ne survive pas.
Ich beziehe mich nun auf Dokument 1726-PS, US-195, das bereits als Beweismaterial vorliegt. Es ist nicht beabsichtigt, dieses Dokument zum Beweis zu verlesen, aber es scheint wichtig, die Aufmerksamkeit des Gerichtshofs auf den Teil des Berichts zu lenken, der sich mit der Verschleppung von holländischen Juden befaßt und auf Seite 5 der Übersetzung zu finden ist. Daraus wird der Gerichtshof entnehmen, daß die Anzahl der für die Deportation in Frage kommenden Volljuden 140000 betrug. Der Gerichtshof wird auch bemerken, daß die Gesamtzahl der. Verschleppten 117000 betrug, eine Zahl, die mehr als 83 % aller niederländischen Juden darstellt. Von diesen wurden dem niederländischen Bericht zufolge 115000 nach Polen zur Zwangsarbeit verschickt; nach ihrer Abfahrt konnte man keine Spur mehr von ihnen finden. Ob Sieg oder Niederlage für Deutschland, der Jude war dem Untergang geweiht. Es war die offen ausgesprochene Absicht des Nazi-Staates, daß, was immer das deutsche Schicksal sein möge, der Jude nicht unter den Überlebenden bleiben sollte.
172
Далее я представляю сообщение начальника полиции безопасности и СД области Радом, документ Л-53, США — 291, адресованное гауптштурмфюреру СС Тилю, на тему: «Чистка в тюрьмах».
I offer in evidence Document 1~53, stamped "top secret," Exhibit USA-291. This message is from the Commandant of the Sipo and SD for the Radom District, addressed to SS Hauptsturmfuehrer Thiel on the subject, "Clearance of Prisons." I read the body of this message:
Je présente comme preuve le document L-53, marqué très secret, USA-291. C’est un message du commandant des SIPO et des SD du district de Radom, adressé au SS Hauptsturmführer Thiel sur le «Nettoyage des prisons». Je lis:
Ich lege nun Dokument L-53, US-291, vor, das den Stempel»Geheime Reichssache«trägt. Diese Mitteilung stammt vom Kommandeur der Sicherheitspolizei und des Sicherheitsdienstes für den Distrikt Radom, ist an den SS-Hauptsturmführer Thiel gerichtet und betrifft die»Räumung von Gefängnissen«. Ich lese nun den Hauptteil dieser Mitteilung:
173
Я вновь обращаю внимание на тот факт, что количество заключенных в тюрьмах ЗИПО и СД должно быть минимальным. При нынешней обстановке заподозренные лица, особенно арестованные гражданской полицией, нуждаются только в коротком формальном допросе, если нет серьезных улик против них. Затем их как можно быстрее надо посылать в концлагеря, если нет необходимости в военно-полевом суде или если они не подлежат освобождению. Пожалуйста, освобождайте как можно меньше арестованных. В случае, если обстановка на фронте делает это необходимым, надо подготовиться к полной очистке тюрем. Если обстановка неожиданно изменится таким образом, что будет невозможно эвакуировать заключенных, их надо ликвидировать, а их тела уничтожить (сжигать, взрывать здания и т. д.). В случае необходимости с евреями, которые еще заняты в промышленности по вооружению или же на других работах, надо поступать таким же образом.
I again stress the fact that the number of inmates of the Sipo and SD prisons must be kept as low as possible. In the present situation, particularly, those suspects handed over by the civil police need only be subjected to a short formal interrogation provided there are no serious grounds for suspicion. They are then to be sent by the quickest route to a concentration camp should no court-martial proceeding be necessary or should there be no question of discharge. Please keep the number of discharges very low. Should the situation at the front necessitate it, early preparations are to be made for the total clearance of prisons. Should the situation develop suddenly in such a way that it is impossible to evacuate the prisoners, the prison inmates are to be eliminated and their bodies disposed of as far as possible (burning, blowing up the building, et cetera). If necessary, Jews still' employed in the armament industry or on other work are to be dealt with in the same way.
Je souligne encore que le nombre d’internés dans les prisons SIPO et SD doit être aussi bas que possible. Dans la situation actuelle, particulièrement les suspects, que la police civile nous a donnés, n’ont besoin que d’un interrogatoire très court s’il n’y a aucune raison sérieuse de les soupçonner. On doit ensuite les envoyer par le moyen le plus rapide dans un camp de concentration. Aucun jugement ne devrait être nécessaire et il ne devrait pas être question de les libérer. Le nombre de ceux à renvoyer chez eux doit être très bas. Si la situation du front se développait et le rendait nécessaire, il faudrait prendre des mesures pour le nettoyage complet des prisons. S’il devenait impossible d’évacuer les prisonniers, les internés doivent être tués et on doit disposer des cadavres le plus vite possible en faisant sauter les bâtiments, en les brûlant, etc. Il faut agir de même avec les Juifs qu’on utilise dans l’industrie de l’armement ou dans d’autres lieux.
Ich weise zum wiederholten Male darauf hin, daß die Insassenzahl der Gefängnisse der Sicherheitspolizei und des SD nach Möglichkeit niedrig gehalten werden muß. Bei der zur Zeit gegebenen Lage können insbesondere von der Ordnungspolizei zugeführte Verdächtige, soweit keine ernstlichen Verdachtsgründe gegen sie vorliegen, nur noch abgekürzt formularmäßig vorgenommen werden. Sie sind alsdann auf dem schnellsten Wege einem KZ zuzuführen, falls nicht ein Standgerichtsverfahren erforderlich wird oder eine Entlassung in Frage kommt. Mit Entlassungen bitte ich sehr zurückhaltend»zu sein. Soweit es die Frontlage erforderlich macht, sind rechtzeitige Vorkehrungen für eine Totalräumung der Gefängnisse zu treffen. Bei überraschender Entwicklung der Lage, die einen Abtransport der Häftlinge unmöglich macht, sind die Gefängnisinsassen zu liquidieren, wobei die Erschossenen nach Möglichkeit beseitigt werden müssen (Verbrennen, Sprengung der Gebäude u. ä.). Gleichermaßen ist eintretendenfalls mit den noch in der Rüstungsindustrie oder an anderen Stellen beschäftigten Juden zu verfahren.
174
Освобождения заключенных или евреев врагом — будь то Красная Армия или «ВБ» — следует избегать при всех обстоятельствах. Они ни в коем случае не должны попадать им в руки живыми».
The liberation of prisoners or Jews by the enemy-be it the WB or the Red Army-must be avoided under all circumstances, nor may they fall into their hands alive.
La libération de prisonniers ou de Juifs par l’ennemi, que ce soit par les WB ou par l’armée rouge, doit être évitée à tout prix, ils ne doivent en aucun cas tomber vivants entre leurs mains.
Unter allen Umständen muß vermieden werden, daß Gefängnisinsassen oder Juden vom Gegner, sei es WB oder Rote Armee, befreit werden beziehungsweise ihnen lebend in die Hände fallen.«
63175
Председатель Что такое ВБ?
THE PRESIDENT What is the WB?
LE PRÉSIDENT Que veut dire le WB?
VORSITZENDER Was ist der WB?
64176
УолшЯ запросил про ВБ, ваша честь, из нескольких источников и не нашёл указания на него. Вероятно перед началом вечернего заседания, я смогу просветить суд. Я пока не нашёл.
MAJOR WALSH I have inquired about the WB, Your Honor, from several sources and have not found an understanding or a statement of it. Perhaps before the afternoon session I may be able to enlighten the Court. I have not yet been able to find out.
COMMANDANT WALSH J’ai demandé plusieurs fois et je n’ai pas trouvé une interprétation ou une explication. Peut-être qu’avant l’audience de cet après-midi je pourrai éclairer le Tribunal. Jusqu’ici je ne sais pas.
MAJOR WALSH Ich habe über den WB bei verschiedenen Quellen Erkundigungen eingezogen, Herr Vorsitzender, habe aber keine Erklärung oder Feststellung gefunden. Ich kann vielleicht vor der Nachmittagssitzung dem Gerichtshof Aufklärung geben. Vorläufig ist es mir noch nicht gelungen, es in Erfahrung zu bringen.
65177
ПредседательГде был обнаружен документ?
THE PRESIDENT Where was the document found?
LE PRÉSIDENT Où ce document a-t-il été trouvé?
VORSITZENDER Wo wurde dieses Dokument gefunden?
66178
Уолш Сэр, это захваченный документ.
MAJOR WALSH It is a captured document, Sir.
COMMANDANT WALSH C’est un document saisi.
MAJOR WALSH Es ist ein erbeutetes Dokument, Herr Vorsitzender.
67179
ПредседательВы сказали он относится к военнопленным?
THE PRESIDENT Does it relate to prisoners of war, did you say?
LE PRÉSIDENT Parle-t-il de prisonniers de guerre?
VORSITZENDER Sagten Sie, daß es sich auf Kriegsgefangene bezieht?
68180
УолшНет сэр, он конечно их включает, заключённых и евреев. Я попытаюсь найти информацию о его истории.
MAJOR WALSH No, Sir; including therein, of course, prisoners of war as well as all Jews. The history of the document, Sir, I will try to gather for the Court's information.
COMMANDANT WALSH Non, mais il y est question de prisonniers de guerre tout comme de Juifs. Je vais essayer d’obtenir l’historique de ce document et de l’apporter au Tribunal.
MAJOR WALSH Nein, Herr Vorsitzender; aber selbstverständlich sind Kriegsgefangene ebenso wie alle Juden davon betroffen worden. Ich werde trachten, die Geschichte des Dokuments zu ermitteln und zur Kenntnis des Gerichtshofs zu bringen.
69181
ПредседательДа. Вы сказали нам, что такое Зипо?
THE PRESIDENT Yes. Did you tell us what the Sipo were?
LE PRÉSIDENT Qu’est-ce que la SIPO?
VORSITZENDER Gut. Sagten Sie uns, was die Sipo ist?
70182
УолшДа, сэр; я сообщил это суду; это полиция безопасности, сэр.
MAJOR WALSH Yes, Sir; I furnished the Court with that; that is the Security Police, Sir.
COMMANDANT WALSH C’est la Police de sûreté.
MAJOR WALSH Ja, Herr Vorsitzender. Ich sagte dem Gerichtshof, daß dies die Sicherheitspolizei ist.
183
С позволения суда, презентация была бы неполной без добавления в неё ссылки на концентрационные лагеря, постольку поскольку это относится к тысячам — миллионам евреев которые погибли от массовых расстрелов, газа, яда, голода и иных мер. Предмет концентрационных лагерей и все его ужасы были показаны трибуналу не только кинофильмом, но и в подробной презентации господина Додда; и сейчас, мы не намерены обращаться к лагерям, лишь постольку поскольку это относится к истреблению еврейского народа. В лагере Освенцим в течение июля 1944 года ежедневно убивали 12 тысяч евреев. Сведения об этом содержатся в документе Л-161, США-292. Этот документ представляет собой официальный польский отчет о концлагере Освенцим, датированный 31 мая 1945 г.Я воспользуюсь коротким фрагментом из этого отчёта…
This presentation, if the Court please, would be incomplete without incorporating herein reference to the concentration camps insofar as they relate to the hundreds of thousands-millions-of Jews who died by mass shooting, gas, poison, starvation, and other means. The subject of concentration camps and all its horrors was shown to the Tribunal not only in the motion picture film but by the most able presentation by Mr. Dodd yesterday; and it is not intended, at this time, to refer to the camps - only in so far as they relate to the part played in the annihilation of the Jewish people. For example, in the camp at Auschwitz during July 1944 Jews were killed at the rate of 12,000 daily. This information is contained in Document L-161, Exhibit USA-292. The Document L-161 is an official Polish report on Auschwitz Concentration Camp. It is dated 31 May 1945. I have taken a short excerpt from this report on the original marked...
Cette présentation ne serait pas complète si on n’y comprenait pas les camps de concentration puisque des millions de Juifs y sont morts fusillés, gazés, empoisonnés, morts de faim, et par tous autres moyens. Les camps de concentration avec toutes leurs atrocités ont été présentés non seulement dans le film mais dans l’excellent exposé de M. Dodd hier. Nous n’avons pas l’intention de parler maintenant de ces camps, mais seulement en tant qu’ils ont joué un rôle dans l’anéantissement de la race juive. Par exemple dans le camp d’Auschwitz, en juillet 1944, les Allemands ont tué environ 12.000 Juifs par jour. Cette information figure dans le document L-161 (USA-292). C’est un compte rendu officiel polonais du camp de concentration d’Auschwitz, daté du 31 mai 1945. Je choisis un extrait de ce compte rendu marqué sur l’original…
Hoher Gerichtshof! Diese Darstellung wäre ohne Erwähnung der Konzentrationslager unvollständig, soweit sie mit den Hunderttausenden, ja Millionen von Juden zusammenhängen, die durch Massenerschießungen, Gas, Gift, Aushungern und auf andere Weise starben. Das Thema der Konzentrationslager und aller ihrer Schrecken wurde vor dem Gerichtshof nicht allein in dem Film, sondern auch gestern in der vorzüglichen Darstellung von Herrn Dodd behandelt; wir wollen heute von Konzentrationslagern nur sprechen, soweit sie bei der Vernichtung des jüdischen Volkes eine Rolle spielten. So wurden z.B. im Juli 1944 im Lager Auschwitz täglich 12000 Juden getötet. Dies ergibt sich aus Dokument L-161, US-292. Dokument L-161 ist ein offizieller polnischer Bericht über das Konzentrationslager Auschwitz. Es ist vom 31. Mai 1945 datiert; ich habe einen kurzen Auszug aus diesem Bericht entnommen, der auf dem Original angezeichnet ist...
71184
ПредседательДумаю, вы ошиблись, разве нет? Это не польский доклад, а британский.
THE PRESIDENT I think you made a mistake, did you not? It is not a Polish report; it is a British report.
LE PRÉSIDENT Est-ce que vous ne faites pas erreur? Ce n’est pas un rapport polonais, c’est un rapport britannique.
VORSITZENDER Ich glaube, Sie haben sich geirrt. Es ist nicht ein polnischer, sondern ein britischer Bericht.
72185
УолшСэр, как я понимаю, он был составлен польским правительством и издан в Лондоне.
MAJOR WALSH I understand, Sir, it was compiled originally by the Polish Government and perhaps distributed from London.
COMMANDANT WALSH Je sais, Monsieur le Président, il vient primitivement du Gouvernement polonais mais il a peut-être été distribué par Londres.
MAJOR WALSH Soviel ich weiß, Herr Vorsitzender, wurde er ursprünglich von der Polnischen Regierung zusammengestellt und vielleicht von London herausgegeben.
73186
Председатель Понятно. Очень хорошо.
THE PRESIDENT I see. Very well.
LE PRÉSIDENT Commandant, vous avez déjà lu ceci vous-même.
VORSITZENDER Ich verstehe, sehr gut.
74187
УолшЯ цитирую:
MAJOR WALSH I quote:
COMMANDANT WALSH Je cite:
MAJOR WALSH Ich zitiere:
188
В течение июля 1944 года убивали в день по 12 тысяч венгерских евреев, а так как крематорий не мог пропустить такое количество трупов, то они сбрасывались в глубокие ямы и засыпались негашеной известью».
During July 1944 Hungarian Jews were being liquidated at the rate of 12,000 daily; and as the crematoria could not deal with such numbers, many bodies were thrown into large pits and covered with quicklime.
Au cours du mois de juillet 1944, on liquidait 12.000 Juifs hongrois par jour. Les fours crématoires ne pouvaient en absorber un tel nombre, beaucoup de corps furent jetés dans de grands trous qu’on recouvrait de chaux vive.
Im Juli 1944 wurden täglich 12000 ungarische Juden liquidiert; da das Krematorium solche Massen nicht bewältigen konnte, wurden viele Leichen in große Gräben geworfen und mit ungelöschtem Kalk bedeckt.
189
Представляю в качестве доказательства официальный отчет польской правительственной Комиссии по расследованию немецких преступлений в Польше. Этот документ, ПС-3311, США-293, описывает концлагерь в Треблинке.
I offer in evidence Document 3311-PS, Exhibit USA-293. This is an official Polish Government Commission report on the investigation of German crimes in Poland. The document describes the concentration camp at Treblinka; and from Page 1, Paragraph 3 and 4, I read as follows:
Je présente comme preuve le document PS-3311 (USA-293). C’est un compte rendu officiel de la Commission du Gouvernement polonais pour la recherche des crimes allemands contre la Pologne. Ce document décrit le camp de concentration de Treblinka, et à la page 1, paragraphes 3 et 4, je lis:
Ich lege nun Dokument 3311-PS, US-293, als Beweismaterial vor. Dies ist ein Bericht der Kommission der Polnischen Regierung für die Untersuchung deutscher Verbrechen in Polen. Dieses Dokument beschreibt das Konzentrationslager in Treblinka; von Seite 1, Abschnitt 3 bis 4, lese ich folgendes:
190
В марте 1942 года немцы приступили к сооружению другого лагеря Треблинка-Б, близ Треблинка-А, который должен был стать местом для истребления евреев.
In March 1942 the Germans began to erect another camp, Treblinka B. in the neighborhood of Treblinka A, intended to become a place of torment for Jews.
En mars 1942, les Allemands commencèrent à établir un autre camp, Treblinka B, près de Treblinka A, qui devait devenir un lieu de tortures pour les Juifs.
Im März 1942 begannen die Deutschen ein anderes Lager, ›Treblinka B‹ in der Nachbarschaft von ›Treblinka A‹, zu errichten; es sollte ein Marterplatz für Juden werden.
191
Создание этого лагеря находилось в тесной связи с германским планом, который имел целью полное уничтожение еврейского населения в Польше. Это, в свою очередь, вызвало необходимость создания аппарата, при помощи которого польских евреев можно было бы убивать в массовом порядке. В апреле 1942 года было закончено строительство первых камер, в которых жертвы должны были умерщвляться газом. Затем было закончено создание основного здания, где производились убийства, в котором имелось 10 камер смерти. Открытие его относится к началу августа 1942 года и ознаменовалось производством массовых убийств».
The erection of this camp was closely connected with the German plans aimed at a complete destruction of the Jewish population in Poland, which necessitated the creation of a machinery by means of which the Polish Jews could be killed in large numbers. Late in April 1942 erection was completed of the first chambers in which these general massacres were to be performed by means of steam. Somewhat later the erection of the real death building, which contains 10 death chambers, was finished. It was opened for wholesale murders early in autumn 1942.
La construction de ce camp était étroitement reliée au plan allemand d’extermination de la population juive en Pologne, qui nécessitait la création d’une organisation permettant de tuer les Juifs polonais en grande quantité. À la fin d’avril 1942, la construction des trois premières chambres était terminée et des massacres en masse devaient y avoir lieu par la vapeur. Un peu plus tard, fut terminée l’érection du vrai bâtiment de la mort avec dix chambres de mort. Le tout fut prêt pour les assassinats en masse au début de l’automne 1942.
Die Errichtung dieses Lagers stand in engem Zusammenhang mit den deutschen Plänen, die jüdische Bevölkerung in Polen völlig auszurotten; dies wiederum machte die Errichtung einer Maschinerie notwendig, durch welche polnische Juden in großen Mengen getötet werden konnten. Gegen Ende April 1942 waren die ersten drei Kammern fertiggestellt, in denen die allgemeinen Massenmorde durch Dampf vollzogen werden sollten. Etwas später wurde das wirkliche Toten-Haus fertiggestellt, welches 10 Todeskammern enthält. Es wurde im Frühherbst 1942 für Massenmorde eröffnet.
192
Затем польская Комиссия описывает последовательность порядкауничтожения в лагере.
And on Page 3 of this report, beginning with the second paragraph, the Polish Commission describes graphically the procedure for the extermination within the camp:
À la page 3 de ce compte rendu, paragraphe 2, la Commission polonaise décrit la procédure d’extermination à l’intérieur de ce camp:
Auf Seite 3 dieses Berichts, beginnend mit dem zweiten Abschnitt, beschreibt die polnische Kommission in anschaulicher Weise das Vernichtungsverfahren innerhalb des Lagers:
193
Среднее число евреев, которые доставлялись в лагерь летом 1942 года, достигало двух железнодорожных эшелонов в день. Но были дни, когда это количество намного превышалось. Начиная с осени 1942 года это количество начало понижаться.
The average number of Jews dealt with at the camp in the summer of 1942 was about two railway transports daily, but there were days of much higher efficiency. From autumn 1942 this number was faring.
Le nombre moyen des Juifs qu’on a liquidé dans ce camp pendant l’été 1942 était d’environ deux trains par jour, mais il y avait des jours où le rendement était bien supérieur. À partir de l’automne 1942, ce nombre alla en décroissant.
Die Durchschnittsanzahl der Juden, die im Sommer 1942 im Lager behandelt wurden, betrug ungefähr 2 Eisenbahnzüge täglich. Es gab jedoch Tage von viel größerer Leistungsfähigkeit. Vom Herbst 1942 ab ging diese Zahl herunter.
194
После разгрузки поезда всех жертв собирали в одном месте, где мужчин отделяли от женщин и детей. В первые дни существования лагеря жертвы убеждались в том, что они должны совершить краткую остановку в лагере для того, чтобы помыться и пройти дезинфекцию, а затем их пошлют дальше на Восток на работу. Такие объяснения давались эсэсовцами, которые помогали при разгрузке эшелона. Кроме того, об этом говорилось в объявлениях, развешенных на стенах бараков. Затем, когда прибывало все большее количество эшелонов, немцы отбрасывали все стеснения и только старались ускорить процедуру.
After unloading in the siding, all victims were assembled in one place, where men were separated from women and children. In the first days of the existence of the camp the victims were made to believe that after a short stay in the camp, necessary for bathing and disinfection, they would be sent farther east for work. Explanations of this sort were given by SS men who assisted at the unloading of the transports, and further explanations could be read in notices stuck up on the walls of the barracks. But later, when more transports had to be dealt with, the Germans dropped all pretenses and only tried to accelerate the procedure.
Après avoir débarqué sur les voies de garages toutes les victimes, elles étaient rassemblées à un endroit, les hommes séparés des femmes et des enfants. Dans les premiers jours de fonctionnement de ce camp, on faisait croire aux victimes qu’elles allaient rester peu de temps, le temps nécessaire pour les bains et la désinfection, puis qu’on les enverrait travailler plus loin à l’Est. Ces explications étaient données par des SS qui assistaient au débarquement, et d’autres explications étaient données au moyen d’affiches collées sur les murs des baraques, mais plus tard, quand il y eut davantage de transports, les Allemands ne prirent plus aucune précaution et essayèrent seulement d’accélérer l’élimination.
Nach der Ausladung auf dem Nebengeleis wurden alle Opfer auf einem Platz zusammengetrieben und Männer von Frauen und Kindern getrennt. In den ersten Tagen des Bestehens des Lagers machte man die Opfer glauben, daß sie nach einem kurzen Aufenthalt im Lager, der zum Baden und zur Des infektion nötig wäre, weiter östlich zur Arbeit gebracht würden. Derartige Erklärungen wurden von SS-Männern gegeben, die beim Ausladen der Transporte halfen. Weitere Erklärungen konnten sie in Bekanntmachungen, die an den Wänden der Baracken angeschlagen waren, lesen. Später, als mehr Transporte bearbeitet werden mußten, ließen die Deutschen alle Vorwände fallen und versuchten nur, das Verfahren zu beschleunigen.
195
Все жертвы должны были снять одежду и обувь, которые потом собирались. Затем все жертвы, в первую очередь женщины и дети, доставлялись в камеры смерти. Те, которые были слишком слабы или двигались медленно, подгонялись ружейными прикладами, хлыстами и кулаками. Зачастую в этом принимал участие сам Зауер. Некоторые поскользнувшиеся падали, остальные жертвы, вынужденные двигаться вперед, шли по их телам. Маленьких детей просто бросали вовнутрь. Затем набитые до отказа камеры герметически закрывались, и в них пускали газ, и через несколько минут все было кончено. Затем мужчиныевреи должны были выносить трупы с платформы и хоронить их в массовых могилах. Прибывали все новые эшелоны, и кладбище все возрастало.
All victims had to strip off their clothes and shoes, which were collected afterwards, whereupon all victims, women and children first, were driven into the death chambers. Those too slow or too weak to move quickly were driven in by rifle butts, by whipping and kicking, often by Saner himself. Many slipped and fell; the next victims pressed forward and stumbled over them. Small children were simply thrown inside. After being filled up to capacity, the chambers were hermetically closed and steam was let in. In a few minutes all was over. The Jewish menial workers had to remove the bodies from the platform and to bury them in mass graves. By and by, as new transports arrived, the cemetery grew, extending in an easterly direction.
Toutes les victimes devaient retirer leurs vêtements et leurs souliers qui étaient rassemblés plus tard. Toutes les victimes, les femmes et les enfants d’abord, étaient menées aux chambres d’extermination. Ceux qui étaient trop lents ou trop faibles pour aller vite étaient poussés à coups de crosse, fouettés, recevaient des coups de pied, souvent Sauer le faisait lui-même. Beaucoup glissaient et tombaient, les suivants trébuchaient par-dessus. Les petits enfants étaient simplement jetés dedans. Quand les chambres étaient complètement remplies, on les fermait hermétiquement et on faisait entrer la vapeur. En quelques minutes tout était fini. Les travailleurs Juifs subalternes devaient retirer les corps et les enterrer dans des fosses communes. De temps en temps, au fur et à mesure que les transports arrivaient, les cimetières s’étendaient, dans la direction de l’Est.
Alle Opfer mußten ihre Kleidungsstücke und Schuhe, die später gesammelt wurden, ausziehen. Dann wurden alle Opfer, zuerst die Frauen und Kinder, in die Totenkammern getrieben. Diejenigen, die zu langsam oder zu schwach waren, um sich schnell zu bewegen, wurden mit Gewehrkolben, durch Peitschen und durch Schläge, häufig von Sauer selbst, angetrieben. Viele glitten aus und fielen; die nächsten Opfer drückten nach vom und fielen über sie. Kleine Kinder wurden einfach hineingeworfen. Nachdem die Kammern bis zu ihrer Fassungskraft vollgestopft waren, wurden sie hermetisch geschlossen und Dampf eingelassen. In wenigen Minuten war alles vorüber. Die jüdischen Knechte mußten die Körper von der Plattform entfernen und in Massengräbern vergraben.
196
Согласно полученным отчетам можно подсчитать, что в Треблинке было истреблено несколько сот тысяч евреев».
From reports received it may be assumed that several hundred thousands of Jews have been exterminated in Treblinka.
Des comptes rendus reçus, on peut déduire que plusieurs centaines de milliers de Juifs ont été exterminés à Treblinka.
Als neue Transporte ankamen, wuchs der Friedhof nach und nach und dehnte sich in östlicher Richtung aus. Nach eingegangenen Berichten kann angenommen werden, daß mehrere hunderttausend Juden in Treblinka vernichtet wurden.«>
197
Документ Л-22, США-294, представляет собой официальный отчет правительства США, составленный управлением по вопросам беженцев при президенте США о положении в немецких лагерях в Освенциме и Биркенау в 1944 году. В этом отчете указано, что общее количество евреев, которые были умерщвлены при помощи газа в немецком лагере Биркенау в период между апрелем 1942 года и апрелем 1944 года, составляет 1 765 000 человек. Меня заверили, что это не типографская ошибка и цифра соответствует действительности.
I now offer in evidence the document identified by Number L-22, Exhibit USA-294. This is an official United States Government report issued by the Executive Office of the President of the United States, War Refugee Board, on the German camps at Auschwitz and Birkenau, dated 1944. On Page 33 of this report is set forth the number of Jews gassed in Birkenau in the 2-year period between April 1942 and April 1944. I have been assured that the figure printed in this report is not a typographical error. The number shown is 1,765,000.
Je présente maintenant comme preuve le document L-22, (USA-294). C’est un rapport officiel du Gouvernement des États-Unis, émanant du Bureau exécutif du Président des États-Unis, Service des Réfugiés de Guerre, concernant les camps de concentration allemands d’Auschwitz et de Birkenau, daté 1944. À la page 33 de ce compte rendu on montre que beaucoup de Juifs furent tués par les gaz à Birkenau en 2 ans, d’avril 1942 à avril 1944. On m’a affirmé que le chiffre figurant ici n’est pas une erreur: il est de 1.765.000.
Ich lege nunmehr ein als L-22, US-294, gekennzeichnetes Dokument zum Beweis vor. Es ist ein amtlicher Bericht der Regierung der Vereinigten Staaten, der von der Kanzlei des Präsidenten der Vereinigten Staaten, Abteilung Kriegsflüchtlinge, über die deutschen Lager in Auschwitz und Birkenau im Jahre 1944 herausgegeben wurde. Auf Seite 33 dieses Berichts ist die Zahl der Juden, die in dem Zeitraum von zwei Jahren, von April 1942 bis April 1944, in Birkenau vergast wurden, angegeben. Es wurde mir versichert, daß die Zahl, die in diesem Bericht abgedruckt ist, kein Druckfehler sei. Die Zahl, die dort erscheint, ist 1765000.
198
I would now like to turn to the German bookkeeping and statistics for enlightenment on the extermination of Jews in Poland. Referring again to the diary of Hans Frank already in evidence, Document 2233-PS, Exhibit USA-281, I read briefly from the beginning of the fourth paragraph on Page 1:
Je vais maintenant revenir aux statistiques et à la comptabilité allemandes pour éclaircir la question de l’extermination des Juifs en Pologne. Je reviens au Journal de Hans Frank déjà présenté, document PS-2233 (USA 281), et je lis rapidement le début du paragraphe 4, page 1:
Ich möchte mich nun der deutschen Buchführung und Statistik zuwenden, um die Vernichtung der Juden in Polen zu beleuchten. Ich wende mich wieder dem Tagebuch von Hans Frank zu, das bereits als Dokument 2233-PS, US-281, vorliegt. Ich lese kurz vom Beginn des vierten Abschnitts auf Seite 1:
199
Для нас евреи являются болезнетворными микробами. В Генерал-губернаторстве находится приблизительно 2500000…
For us the Jews also represent extraordinarily malignant gluttons. We have now approximately 2,500,000 of them in the Government General...
À nos yeux les Juifs sont des gloutons extraordinairement néfastes. Nous en avons environ 2.500.000 dans le Gouvernement Général. Oui, mais j’en parle afin de le comparer avec autre chose.
Die Juden sind auch für uns außergewöhnlich schädliche Fresser. Wir haben im Generalgouvernement schätzungsweise 2500000.«>
75200
ПредседательМайор Уолш, вы уже сами это читали.
THE PRESIDENT Major Walsh, you have read this already yourself.
VORSITZENDER Major Walsh, Sie haben das bereits selbst vorgelesen.
76201
УолшДа, сэр, правильно. Я лишь хочу снова на это сослаться, сэр для сравнения с другими цифрами.
MAJOR WALSH Yes, Sir, that is true. I just want to make reference to it again, Sir, for comparison with other figures.
MAJOR WALSH Ja, das stimmt. Ich möchte hierauf nochmals zurückgreifen, um andere Zahlen damit zu vergleichen.
77202
ПредседательОчень хорошо.
THE PRESIDENT Very well.
LE PRÉSIDENT Très bien.
VORSITZENDER Gut.
78203
Уолш«…вероятно вместе помесными евреями, цифра доходит до 3 500 000 евреев».
MAJOR WALSH ". . . perhaps with the Jewish mixtures, and everything that goes with it, 3,500,000 Jews."
COMMANDANT WALSH «… peut-être avec les sang-mêlé et tout ce qui s’ensuit, 3.500.000 Juifs.»
MAJOR WALSH»...vielleicht mit den jüdischen Versippten und dem, was alles daran hängt, jetzt 3500000 Juden.«
204
Итак, такой была цифра 16 декабря 1941. Сейчас я хочу обратиться к 25 января 1944, 3 годами и 1 месяцем спустя, и я ссылаюсь на ещё один фрагмент из дневника Франка, 2233-ПС, многотомный экземпляр США-295. Этот том охватывает период с 1 января по 28 февраля 1944 и страница 5 оригинала гласит:
Now this figure, if the Court please, was as of 16 December 1941. I now wish to turn to 25 January 1944, 3 years and 1 month later, and make reference to another excerpt from Frank's diary, 2233-PS, loose-leaf volume Exhibit USA-295. This volume covers the period from 1 January 1944 to 28 February 1944, and Page 5 of the original reads:
Ce chiffre, plaise au Tribunal, est du 16 décembre 1941. Je passe au 25 janvier 1944, trois ans et un mois après, à un autre extrait du journal de Frank PS-2233 (USA-295). Ce volume comprend la période du 1er janvier 1944 au 28 février 1944, et je lis à la page 5:
Hoher Gerichtshof! Diese Zahl wurde am 16. Dezember 1941 genannt. Ich wende mich nun dem 25. Januar 1944 zu, also drei Jahre und einen Monat später, und beziehe mich auf einen anderen Auszug aus dem Tagebuch Franks, Dokument 2233-PS, ungebundener Band US-295. Dieser Band umfaßt den Zeitabschnitt vom 1. Januar 1944 bis zum 28. Februar 1944; auf Seite 5 des Originals heißt es:
205
В настоящее время в Генерал-губернаторстве вероятно находятся 100000 евреев.
At the present time we still have in the Government General perhaps 100,000 Jews.
Actuellement, nous avons encore dans le Gouvernement Général environ 100.000 Juifs.
Juden haben wir im Generalgouvernement zur Zeit vielleicht noch 100000.
206
В этот период 3 лет, согласно материалам Генерал-губернаторства оккупированной Польши, 2 400 000–3 400 000 евреев было уничтожено.
In this period of 3 years, according to the records of the then Governor General of Occupied Poland, between 2,400,000 and 3,400,000 Jews had been eliminated.
Au cours de ces trois ans, d’après les chiffres du Gouverneur Général de la Pologne occupée, environ 2.400.000 à 3.400.000 Juifs ont été exterminés.
In dieser Zeitspanne von drei Jahren wurden also nach dem Bericht des damaligen Generalgouverneurs für das besetzte Polen zwischen 2400000 und 3400000 Juden vernichtet.
207
Обвинение способно представить трибуналу достоверное доказательство количества евреев, которые погибли в нацистских руках, но это кажется кумулятивным доказательством которое не изменяет вины этих подсудимых.
The Prosecution could offer this Tribunal a wealth of evidence on the total number of Jews who died by Nazi hands, but it is believed that cumulative evidence would not vary the guilt of these defendants.
Le Ministère Public pourra présenter beaucoup de preuves concernant le nombre de Juifs morts aux mains des nazis, mais les preuves cumulatives ne pourront changer la culpabilité de ces accusés.
Die Anklagebehörde könnte dem Gerichtshof Unmengen von Beweismaterial über die Gesamtzahl der Juden, die durch die Hand der Nazis starben, vorlegen; doch glaube ich, daß zusätzliches Beweismaterial an der Schuld dieser Angeklagten nichts ändern würde.
208
В заключение я хочу представить документ, доказывающий, что 4 млн. евреев было уничтожено в лагерях и 2 млн. — умерщвлено полицией на Востоке, что составляет 6 млн. уничтоженных евреев. Документ ПС-2738, США-296, содержит изложение устного заявления Адольфа Эйхмана, начальника еврейской секции гестапо, который сообщил цифры, воспроизведенные в письменном показании доктора Вильгельма Хеттля, заместителя начальника главного управления иностранной разведки управления имперской безопасности. Доктор Вильгельм Хеттль показал следующее:
I do wish, however, to offer one document, a statement, to establish the deaths of 4 million Jews in camps and deaths of 2 million Jews by the State Police in the East, making a total of 6 million- Document 2738-PS, Exhibit USA-296. This is a statement-of Adolf Eichmann, Chief of the Jewish Section of the Gestapo, and the source of the figures quoted-made by Dr. Wilhelm Hoettl, Deputy Group Leader of the foreign section of the Security Police Amt IV of the RSHA. Dr. Wilhelm Hoettl, in affidavit form, made the following statement; and I quote from Page 2:
Je vais vous présenter néanmoins un document, une déclaration qui établit la mort de 4.000.000 de Juifs dans les camps et la mort de 2.000.000 de Juifs par la Police d’État de l’Est, formant un total de 6.000.000. Document PS-2738 (USA-296). Les chiffres cités émanent d’une déclaration d’Adolf Eichmann, chef de la section juive de la Gestapo faite par le Dr Wilhelm Höttl, chef adjoint du groupe de la section étrangère de la section de sécurité, AMT VI du RSHA. Le Dr Wilhelm Höttl fit la déclaration suivante sous forme d’affidavit et je cite la page 2:
Ich möchte jedoch ein Dokument vorlegen, eine Erklärung über den Tod von 4000000 Juden in Lagern und den Tod von 2000000 Juden durch die Hand der Staatspolizei im Osten, also eine Gesamtzahl von 6000000 Juden, Dokument 2738-PS, US-296. Diese Erklärung geht auf Adolf Eichmann, den Chef der Judenabteilung in der Gestapo, zurück, und von ihm rühren auch die zitierten Zahlen her. Die Erklärung selbst ist von Dr. Wilhelm Höttl, dem stellvertretenden Gruppenleiter des Auslandsamts des SD, des Amtes VI des RSHA, abgegeben. Dr. Wilhelm Höttl machte die folgende Aussage in Form einer eidesstattlichen Erklärung, und ich zitiere von Seite 2:
209
Примерно 4 миллиона евреев было истреблено в различных концлагерях. В то же время еще дополнительно 2 миллиона евреев умерщвлено другими путями. Большая часть их была расстреляна эйнзатцгруппами полиции безопасности в период кампании против России».
Approximately 4 million Jews had been killed in the various concentration camps, while an additional 2 million met death in other ways, the major part of which were shot by operational squads of the Security Police during the campaign against Russia.
Environ 4.000.000 de Juifs ont été tués dans les divers camps de concentration et 2.000.000 ont trouvé la mort autrement, la plus grande partie fusillée par des détachements de Police de sûreté pendant la campagne de Russie.
In den verschiedenen Vernichtungslagern seien etwa vier Millionen Juden getötet worden, während weitere zwei Millionen auf andere Weise den Tod fanden, wobei der Großteil davon durch die Einsatzkommandos der Sicherheitspolizei während des Feldzuges gegen Rußland durch Erschießen getötet wurde.
210
Я могу в завершение подчеркнуть, что захваченные документы, почти все без исключения исходят из официальных источников нацистской партии.
May I, in conclusion, emphasize that the captured documents in evidence are, almost without exception, from the official sources of the Nazi Party.
Puis-je pour terminer dire que les documents saisis et présentés comme preuve émanent presque sans exception d’une source officielle du parti nazi.
Darf ich zum Schluß betonen, daß die erbeuteten Dokumente, die als Beweismaterial vorliegen, fast ausnahmslos von den amtlichen Quellen der Nazi-Partei stammen.
79211
ПредседательВы прочли единственное заявление, но кто то лицо о котором даны письменные показания?
THE PRESIDENT You only read that one statement, but where does the person who made the affidavit get his information from?
LE PRÉSIDENT Vous n’avez lu qu’une déclaration. Mais où la personne qui a prêté serment a-t-elle obtenu ce renseignement?
VORSITZENDER Sie haben nur diese eine Erklärung vorgelesen; woher aber erhielt die Person, die diese eidesstattliche Erklärung abgab, ihre Information?
80212
УолшЯ буду рад прочитать это сэр. Я сделал заявление о том, что Эйхман является источником информации предоставленной Вильгельму Хоттлю, одним из его помощников и на странице 1 говорится:
MAJOR WALSH I shall be pleased to read that in there, Sir. I made a statement that Eichmann has been the source of the information given to Dr. Wilhelm Hoettl, one of his assistants, and on Page 1 it says:
COMMANDANT WALSH Monsieur le Président, je vais vous le lire avec plaisir. J’ai déclaré que Eichmann était à l’origine des précisions fourmes à Wilhelm Höttl, un de ses adjoints, il est dit, page 1:
MAJOR WALSH Ich will das gerne nachtragen. Ich habe erklärt, daß Eichmann die Nachrichtenquelle für Dr. Wilhelm Höttl, einen seiner Mitarbeiter, war. Auf Seite 1 heißt es:
213
Мне известно, что Эйхман был в то время начальником секции по еврейским вопросам гестапо и плюс к этому ему было поручено руководить арестом всех евреев в европейских странах и отправкой их в Германию лично Гиммлером. Эйхман тогда был весьма удручен тем фактом, что Румыния в те дни вышла из войны. Более того, он пришел ко мне, чтобы получить информацию относительно обстановки на фронте, которую я ежедневно получал от венгерского военного министра и от командующего войсками СС в Венгрии. Он выразил свою убежденность в том, что Германия проиграла войну и что для него все потеряно. Он знал, что Объединенные Нации будут рассматривать его как одного из главных военных преступников, так как он имел на своей совести миллионы загубленных еврейских жизней. Я спросил, сколько их было всего. Он ответил, что, хотя это было величайшим государственным секретом, он ответит на этот вопрос, так как для меня как для историка это составит интерес и так как он был убежден, что уже не вернется из Румынии. Он незадолго до этого составил отчет для Гиммлера, который хотел знать точно, какое число евреев он убил».
According to my knowledge, Eichmann was at the time a section leader in Amt IV (Gestapo) of RSHA; and in addition he had been ordered by Himmler to get hold of the Jews in all the European countries and to transport them to Germany. Eichmann was then very much impressed with the fact that Romania had withdrawn from the war in those days. Therefore, he had come to me to get information about the military situation, which I received daily from the Hungarian. . .Ministry of War and from the Commander of the Waffen-SS in Hungary. He expressed his conviction that Germany had lost the war and that he personally had no further chance. He knew that he would be considered one of the main war criminals by the United Nations, since he had minions of Jewish lives on his conscience. I asked him how many that was, to which he answered that although the number was a great Reich secret, he would tell me since I, as a historian too, would be interested and that probably he would not return anyhow from his command in Romania. He had, shortly before that, made a report to Himmler, as the latter wanted to know the exact number of Jews who had been killed.
À ma connaissance le chef de section de l’AMT IV (Gestapo) de RSHA était alors Eichmann. Il avait reçu de Himmler, l’ordre de saisir les Juifs de tous les pays d’Europe et de les transporter en Allemagne. Eichmann était alors très impressionné par le fait que la Roumanie s’était retirée de la guerre à cette époque. En ce qui concerne la situation militaire, je recevais tous les jours des informations du ministère de la Guerre hongrois et du commandement des Waffen SS en Hongrie et Eichmann venait se renseigner auprès de moi. Il exprima sa conviction que l’Allemagne avait perdu la guerre et que lui personnellement n’avait plus aucune chance. Il savait qu’il serait considéré comme Grand Criminel de guerre par les Nations alliées puisqu’il avait des millions de vies juives sur la conscience. Je lui en demandai le chiffre. Il me répondit que bien que le chiffre fût un très grand secret il me le dirait parce que, comme historien cela m’intéressait également et que, vraisemblablement, il ne reviendrait pas de son commandement en Roumanie. Peu de temps auparavant, il avait fait un compte rendu à Himmler qui voulait savoir exactement le nombre de Juifs qui avaient été tués.
Eichmann war zu diesem Zeitpunkte nach meinem Wissen Abteilungsleiter im Amte IV (Gestapo) des Reichssicherheitshauptamtes und darüber hinaus von Himmler beauftragt, in allen europäischen Ländern die Juden zu erfassen und nach Deutschland zu transportieren. Eichmann stand damals stark unter dem Eindruck des in diesen Tagen erfolgten Kriegsaustrittes Rumäniens. Deswegen war er auch zu mir gekommen, um sich über die militärische Lage zu informieren, die ich täglich vom ungarischen... Kriegsministerium und dem Befehlshaber der Waffen-SS in Ungarn bekam. Er gab seiner Überzeugung Ausdruck, daß der Krieg nunmehr für Deutschland verloren sei und er damit für seine Person keine weitere Chance mehr habe. Er wisse, daß er von den Vereinigten Nationen als einer der Hauptkriegsverbrecher betrachtet würde, weil er Millionen von Judenleben auf dem Gewissen habe. Ich fragte ihn, wieviele das seien, worauf er antwortete, die Zahl sei zwar ein großes Reichsgeheimnis, doch würde er sie mir sagen, da ich auch als Historiker dafür Interesse haben müßte und er von seinem Kommando nach Rumänien wahrscheinlich doch nicht mehr zurückkehren würde. Er habe kurze Zeit vorher einen Bericht für Himmler gemacht, da dieser die genaue Zahl der getöteten Juden wissen wollte.
214
Согласно этой информации я зачитал цитату.
It was on that basis of this information, Sir, that I read the following quotation.
C’est en me basant sur cette information que j’ai lu la citation de ce document.
Diese Information bildete die Grundlage für das folgende Zitat, das ich verlesen habe.
81215
ПредседательТрибунал откладывается.
THE PRESIDENT The Tribunal will adjourn now.
LE PRÉSIDENT Le Tribunal lève l’audience.
VORSITZENDER Der Gerichtshof vertagt sich nunmehr.
Объявлен перерыв до 14 часов
A recess was taken until 1400 hour.
L’audience est suspendue jusqu’à 14 heures.
Das Gericht vertagt sich bis 14.00 Uhr.
Помощник Главного обвинителя от США У. Уолш представляет сборник документов Т: «Преследование евреев».Продолжение от 13 декабря 1945 г. (14.12.1945)
1518
УолшВ ходе утреннего заседания суд запросил информацию о приобщённых документах и принял их в качестве доказательства. Я ссылаюсь на документ 1061-ПС, отчёт «Варшавского гетто больше нет». Этот отчёт, как мне сказали, был подготовлен для презентации на встрече руководителей СС и полиции 18 мая 1943. Это указано на странице 45 перевода у суда.
MAJOR WALSH During the morning session the Court requested certain information concerning documents that had been offered and accepted in evidence. I refer to Document 1061-PS, the report "The Warsaw Ghetto Is No More." This report, I am told, was prepared for presentation at a meeting of the SS Police leaders to be held on 18 May 1943. That is indicated on Page 45 of the translation before the Court.
COMMANDANT WALSH Au cours de la séance de ce matin, le Tribunal a demandé quelques précisions au sujet de documents fournis et admis comme preuves. Il s’agit du document PS-1061 (USA-275), le rapport: «Le ghetto de Varsovie n’existe plus.» Ce rapport, m’a-t-on dit, a été préparé en vue d’une réunion des chefs de la Police SS qui devait avoir lieu le 18 mai 1943. Ceci est indiqué à la page 45 de la traduction, qui est entre les mains du Tribunal.
MAJOR WALSH In der Vormittagssitzung verlangte der Gerichtshof Auskunft über einige Dokumente, die als Beweis vorgelegt und zugelassen wurden. Ich verweise auf Dokument 1061-PS, den Bericht:»Es gibt keinen jüdischen Wohnbezirk in Warschau mehr«. Dieser Bericht wurde, wie ich erfahren habe, vorbereitet, um bei einem Treffen der SS-Polizeiführer, das am 18. Mai 1943 gehalten werden sollte, vorgetragen zu werden. Dies ergibt sich aus Seite 45 der Übersetzung, die dem Gerichtshof vorliegt.
19
Этот документ был захвачен 7-й армией Соединённых Штатов и доставлен в Г-2 сил Соединённых Штатов на европейском театре. В свою очередь она доставила его полковнику Стори в штат обвинителей Соединённых Штатов, несколько месяцев тому назад. Суд также не заметил…
This document was captured by the 7th United States Army and delivered by them to the G-2 of the United States Forces in the European Theater. In turn they were delivered to Colonel Storey of the United States prosecutors stay, some months ago. The Court also ignored...
Il a été saisi par la 7e Armée américaine et a été remis par elle au Service G 2, des Forces des États-Unis dans le théâtre d’opérations européen; il fut remis ensuite au Colonel Storey du Ministère Public américain, il y a quelques mois.
Dieses Dokument wurde von der 7. US-Armee erbeutet und von dieser dem Nachrichtendienst der US-Armee im europäischen Operationsgebiet übergeben. Letzterer wiederum übergab es vor einigen Monaten Oberst Storey aus dem Stab der amerikanischen Anklagebehörde. Der Gerichtshof fragte auch...
1620
ПредседательМайор Уолш, я думаю, трибунал также желает знать можете ли вы сказать нам для кого был подготовлен отчёт?
THE PRESIDENT Major Walsh, I think the Tribunal also wished to know whether you could tell us to whom the report had been made?
LE PRÉSIDENT Commandant Walsh, je crois que le Tribunal voudrait aussi savoir à qui le rapport avait été adressé.
VORSITZENDER Major Walsh, ich glaube, der Gerichtshof möchte auch wissen, an wen der Bericht gerichtet war.
1721
УолшСэр, согласно телетайпам, ежедневные телетайпы, сэр, направлялись высшему руководителю СС и полиции Востока, обергруппенфюреру СС и генералу полиции Крюгеру или его заместителю.
MAJOR WALSH The report, Sir, according to the teletypes- the daily teletypes, Six-was addressed to the Higher SS and Police Leader East, SS Obergruppenfuehrer and General of the Police Kruger, or his deputy.
COMMANDANT WALSH Le rapport, d’après le télétype, était adressé au Chef suprême des SS et de la Police, Krüger, ou à son adjoint.
MAJOR WALSH Der Bericht, Herr Vorsitzender, war nach den Fernschreiben, den täglichen Fernschreiben, an den höheren SS- und Polizeiführer Ost, SS-Obergruppenführer und Polizeigeneral Krüger, oder seinen Vertreter gerichtet.
1822
Председатель Спасибо.
THE PRESIDENT Thank you.
LE PRÉSIDENT Je vous remercie.
VORSITZENDER Danke sehr.
23
COMMANDANT WALSH Le Tribunal a également demandé des précisions en ce qui concerne le document L-53. Ce document a été saisi par les forces «T» du détachement nº 220 du corps de contre-espionnage et trouvé parmi les rapports allemands saisis à Weimar, en Allemagne, avant le 10 mai 1945.
24
LE PRÉSIDENT Le L-53 n’est-ce pas?
1925
Уолш Суд также запрашивал о документе Л-53 и я получил некоторые сведения о документе. Этот документ был захвачен отрядом Т подразделения корпуса контрразведки номер 220, в числе германских материалов в Веймаре, Германия, где то до 10 мая 1945.
MAJOR WALSH The Court further inquired about Document L-53 and I have obtained some information concerning this document. This document was captured by T-Force of the Counter Intelligence Corps Detachment Number 220, found among the German records at Weimar, Germany, sometime prior to 10 May 1945.
COMMANDANT WALSH Oui.
MAJOR WALSH Der Gerichtshof verlangte auch Auskunft über Dokument L-53; und ich habe einige Informationen über diese Urkunde erhalten. Dieses Dokument wurde von der T-Gruppe des Abwehrdienstes, Abteilung Nummer 220, kurz vor dem 10. Mai 1945 erbeutet; und zwar wurde es unter den deutschen Akten in Weimar, Deutschland, gefunden.
26
Далее суд запросил, о документе, означающем буквы «ВБ». Я сожалею, что я не смог получить точную информацию о значении «ВБ», но мне предложили, что это означает Вестбунд или западные союзники, потому что оно используется в связи с захватом — уничтожением всех заключенных до захвата ВБ и Красной армией, и я полагаю, это означает Вестбунд.
The Court further inquired, concerning this document, the meaning of the letters "WB." I regret that I have been unable to obtain definite information as to the meaning of "WB" but it has been suggested to me that it might mean Westbund or Western Ally because it is used in connection with the capture-the destruction of all prisoners before capture by either the WB or the Red Armies, and I presume that it may mean Westbund.
D’autre part, le Tribunal a demandé la signification des lettres WB. Je regrette, mais je n’ai pu m’en informer de façon décisive. Mais on m’a suggéré que cela pouvait signifier «West-Bund» ou «Western Ally» (Allié de l’Ouest), parce que cela a trait au massacre de tous les prisonniers après leur capture par les WB ou l’Armée Rouge, et je présume que cela peut signifier «West-Bund».
Im Zusammenhang mit diesem Dokument verlangte der Gerichtshof eine weitere Auskunft über die Bedeutung der Buchstaben»W B«. Ich bedauere, daß ich keine definitive Erklärung über die Bedeutung der Buchstaben»W B« erhalten konnte. Es wurde die Vermutung geäußert, daß sie möglicherweise »Westbund«bedeuten, weil sie im Zusammenhang mit der Ergreifung, mit der Ermordung aller Gefangenen vor der Ergreifung entweder durch den WB oder die Rote Armee verwendet wurden. Deshalb nehme ich an, daß der Westbund damit gemeint ist.
27
Убийство евреев в Европе нельзя выразить только цифрами, так как воздействие этого убийства ещё более трагично для будущего еврейского народа и человечества. Древние еврейские сообщества со своей богатой духовной, культурной и экономической жизнью, веками связанные с народами в которых они процветали, были полностью уничтожены. Вклад еврейского народа в цивилизацию, искусства, науку, промышленность и культуру, не требует, рассмотрения в этом трибунале. Их уничтожение, проводимое продолжительно, осмысленно, запланировано и методично нацистами, представляет утрату цивилизацией особых качеств и способностей которые невозможно будет возродить.
The slaughter of the Jews in Europe cannot be expressed in figures alone, for the impact of this slaughter is even more tragic to the future of the Jewish people and mankind. Ancient Jewish communities with their own rich spiritual, cultural, and economic life, bound up for centuries with the life of the nations in which they flourished, have been completely obliterated. The contribution of the Jewish people to civilization, the arts, the sciences, industry, and culture, need not) I am sure, be elaborated upon before this Tribunal. Their destruction, carried out continuously, deliberately, intentionally, and methodically by the Nazis, represents a loss to civilization of special qualities and abilities that cannot possibly be recouped.
Le carnage des Juifs en Europe ne peut être exprimé seulement par des chiffres car l’influence de ce carnage est encore plus tragique pour l’avenir du peuple juif et de l’Humanité. D’anciennes communautés juives, avec leur vie culturelle très riche, leur développement spirituel et économique, liées depuis des siècles à la vie des nations dans lesquelles elles s’épanouissaient, ont été complètement anéanties. L’apport du peuple juif à la civilisation, aux arts, aux sciences, à l’industrie et à la culture n’a pas besoin, j’en suis sûr, d’être souligné devant ce Tribunal. Leur destruction, exécutée d’une façon continue, délibérément, intentionnellement et méthodiquement par les nazis, représente une perte pour la civilisation, perte de qualités et de valeurs bien définies qui ne peuvent être remplacées.
Das Gemetzel der Juden in Europa kann nicht allein in Zahlen ausgedrückt werden, denn die Wirkung dieses Blutbades stellt sich für die Zukunft des jüdischen Volkes und der Menschheit noch weit tragischer dar. Alteingesessene jüdische Gemeinden mit ihrem eigenen reichhaltigen geistigen, kulturellen und wirtschaftlichen Leben, Jahrhunderte hindurch mit dem Leben der Nationen verbunden, in denen sie sich entwickelten, sind vollständig ausgemerzt. Der Anteil des jüdischen Volkes an der Zivilisation, den Künsten, Wissenschaften, der Industrie und Kultur braucht sicherlich nicht besonders vor diesem Gerichtshof ausgeführt zu werden; ihre Vernichtung, von den Nazis in beständiger, vorsätzlicher, vorbedachter und methodischer Weise ausgeführt, bedeutet für die Zivilisation den Verlust besonderer Qualitäten und Fähigkeiten, der unmöglich ersetzt werden kann.
28
Я не пытался сосчитать множественные и дьявольские преступления совершённые против еврейского народа государством, которым правили эти подсудимые, потому что с печалью за нынешнюю и историческую истину, подробное описание некоторых этих преступлений выйдет за рамки границ человеческого восприятия. Разум уже ужаснулся и потрясён теми фактами на которые уже ссылались. Скорее моя цель заключается в том, чтобы прояснить путь, успешные и последовательные этапы и следствие совершенных преступлений, предопределённые средства предопределили конец.
I have not attempted to recount the multitudinous and diabolical crimes committed against the Jewish people by the state which these defendants ruled, because, with sober regard for contemporary and historical truth, a detailed description of some of these crimes would transcend the utmost reaches of the human faculty of expression. The mind already recoils and shrinks from the acceptance of the incredible facts already related. Rather, it is my purpose to elucidate the pattern, the successful and successive stages, the sequence and concurrence of the crimes committed, the pre-determined means to a pre-ordained end.
Je n’ai pas essayé de décrire les crimes multiples et diaboliques commis contre le peuple juif par l’État dirigé par ces accusés, parce qu’en respectant la vérité contemporaine et historique, la description détaillée de quelques-uns de ces crimes dépasserait les limites extrêmes de la faculté humaine d’expression. L’esprit recule et se refuse à admettre les faits incroyables déjà relatés. Mon but est plutôt de mettre en lumière la ligne générale, les étapes successives de la réussite, la suite et l’aboutissement des crimes commis, enfin les moyens prévus pour atteindre au résultat projeté.
Ich habe nicht versucht, die zahllosen und teuflischen Verbrechen, die gegen das jüdische Volk durch den Staat, den diese Angeklagten regierten, ausgeführt wurden, im einzelnen aufzuzählen, weil eine detaillierte Beschreibung einiger dieser Verbrechen auch bei ihrer nüchternen Beurteilung im Lichte zeitgenössischer und geschichtlicher Wahrheit den äußersten Rahmen menschlicher Ausdrucksmöglichkeit überschreiten würde. Der Verstand bebt und schaudert schon davor zurück, die bisher angeführten unglaublichen Tatsachen als wahr anzunehmen. Es liegt vielmehr in meiner Absicht, erläuternde Aufklärungen über die Schablone zu geben, die erfolgreichen und aufeinanderfolgenden Phasen, die Reihenfolge und das Zusammentreffen der begangenen Verbrechen und über die vorher beschlossenen Mittel zu einem vorher bestimmten Ende.
29
До сих пор, эти хладнокровные, жестокие факты и цифры, исходящие в основном из собственных источников подсудимых и представленные трибуналу, не имеют опровержения.
Yet, these cold, stark, brutal facts and figures, drawn largely from the defendants' own sources and submitted in evidence before this Tribunal, defy rebuttal
Pourtant, ces faits, ces chiffres nus, froids et brutaux, tirés pour la plupart des propres documents des accusés et présentés comme preuves à cette barre, défient la contradiction.
Und doch trotzen diese kalten, starren, brutalen Tatsachen und Zahlen, die zum großen Teile dem eigenen Quellenmaterial der Angeklagten entnommen und dem Gerichtshof hier als Beweis vorgelegt sind, jeder Widerlegung.
30
От концепции до исполнения, от программы партии 1920 до омерзительных заявлений Гиммлера и подсудимого Франка в 1943 и 1944, уничтожение еврейского народа в Европе являлось рукотворным.
From conception to execution, from the Party program of 1920 to the gloating declarations of Himmler and the Defendant Frank in 1943 and 1944, the annihilation of the Jewish people in Europe was man-made-made by the very men, sitting in the defendants' box, brought to judgment before this Tribunal.
Partant de la conception pour aboutir à l’exécution, du programme du parti en 1920 aux déclarations pompeuses de Himmler et de l’accusé Frank en 1943 et 1944, l’annihilation de la race juive en Europe a été faite par ces hommes, ceux-là mêmes qui sont assis au banc des accusés et appelés à être jugés devant ce Tribunal.
Von der Planung bis zur Ausführung, vom Parteiprogramm im Jahre 1920 bis zu den triumphierenden Erklärungen Himmlers und des Angeklagten Frank in den Jahren 1943 und 1944 war die Vernichtung des jüdischen Volkes in Europa Menschenwerk, ausgeführt von denselben Männern, die jetzt auf der Anklagebank sitzen und dem Urteil dieses Gerichtshofs unterworfen sind.
31
Перед завершением, могу я выразить признательность за безустанные услуги группы обвинения Соединённых Штатов, благодаря постоянному поиску, анализу, изучению, эта презентация доказательства стала возможной: капитан Сеймур Кригер, лейтенант Брэди Брайсон, лейтенант Фредерик Фелтон, сержант Исак Стоун и господин Ганс Натан.
Before closing may I acknowledge with appreciation the untiring services of the group of the staff of the United States' Prosecution, through whose painstaking search, analysis, and study, this presentation of evidence was made possible: Captain Seymour Krieger, Lieutenant Brady Bryson, Lieutenant Frederick Felton, Sergeant Isaac Stone, and Mr. Hans Nathan.
Avant de terminer, puis-je exprimer ma reconnaissance des services incessants rendus par le personnel du Ministère Public des États-Unis? Grâce à leurs recherches laborieuses, à leurs analyses et à leurs travaux, la présentation des preuves a été possible; je veux nommer: le capitaine Seymour Krieger, le lieutenant Brady Bryson, le lieutenant Frederic Felton, le sergent Isaac Stone, et Mr. Hans Nathan.
Bevor ich schließe, möchte ich noch meine Anerkennung für die unermüdliche Arbeit aussprechen, die eine Gruppe des Stabes der US-Anklagebehörde geleistet hat. Dank ihrer mühsamen Forschungsarbeit, ihren Analysen und Untersuchungen ist es möglich geworden, dieses Beweismaterial vorzulegen. Ich spreche meine Anerkennung besonders den folgenden Personen aus: Hauptmann Seymour Krieger; Leutnant Brady Bryson; Leutnant Frederik Felton; Sergeant Isac Stone und Herrn Hans Nathan.
2032
СториС позволения трибунала, следующая презентация о германизации и разграблении оккупированных стран, будет представлена капитаном Сэмом Гаррисом.
COL. STOREY If the Tribunal please, the next presentation, concerning Germanization and spoliation in occupied countries, will be presented by Captain Sam Harris.
COLONEL STOREY Plaise au Tribunal. La présentation suivante des preuves concernant la germanisation et la spoliation dans les pays occupés sera faite par le capitaine Harris.
OBERST STOREY Hoher Gerichtshof! Der nächste Teil der Anklage, der sich auf die Germanisierung und Ausplünderung der besetzten Gebiete bezieht, wird von Hauptmann Samuel Harris vorgetragen werden.